Sie sind auf Seite 1von 133

IV.

Personal Status and Capacity

JIMENEZ VS REPUBLIC

Republic of the Philippines


SUPREME COURT
Manila

EN BANC

G.R. No. L-24529 February 17, 1968

EDUARDO JIMENEZ, petitioner,


vs.
REPUBLIC OF THE PHILIPPINES and JUDGE PEDRO NAVARRO, Court of First Instance
of Rizal,respondents.

Office of the Solicitor General, Bengzon, Villegas & Zarraga and Alfredo R. Mabanag for
respondents.
Neptali A. Gonzales for petitioner.

ANGELES, J.:

On petition for writs of certiorari, prohibition and mandamus, with preliminary injunction, to
the Court of First Instance of Rizal, praying: (a) to review and thereafter annul the order of
respondent Judge denying petitioner's motion to set aside decision and promulgation thereof;
(b) to restrain respondent Judge from promulgating the decision; and (c) to direct respondent
Judge "to make re-examination of the evidence presented" during the trial of the case and to
render a decision upon the evidence.

The antecedent facts of the case which are not disputed, are the following:

Eduardo Jimenez, herein petitioner, together with others, was charged with homicide in
an information, dated May 13, 1960, before the Court of First Instance of Rizal, criminal case
No. 9531, of said court. The case was heard and tried before Judge Eulogio Mencias, presiding
one of the branches of the court. Admittedly, the decision prepared and signed by Judge
Mencias was delivered to the clerk of court on January 16, 1965. On the same date the clerk of
court issued and served notice on the petitioner to appear in court on January 21, 1965 for the
promulgation of the sentence. In view that January 21, was declared by the President a special
holiday, the promulgation of the decision could not be carried out on that day. On January 21,
1965, Judge Eulogio Mencias had reached the age of 70 and was retired on that day from the
bench. Respondent Judge Pedro Navarro was immediately designated to take the place of
Judge Mencias. The former judge ordered that the sentence be promulgated on January 29,
1965, but for some reason, it was postponed to March 1, 1965.

On March 1, 1965, petitioner Jimenez filed a motion to set aside decision and
promulgation thereof, on the following grounds: (a) "That the case was heard and tried by the
Hon. Eulogio Mencias and judgment was rendered by him before he retired on January 21,
1965, having reached the age of 70 years"; and (b) "That said judgment cannot be validly
promulgated since it is no longer the official act of a judge, either de jure or de facto."

The motion was opposed by the private prosecutor.

On April 2, 1965, the respondent Judge issued an order denying the motion, and ordered
that the decision be promulgated.

Hence, the instant petition.

The theory of the petitioner is, that for a decision to be validly promulgated, the same
must not only be rendered be a judge legally appointed and acting either as de jure or de facto,
but that the decision must also be promulgated during the incumbency of the judge who penned
the decision, reasoning that if the judge who penned the decision is no longer a judge at the
time of the promulgation, there is nothing that can legally be promulgated because the decision
is no longer an official act of the judge.

The Solicitor General, on the other hand, contends that, while he is in accord with the
view that for a decision to be validly promulgated there must be a judge legally appointed and
acting, either de jure or de facto, it is not necessary that the promulgation be made during the
incumbency of the judge who prepared and signed the decision, and the decision can be validly
promulgated as long as it was signed and delivered to the clerk of court for promulgation during
the judge's term of office.

Before the effectivity of the Revised Rules of Court, on January 1, 1964, the rule on
promulgation of judgment in criminal cases was Section 6, Rule 116 of the Rules of Court,
which reads thus:

Sec. 6. Promulgation of judgment. — The judgment is promulgated by reading the


judgment or sentence in the presence of the defendant and the judge of the court who
has rendered it. The defendant must be personally present if the conviction is for a grave
or less grave offense; if for light offense, the judgment may be pronounced in the
presence of his attorney or representative. And when the judge is absent or outside of
the province, his presence is not necessary and the judgment may be promulgated or
read to the defendant by the clerk of court. [Emphasis supplied]

After January 1, 1964, the rule now is Section 6, Rule 120 of the Revised Rules of Court,
which reads thus:

Sec. 6. Promulgation of judgment. — The judgment is promulgated by reading the


judgment or sentence in the presence of the defendant and any judge of the court in
which it was rendered.

If the defendant is confined or detained in another province or city, the judgment of


conviction may be promulgated by the judge of the Court of First Instance having
jurisdiction over the place of confinement or detention upon the request of the court that
rendered the judgment. The court promulgating the judgment shall have authority to
accept the notice of appeal and to approve the appeal bond. [Emphasis supplied]
In Ong Siu et al. v. Hon. Antonio P. Paredes, et al., G.R. No. L-21638, July 26, 1966, the
facts are as follows:1äwphï1.ñët

Four criminal cases, in which there were 6 accused, were tried jointly by Judge Sta. Maria
of the Municipal Court of Manila, and a single decision was rendered, under date of July 7,
1962. On July 9, 1962, before the decision could be promulgated, Judge Sta. Maria was
appointed to and assumed the position of judge of the Court of First Instance of Mindoro. Judge
German succeeded him as Municipal Judge of Manila. Two of the accused, Fung and Lu
petitioned the court that the unpromulgated decision of Judge Sta. Maria be declared null and
void. Judge German granted the petition, but before retrial of the cases could be had, Judge
German resigned. On August 23, 1962, Judge Paredes was appointed to the vacant position of
Municipal Judge of Manila. On the same day, he scheduled the promulgation of the decision of
Judge Sta. Maria. This was done with respect to the four accused, but not with regard to
defendants Fung and Lu who did not appear during the promulgation of the judgment. The latter
two instituted certiorari and prohibition proceedings in the Court of First Instance of Manila to
restrain the promulgation of the decision. The Court of First Instance of Manila granted the writ
on the ground that since Judge Sta. Maria was no longer a judge of the Municipal Court, the
decision, prepared and signed by him could no longer be validly promulgated. So Judge
Paredes of the municipal Court ordered a retrial of the four criminal cases which was set for
March 14, 1963. The four defendants Ong, Siu Sy So Ty, Francisco Ong and Lucio Ong, went
to CFI Manila and applied for a writ to restrain the Municipal Judge from retrying the four cases,
on the ground that as the decision acquitting them had already been promulgated with respect
to them, a retrial of the cases would subject them to double jeopardy for the same offense. The
CFI Manila dismissed the petition for the reason that the decision of Judge Sta. Maria being
invalid because its promulgation was effected when the judge had already ceased to be a
municipal judge, the same cannot place the defendants twice in jeopardy for the same offense.
The four defendants appealed to the Supreme Court. In affirming the appealed decision of the
Court of First Instance, the Supreme Court in applying Sec. 6, Rule 120 of the Revised Rules,
said:

The appellants in effect contend that since the decision of Judge Sta. Maria was
signed by him while he was still the judge of the Municipal Court of Manila where they
were tried, its promulgation, although made in his absence, was valid. In support of this
contention, they cite Section 6 of Rule 116 (now Rule 120) of the Rules of Court, which
reads:

Sec. 6. Promulgation of judgment. — The judgment is promulgated by


reading the judgment orsentence in the presence of the defendant and any judge
of the court in which it was rendered. The defendant must be personally present
if the conviction is for a grave offense; if for a light offense, the judgment may be
pronounced in the presence of his attorney or representative. And when the
judge is absent or outside of the province or city, his presence is not necessary
and, the judgment may be promulgated or read to the defendant by the clerk of
court. . . . [Emphasis supplied.]

Pursuant to the above-quoted provisions, the petitioners-appellants argue, the


decision of Judge Sta. Maria was promulgated in the presence of Judge Paredes,
another judge of the Municipal Court. They claim that the absence of Judge Sta. Maria
during the promulgation does not render the decision he penned prior to his appointment
to the position of judge of the court of first instance null and void. Thus, it is alleged, the
promulgation thereof, upon order of Judge Paredes, was valid and could be the basis of
the defense of double jeopardy.

The above-quoted Section 6 of Rule 116 (now Rule 120) of the Rules of Court,
allowing the dispensability of the presence of the judge in the reading of a
sentence refers only to the physical absence of the judge, and not to his inability to be
present during the promulgation of the judgment because of the cessation of or his
removal front office. This is clear from the use of the disjunctive clause "absent or
outside of the province or city" in the provision. In other words, the decision of the judge
may be promulgated even without his presence, as long as he is still a judge of that
court (Luna v. Rodriguez, 37, Phil. 186; Garchitorena v. Criscini 37 Phil. 675; Barredo v.
Commission on Elections, 45 O.G. 4457; People v. Court of Appeals, G.R. Nos. L-9111-
9113, Aug. 28, 1956; People v. So, G.R. No. L-8732, July 30, 1957).

In the present case, what we have is not merely physical absence of the judge
who penned the decision, but the cessation of termination of his incumbency as such
judge. In the case of People v. Bonifacio So y Ortega, (G.R. No. L-8732, supra) this
Court ruled:

It is well-settled that to be binding a judgment must be duly signed, and


promulgated during the incumbency of the judge who signed it.

In Lino Luna v. Rodriguez, supra, Judge Barreto signed his decision on


January 14; two days later (January 16), he qualified as Secretary of Finance
thereby retiring from the Judiciary; and on January 17, his decision was
promulgated. This Court held such decision to be void, because at the time of the
promulgation the judge who prepared it was no longer a judicial officer.

In criminal proceedings the Rules are more explicit. They require the
judgment to be promulgated by reading the judgment or sentence in the
presence of the defendant and the judge of the court who has rendered it. (Rule
116, Sec. 6); and although it is true that it may be read by the clerk "when the
judge is absent or outside the province," it is implied that it may be read, provided
he is still the judge therein.

It is contended that herein decision was promulgated to all intents and


purposes, when it was delivered to the clerk for promulgation — June 18. That
contention was, however, indirectly overruled in People v. Court of Appeals, a
case similar to this wherein we regarded compliance with Sec. 6 of Rule 116 as
essential to promulgation, and held that as the judgment was
promulgated after the judge who penned it had ceased to be judge, it was not
legally binding.

It is true that in Cea v. Cinco (50 O. G. 5254) this section was interpreted
to mean that where judgment is one of acquittal, "reading in the presence of the
defendant" may be substituted by giving a copy of the decision to him. We
declared that such act — delivery of copy — amounted to promulgation. In the
case before us, notice that the decision would be read (on June 30) was sent out,
while Judge Encarnacion was still a judge. Yet no copy of such decision was
given the accused, and he was not informed thereof during said judge's
incumbency. No judgment was therefore validly entered. (Cf. Landicho v. Tan, 48
O.G. 1007).

Here, in the present case, when the notice for the promulgation of the decision
was sent out, the judge who signed the decision was no longer the judge of the court,
and no copy of the judgment of acquittal was delivered to the appellants. With more
reasons, therefore, is there no judgment validly entered in this case.

UPON THE FOREGOING CONSIDERATIONS, We hold that the decision rendered by


the retired Judge Eulogio Mencias cannot be validly promulgated and acquire a binding effect
for the same has become null and void under the circumstances.

Concepcion, C.J., Reyes, J.B.L., Dizon, Makalintal, Bengzon, J.P., Zaldivar, Sanchez, Castro
and Fernando, JJ., concur.

RECTO VS JARDEN

Republic of the Philippines


SUPREME COURT
Manila

EN BANC

G.R. No. L-22174 July 21, 1967

ESPERANZA P. DE HARDEN, plaintiff, vs. FRED M. HARDEN, ET AL., defendants.

AURORA R. DE RECTO, Administratrix of the Estate of Claro M. Recto, claimant-appellee,


vs. JOSE SALUMBIDES, oppositor-appellant.

BENGZON, J.P., J.: virtual law library

Fred Harden, an American citizen, and Esperanza Perez were married in the Philippines on
December 14, 1917. They lived together, acquiring considerable conjugal properties, until 1938
when they separated. In July 1941, Mrs. Harden hired the late Claro M. Recto as her counsel in
the suit she was contemplating to file against her husband. In their contract, she agreed, inter
alia, to pay Recto 20% of her share in the conjugal partnership. On July 12, 1941, Mrs. Harden,
thru Recto, filed her complaint for administration and/or accounting of the conjugal properties
against Mr. Harden, and Jose Salumbides, herein oppositor-appellant, as his attorney-in-fact.
The war suspended the proceedings. After liberation, the records of the case were reconstituted
and on November 20, 1946, the conjugal properties of the Harden spouses were placed under
receivership. On October 31, 1949, the lower court rendered judgment for Mrs. Harden. Mr.
Harden appealed to this Court1 and then left the Philippines. Mrs. Harden must have followed
her husband for in January 29, 1952, an amicable settlement was effected between them in
Canada. As a consequence thereof, Recto was instructed by Mrs. Harden to discontinue the
proceedings.virtualawlibrary virtual law library
On February 20, 1952, Recto filed a motion in the Supreme Court to establish his attorney's
charging lien. The Hardens opposed. This Court, by resolution dated July 22, 1952, remanded
the case to the trial court to determine the amount of Recto's attorney's fees. But all the ancillary
writs and processes issued in the case were dissolved except the receivership on the conjugal
properties, which was maintained. Subsequently, the lower court, after hearing, held that Recto
was entitled to P384,110.97 as counsel fees. Mrs. Harden appealed to this Court2 which upheld
Recto but modified the amount of P304,110.97 only.virtualawlibrary virtual law library

On January 22, 1957, Recto moved for execution of the judgment. The lower court having
granted the motion, the Hardens went on certiorari3 to this Court. We dismissed the petition on
August 2, 1957 for lack of merit. Recto was then able to secure an alias writ of execution. Again
this was questioned on certiorari4 by the Hardens in this Court. On February 10, 1958, We
upheld Recto once more. This finally enabled the latter to levy upon the stocks and other
properties of the Hardens, the public sales of which realized P100,805.00. A balance of
P203,305.97 thus remained in Recto's favor.virtualawlibrary virtual law library

On July 2, 1958, Recto moved ex parte to levy on other shares of stock owned by the Hardens
but registered in the name of Salumbides, including the 410,638 shares in the Surigao
Consolidated Mining Co. Upon being notified that the 410,638 Surigao shares, inter alia, were to
be sold a t public auction, Salumbides filed an opposition claiming that he owned said shares,
the same being registered in his name. This was denied. His motion to reconsider the denial
also met the same fate, the lower court holding that Salumbides did not own the said Surigao
shares of stock. Whereupon, Salumbides appealed to this Court. 5 We dismissed the same on
December 22, 1958 for being frivolous. The motion to reconsider subsequently filed failed to
save the appeal. On April 21, 1959, the said 410,638 shares were sold at public auction for
P147,679.97 [sic] leaving an unsatisfied judgment balance of P55,624.00 in Recto's
favor.virtualawlibrary virtual law library

The next incident concerns the return to the receiver of the P20,581.90 cash dividends from
December 14, 1955 to December 14, 1956, received by Salumbides on the same 410,638
Surigao shares. As early as April 4, 1957, Recto had already moved that Salumbides be
ordered to deliver to the receiver all the dividends from the said shares which were under
receivership. On July 1, 1957, the lower, court issued an order requiring Salumbides to "turn
over to the receiver x x x all the dividends he has already received from the Surigao Mining
Company, Inc." Salumbides' motion to reconsider this order was denied.virtualawlibrary virtual
law library

On February 10, 1958, Recto moved for a writ of execution to implement the order of July 1,
1957. This was approved on February 21, 1958. Salumbides filed a motion to reconsider,
claiming that he owned the dividends pertaining to the 410,638 shares. On July 30, 1959, the
lower court ordered Salumbides to comply with the order of July 1, 1957 by depositing
P20,531.90 in the Commercial Bank & Trust Co. The latter moved for reconsideration
alleging, inter alia, that he had spent P45,900.99 as expenses for the Hardens from 1955 to
1957 and for which he must be reimbursed. When this was denied, a second motion to
reconsider was filed, Salumbides claiming that the P20,531.90 cash dividends had already been
disbursed for the benefit of the Harden family. On August 29, 1961, the lower court, after
hearing and presentation of evidence, denied the second motion to reconsider, holding that the
alleged incurring of expenses by Salumbides was a mere afterthought concocted by
him.virtualawlibrary virtual law library
Preliminary steps were taken by Salumbides to appeal this order. Meanwhile, on October 2,
1960, Recto died and his wife, as his administratrix, was substituted as claimant. On October 7,
1961, the lower court required Salumbides to submit a P25,000.00 supersedeas bond to
prevent execution pending appeal. This compelled Salumbides to abandon the intended appeal.
On October 23, 1961, he deposited P20,531.90 in the bank in compliance with the order of
August 29, 1961. On November 21, 1961, Mrs. Recto, with court approval, withdrew P25,000.00
from the Harden funds under receivership in the bank, thus reducing the judgment balance to
P30,624.00.virtualawlibrary virtual law library

On November 27, 1961, Mrs. Recto moved for full compliance with the order of July 1, 1957 to
satisfy the remaining judgment balance, relying upon a statements 6 issued by the Surigao
Consolidated that from April 15, 1950 to July 2, 1955, Salumbides had received all the cash
dividends on the 410,638 shares, amounting to P60,797.29. Resolving the motion and
opposition interposed by Salumbides, the lower court on December 11, 1962 ordered
Salumbides to deposit P30,624.00 in the Commercial Bank and Trust Company for final
satisfaction of the judgment balance in Recto's favor. This is the incident under the present
appeal, first taken to the Court of Appeals but subsequently certified to Us.virtualawlibrary virtual
law library

Appellant Salumbides first submits that the order of July 1, 1957 which is sought to be fully
enforced did not include the cash dividends received by him before December 14, 1955 since
Recto's motion of April 4, 1957 was limited to those dividends received after said date. This is
without merit. The dispositive portion of the order of July 1, 1957, which reads:

Finding the said petition to be well founded this Court hereby orders Jose Salumbides to
turn over to the Receiver, Atty. Juan S. Ong all the dividends that he has already
received from the Surigao Consolidated Mining Company, Inc.

clearly includes all dividends received as of then by Salumbides. The Surigao Consolidated
statement dated April 5, 1957 shows that the cash dividends on the 410,638 shares from April
15, 1950 to July 1955 had also been delivered to and already received by Salumbides. And the
lower court found, in its order of August 29, 1961, that Salumbides never appealed the order of
July 1, 1957. Hence, the same can no longer be questioned now.virtualawlibrary virtual law
library

Salumbides would also argue that those dividends had already been disbursed by him for the
benefit of the Harden family. This question, however, had already been raised and
argued twicebefore the lower court which tried and decided it adversely in the order of August
29, 1961. Although Salumbides filed his notice of appeal and appeal bond, the appeal was
never really pursued. In fact, on October 23, 1961, he manifested to the lower court that he had
already complied with the order of August 29, 1961, thus making the same final and conclusive
as against him.virtualawlibrary virtual law library

The defenses of (a) bar by prior judgments, (b) prescription, extinctive and acquisitive, (c)
laches, and (d) waiver, set up by Salumbides, are without merit. For the first, he would rely upon
the lower court's orders of December 7, 1953 and January 24, 1956, which declared that the
receivership did not include future dividends on the shares of stock. But the more recent order
of August 29, 1961 expressly declared these orders erroneous and already superseded and
reversed by the later court orders of December 14, 1955, July 1, 1957 and February 21,
1958.virtualawlibrary virtual law library
There could be no prescription, extinctive or acquisitive. Even if the period for bringing the
action be five years as appellant suggests, still the same has not yet lapsed. The dividends
being litigated were declared from April 15, 1950 to July 2, 1955. But the receiver's letter of May
9, 19537 asking for the dividends and claimant's motions of November 4, 1953, December 15,
1955, April 4, 1957, February 10, 1958 and November 27, 1961, to the same effect, seasonably
interrupted the prescriptive period. These extra-judicial and judicial demands also negative
laches on claimant's part.virtualawlibrary virtual law library

Salumbides could not acquire the dividends in question by prescription since he possessed
them, not in concept of owner, adverse to the Hardens, but rather as attorney-in-fact of Mr.
Harden. He first claimed ownership only in his omnibus opposition dated July 1, 1957. But two
years later, or on August 24, 1959, in his motion to reconsider, Salumbides admitted that these
dividends belonged to the Hardens.virtualawlibrary virtual law library

Neither is Recto's demand for the P20,531.00 cash dividends which were declared from
December 14, 1955 to December 14, 1956, a waiver of the previous dividends. He merely
wanted to satisfy his judgment credit from among any of the Harden assets available. Since the
later dividends failed to fully satisfy the judgment, Recto could still enforce his valid claim
against the previous dividends. As to the cash dividend of October 3, 1955, the order of
December 14, 1955 is very clear that it "shall not constitute a precedent with respect to the
disposition of all dividends whether already declared or to be hereinafter declared." The defense
of waiver, therefore, fails.virtualawlibrary virtual law library

Lastly, appellant would insist that upon the death of Mr. Harden in Canada on May 1, 1959, or
during the pendency of the proceedings, Recto's claim should have been forthwith dismissed
and filed in the administration proceedings of Mr. Harden's estate. But appellant erroneously
assumes that Recto's claim is a "money claim" under the Rules 8 when it is neither a claim nor a
judgment for money directed against the decedent, Mr. Harden. Recto's claim is founded on a
personal obligation of Mr. Harden. But granting that Recto's claim is a money claim against Mr.
Harden, that would not help appellant any. We have already ruled 9 that a charging lien
established on the property in litigation to secure payment of attorney's fees partakes of the
nature of a collateral security or of a lien on real or personal property, the enforcement of which
need not be made in the administration proceedings.virtualawlibrary virtual law library

Wherefore, the order appealed from is hereby affirmed. Costs against oppositor-appellant. So
ordered.

Reyes, J.B.L., Makalintal, Zaldivar, Sanchez, Castro, Angeles and Fernando, JJ., concur.
Concepcion, C.J. and Dizon, J., are on leave.

Endnotes:
1
Harden v. Harden, L-3687.
2
In the Matter of the Claim for Attorney's Fees - Recto v. Harden, L-6897, Nov.
29, 1956.
3
Harden v. Bayona, L-12611.
4
Harden v. Bayona, L-13386.
5
Harden v. Harden, Recto v. Salumbides, L-14751.
6
Record on Appeal, pp. 466-467.
7
Record on Appeal, pp. 28-29.
8
Sec. 5, Rule 86, Rev. Rules of Court.
9
Olave V. Canlas, L-12707, Feb. 28, 1962.

BARNUEVO VS FUSTER

Republic of the Philippines


SUPREME COURT
Manila

EN BANC

G.R. No. L-7487 December 29, 1913

CONSTANZA YAÑEZ DE BARNUEVO, plaintiff and appellant,


vs.
GABRIEL FUSTER, defendant and appellant.

O'Brien & DeWitt for plaintiff.


Chicote & Miranda for defendant.

JOHNSON, J.:

On the 7th of February, 1875, Gabriel Fuster and Constanza Yañez were joined in a Catholic or
canonical marriage in the city of Malaga, Spain. In February of 1892, Gabriel Fuster came to the
Philippine Islands, settled, and acquired real and personal property. Toward the middle of 1896,
Constanza Yañez came to Manila, where her husband was residing, and here lived with him in
conjugal relations until the month of April, 1899. On the 4th day of that month and year they
made an agreement, in a public document, by which they "resolved to separate and live apart,
both consenting to such separation, and by virtue thereof the husband authorized the wife to
move to Spain, there to reside in such place as the said lady pleases." (B. of E., p. 13.) In the
same document, the husband undertook to send his wife the sum of 300 pesetas monthly for
her support, payable in Madrid, Spain, from the month of June of the said year 1899. The
husband complied with this obligation until August, 1899, after which time he ceased to make
further payments.

In the beginning of March, 1909, the wife returned to the Philippines, but the husband had
absented himself therefrom in the early days of February of the same year. On the 11th of
March, 1909, the wife commenced divorce proceedings against her husband, alleging as cause
of action the adultery committed by him in or about the year 1899 with a certain woman that she
named in the complaint and with whom he had lived and cohabited and by whom he had had
two children. She prayed that she be granted a decree of divorce; that the court order the
separation of the properties of the plaintiff and the defendant, to date from the date of the said
decree; that the conjugal society be therefore liquidated, and after the amount of the conjugal
property had been determined, that one-half thereof be adjudicated to her; furthermore, as to
the amount of pension owing for her support but not paid to her, that the defendant be ordered
to pay her the sum of 36,000 Spanish pesetas, that is, 7,220 Spanish dollars, which, reduced to
Philippine currency at the rate of exchange on the date of the complaint, amounted to
P12,959.90.

The defendant denied that either he or his wife was a resident of the city of Manila, as they had
their domicile in Barcelona, Spain, and he alleged that both of them were natives and subjects
of Spain. He admitted that he was married to Constanza Yañez; he also admitted having
executed the document of the 4th of April, 1899, in which he had undertaken to make an
allowance for the support of his wife in Madrid, but he denied the other paragraphs of the
complaint. As a special defense with regard to the allowance, he alleged: "That in or about the
month of May, 1900, he wrote to his wife, the plaintiff, instructing her to return to Manila, with a
view of joining her husband and being maintained by him in his own house; that the
communication was ignored by the plaintiff, who against the will of the defendant, continued to
live separately from him that from the year 1901, the defendant did not know her address; that
since 1900, the plaintiff has lived in comfort and has known where her husband resided; that the
plaintiff, during all of the time referred to, in addition to dispossing of valuable property belonging
to her husband, possessed and still possesses property of her own, acquired by her, in greater
amount than that owned by her husband; and that in any case the action has prescribed by
operation of law."(B. of E., pp. 7 and 8.) As to the divorce, he admits that he had by the plaintiff
two children that have died. He expressly denied the contents of paragraph 5 of the complaint,
relating to the charge of adultery and also those of paragraphs 6, 7, and 8, concerning the
possession of real and personal property of the conjugal partnership, the statement of their
amount, and their qualification as being all conjugal property. As a special defense, he alleged
that prior to the year 1899 he conferred powers of attorney upon the plaintiff to administer and
collect property and credits pertaining to him to the value of about 200,000 pesos; that the
plaintiff accepted and exercised the said power of attorney, attached the property and collected
the credits without ever having rendered any account of them. As a special preferred defense,
he alleged that neither the trial court nor any other court in the Philippine Islands has jurisdiction
over the subject matter of the complaint, because, as to the allowance for support, since neither
the plaintiff nor the defendant are residents of Manila, or of any other place in the Philippine
Islands, the agreement upon the subject was neither celebrated, nor was it to be fulfilled, in the
Philippine Islands; and as to the divorce, because the action therefore ought to be tried by the
ecclesiastical courts. In conclusion, he prayed that the court find: That the court was without
jurisdiction over the two causes of action; that even if it had jurisdiction, it could not order the
payment of the sum claimed as arrears of alimony; that, after all, the action with regard to this
cause of action has prescribed; and as to the prayer for a decree of divorce, the defendant
should be acquitted, while on the other hand the plaintiff should be required to render to the
defendant an accounting, supported by proofs, of her operations as his attorney and
administratrix of his property in Spain.

In deciding the case, the Court of First Instance of the city of Manila held itself to have
jurisdiction, decreed the suspension of life in common between the plaintiff and defendant,
ordered the latter to pay the former P5,010.17, directed that the communal property be divided
between the parties, with costs against the defendant, and in event that the parties could not
agree to the division, it was to be effected by commissioners according to law.

Both parties appealed from this judgment, but notwithstanding the appeal, the partition of the
property, by means of commissioners, was proceeded with. These latter, after various
vicissitudes, rendered their report and account of the partition to the court, who then rendered
final judgment, from which, also, both parties appealed.

I. DEFENDANT'S APPEAL.

The first error assigned is the utter lack of jurisdiction of the trial court and of all other courts of
the Islands to try the case, either with regard to the fulfillment of the contract to furnish alimony,
or to decree a divorce or suspension of life in common between the spouses: lack of jurisdiction
over the persons and over the subject matter of the litigation; and over the persons of the
contending parties, because neither of the spouses was a resident of the Philippines on the date
of the complaint.

The lower court did not commit this error attributed to him. The defendant had not proved that
he had elsewhere a legal domicile other than that which he manifestly had in the Philippines
during the seventeen years preceding the date of the complaint. On the contrary, it plainly
appears, without proof to the contrary, that during this not inconsiderable period, extending from
the year 1892 until a month prior to the arrival of his wife in the Philippines in March, 1909, he
had constantly resided in the said Islands, had kept open house, and had acquired in the city of
Manila quite a little real property which is now the object of the division of the conjugal society. It
is also plainly shown, without proof to the contrary, that his wife resided in this city of Manila
from the middle of 1896 until April, 1899, at which time she was permitted by him to change her
residence. It is affirmed by the defendant in point five of his answer to the complaint, that in
May, 1900, he sent a letter instructing the plaintiff to return to Manila to live with her husband
and to be supported by him in his house, but that the plaintiff, against the will of the defendant,
continued to live part from him. (B. of E., p. 7.) It is also affirmed in the said answer, that during
all of the time referred to in the complaint, and especially since 1900, the plaintiff knew where
her husband resided. (B. of E., p. 7.) It is also very evident that the contract, by virtue of which
he authorized his wife to move to Spain and residethere in such place as was agreeable to her,
was executed in these Islands, "in the city of Manila on the 4th of April, 1889," as is to be seen
in the heading of the document. (B. of E., p. 12.) Finally, at page 11 of his brief, he says that the
record shows him to be a Spanish subject, inscribed in the consulate of his nation, and cities
article 26 of the Civil Code, the Treaty of Paris and the Philippine Bill.

Granting these facts, there can be no doubt that the defendant, although a Spanish subject, was
a resident of these Islands. Article 26 of the Civil Code that he cites itself provides that
"Spaniards who change their domicile to a foreign country, where they may be considered as
natives without other conditions than that of residents therein, shall be required, in order to
preserve the Spanish nationality, to state that such is their wish before the Spanish diplomatic or
consular agent, who must record them in the registry of Spanish residents, as well as their
spouses, should they be married, and any children they may have." From this provision, which
is the exclusive and irrefutable law governing the defendant, we are to conclude that the
domicile of the defendant and the plaintiff is fully proven, irrespective of the Treaty of Paris.
Without this supposition of having acquired his domicile and residence in these Islands, he
could not have required his wife to return to live with him therein because this requirement could
only be based on articles 58 of the Civil Code of Spain, according to which the wife is obliged to
follow her husband wherever he wishes to establish his residence, or on article 48 of chapter 5
of the Marriage Law in force in the Philippines, which imposes upon the wife the duty of obeying
her husband, living in his company, or of following him to wherever he transfers his domicile or
residence. And just because he was absent for a month before his wife returned to the
Philippines, he cannot be understood to have surrendered his habitual domicile of more than
seventeen years, without having established any other afterwards, and without making any
declaration in legal form, before he absented himself, of it being his intention to change his
domicile, while at the same time he retains here his house, real property and all manner of
means of subsistence. Section 377 of the Code of Civil Procedure leaves to the election of the
plaintiff the bringing of a personal action like the one at bar either in the place where the
defendant may reside or be found, or in that where the plaintiff resides.

The litigating spouses have gained not only domicile (domicilio) but also residence (vecindad) in
Manila. In this litigation the defendant claims that, born as he says in Mallorca, in the Balearic
Islands, he is not subject, in his marriage, to the rules governing conjugal property, that are in
force in the territories of Spain that are governed by the common law of Castillo (as the
Philippines in their day), because they are opposed to the Foral Law in force in the said Islands
and which is respected by the Civil Code. Even if this defense could be sustained herein,
paragraph 2 of article 15 of the said Civil Code would be applicable. It provides: "For the
purposes of this article, residence (vecindad) will be acquired: By residence of ten years
in common law provinces or territories, unless before the termination of that time he manifests
his will to the contrary; or by a residence of two years, if the interested person declares this to
be his will . . . In any case, the wife will follow the condition of her husband. . . ." On no occasion
had the defendant manifested his will to the contrary, not even as he was leaving, after a
residence of seventeen years, a month before the return of his wife to these Islands. On the
contrary, when he inscribed himself in the Spanish consulate, he declared his intention of
continuing to reside in the Islands as a Spaniard and not as a Mallorquin, subject as such to the
common law of Spain.

In an endeavor to demonstrate the lack of jurisdiction of the courts of these Islands over the
subject matter of the complaint that is to try an action for divorce between two Catholic
Spaniards, he alleges in his appeal: That both litigants are Spanish subjects and that they
contracted a Catholic marriage; that in accordance with article 9 of the Civil Code of Spain (the
same as that of these Islands) the laws relating to family rights and duties, or to the status,
condition and legal capacity of persons, govern Spaniards although they reside in a foreign
country; that, in consequence, "all questions of a civil nature, such as those dealing with the
validity or nullity of the matrimonial bond, the domicile of the husband and wife, their support, as
between them, the separation of their properties, the rules governing property, marital authority,
division of conjugal property, the classification of their property, legal causes for divorce, the
extent of the latter, the AUTHORITY to decree it, and, in general, the civil effects of marriage
and divorce upon the person and properties of the spouses, are questions that are governed
exclusively by the national law of the husband and wife, and, in our case, by the Spanish law by
virtue of article 9 as above set out." (Brief, p. 12.) The appellant and defendant continues his
argument, saying: That by the express provision of article 80 of the Civil Code of Spain,
"jurisdiction in actions for divorce and nullification of canonical marriages lies with ecclesiastical
courts," while that of civil tribunals is limited to civil marriages; that this being so, the action for
divorce brought by the plaintiff in the cause does not fall within the jurisdiction of the civil courts,
according to his own law of persons, because these courts ought to apply the Spanish law in
accordance with the said article 9 of the Civil Cod of Spain, and this Spanish law grants the
jurisdiction over the present cause to the ecclesiastical courts, in the place of which no tribunal
of these Islands con subrogate itself. Says this appellant: "If a law of a foreign country were of
rigorous application in a given case, a North American tribunal would have no jurisdiction upon
an ecclesiastical court and therefore the North American tribunal in applying it would have to
exercise a faculty which that law reserved to the ecclesiastical court." (Brief, pp. 13, 14, and 15.)

Unless we take the question itself for granted, the foregoing reasoning cannot be upheld. The
question is precisely whether the courts of the Philippines are competent or have jurisdiction to
decree the divorce now on appeal, and it is taken for granted that the power to decree it is one
of the rights included in the personal statute, but appellant does not prove by any law or legal
doctrine whatever that the personal statute of a foreigner carries with it, to whether he transfers
his domicile, the authority established by the law of his nation to decree his divorce, which was
what he had to demonstrate.

The authority of jurisdictional power of courts to decree a divorce is not comprised within the
personal status of the husband and wife, simply because the whole theory of the statutes and of
the rights which belong to everyone does not go beyond the sphere of private law, and the
authority and jurisdiction of the courts are not a matter of the private law of persons, but of the
public or political law of the nation. "The jurisdiction of courts and other questions relating to
procedure are considered to be of a public nature and consequently are generally submitted to
the territorial principle. . . . All persons that have to demand justice in a case in which foreigners
intervene, since they can gain nothing by a simple declaration, should endeavor to apply to the
tribunales of the state which have coercive means (property situated in the territory) to enforce
any decision they may render. Otherwise, one would expose himself in the suit to making
useless expenditures which, although he won his case, would not contribute to secure his rights
because of the court's lack of means to enforce them." (Torres Campos, "Elementos de
Derecho International Privado," p. 108.) "Justice," says the same professor, "is a principle
superior to that of nations, and it should therefore be administered without taking into any
account whatsoever the state to which the litigants belong. . . . In order to foster their relations
and develop their commerce, all civilized nations are interested in doing justice, not alone to
their own people, but to those foreigners who contract within the country or outside of it juridical
ties which in some manner effect their sovereignty. (Ibid, p. 107.) Might its courts, in some
cases, in suits between foreigners residing in its territory, apply the personal law of the parties,
but abdicate their jurisdiction, refrain from administering justice because the personal law of the
foreigner gave the jurisdiction of the given case to some court that is not the territorial one of the
nation? This has never yet been claimed in any of the theories regarding the conflict of laws
arising out of questions of nationality and domicile; it would be equivalent to recognizing
extraterritorial law in favor of private persons. The provisions of article 80 of the Civil Law of
Spain is only binding within the dominions of Spain. It does not accompany the persons of the
Spanish subject wherever he may go. He could not successfully invoke it if he resided in Japan,
in China, in Hongkong or in any other territory not subject to the dominion of Spain. Foreign
Catholics domiciled in Spain, subject to the ecclesiastical courts in actions for divorce according
to the said article 80 of the Civil Code, could not allege lack of jurisdiction by invoking, as the
law of their personal statute, a law of their nation which gives jurisdiction in such a case to
territorial courts, or to a certain court within or without the territory of their nation.1awphi1.net

It is a question that has already been settled in two decisions of the Supreme Court (Benedicto
vs. De la Rama, 3 Phil. Rep., 34, and Ibañez vs. Ortiz, 5 Phil. Rep., 325).

In the present action for divorce the Court of First Instance of the city of Manila did not lack
jurisdiction over the persons of the litigants, for, although Spanish Catholic subjects, they were
residents of this city and had their domicile herein.
The Courts of First Instance of the Philippine Islands have the power and jurisdiction to try
actions for divorce. That of the city of Manila did not lack jurisdiction by reason of the subject
matter of the litigation.

The second assignment of error is directed against the finding of the court that the defendant
had committed adultery with a certain woman in this city from the year 1899 until 1909; the third
was against the finding that the adultery was accompanied by public scandal and injured the
dignity of his wife; and the fourth for having decreed the divorce, suspension of the married life,
and the separation of the properties of the parties.

The evidence relating to the foregoing not being sent up on appeal, we are unable to review it,
so we accept the findings of the trial court.

There is a point of law regarding the claim that the adultery, even though it were proven would
not be a cause for divorce, because no public scandal resulted therefrom nor was there
contempt displayed for the wife. (Appellant's brief, p. 26.) The facts must be accepted by this
tribunal as they were found by the trial court, since the evidence cannot be reviewed; moreover,
the appellee affirms the contrary and maintains that it is a proven fact, public and notorious, an
assertion that the trial court must have found to be proven. (Appellee's brief, p. 5.) In law, it is
not necessary that adultery, to be a cause for divorce, should be accompanied by public
scandal and contempt for the wife. There is no law that requires this. Law 2, title 9, of the Fourth
Partida does not require it.

The fifth and sixth assignments of error are directed against the finding of the trial court that
there exists conjugal property, a finding that the appellant maintains is without foundation, and
that which holds that the property in the hands of the receiver (that sought to be divided) is
conjugal property, a conclusion which the appellant claims to be contrary to the law which
should be applied to the case and according to which, as alleged in the tenth assignment of
error, the whole of the property should be adjudicated to the defendant as being exclusively his.

Facts: The appellant affirms that he is a native of Mallorca in the Balearic Islands and that is
also the condition of his wife, the plaintiff. Law: That although the rule of the Civil Code is that
which legally governs conjugal property, yet at the same time it admits, as an exception, the
laws, usages, and customs of the Foral Law, according to which, as applied in the Balearic
Islands, the law of the family is that of the division of property and that of conjugal property is
not known; so that the property pertains exclusively to the spouse who, by whatever title, has
acquired it. In support of the facts, appellant cites pages 27 to 37 and 39 to 41 in the bill of
exceptions; and of the law, the doctrinal authority of Manresa, Gutierrez, and Alcubilla.

The citation from pages 39 to 41 of the bill of exceptions, the only pertinent one, is but an
affidavit filed by the defendant in which, under oath, he himself testifies as to the Foral Law in
the Balearic Islands. The adverse party says with regard to this: "This affidavit was never
presented in proof, was never received by the trial judge, and cannot seriously be considered as
an effort to establish the law of a foreign jurisdiction. Sections 300, 301 and 302 of the Code of
Civil Procedure, now in force in these islands, indicate the method by which the law of a foreign
country may be proved. We maintain that the affidavit of a person not versed in the law, which
was never submitted as proof, never received by the trial court, and which has never been
subjected to any cross-examination, is not a means of proving a foreign law on which the
defendant relies." (Brief, pp. 6 and 7.)
Furthermore, on the supposition that the defendant could invoke the Foral Law as the law of his
personal status in the matter of the regimen of his marriage, and that to allege this he be
considered as authorized by article 15 of the Civil Code, we have said before, in dealing with his
law of domicile, that paragraph 2 of this article 15 of the Civil Code would be entirely adverse to
his claim, and if it be advanced that there is a similar Foral Law in the Philippines by virtue of
paragraph 1 of the said article 15, it might be said, though there is not at present any need to
say it, that it is not in force. The two findings attacked are in perfect accord with the law. All the
property of the marriage, says article 1407 of the Civil Code, shall be considered as conjugal
property until it is proven that it belongs exclusively to the husband or to the wife. No proof has
been submitted to this effect.

As seventh assignment of error it is alleged that the court below erred in holding in the judgment
that the plaintiff had brought to the marriage a dowry of 30,000 Spanish dollars. But the
defendant himself adds that the court made no order or decree regarding the alleged dowry. On
the other hand, the plaintiff, in her fourth assignment of errors, claimed that the court erred in
not confirming the report of the commissioners which gave to the said plaintiff the sum of 30,000
Spanish dollars. It is unnecessary to say anything further.

The eighth error consists in that the court below ordered the defendant to pay to the plaintiff
P56,010.17 Philippine currency, whereas the plaintiff had made no demand in her complaint
with respect to this sum; that no arrears of payment are owing for alimony, even though
payments had been stipulated in the contract, unless they are claimed by the person who had
furnished the actual support, and that alimony is due only when it is necessary; so that, as the
plaintiff has had no need of it for ten years, nor has she stated who has furnished it, there is no
reason for awaring her the amount of the arrears for all that time; that as she has allowed ten
years to elapse before claiming it, her action prescribed in 1904, that is to say, after five years.

The plaintiff acknowledges that there is no petition or prayer in her complaint as to this cause of
action, but she considers that in equity such an omission can be supplied.

Paragraph 3 of section 89 (90) of the Code of Civil Procedure determines one of the requisites
of the complaint: "A demand for the relief which the plaintiff claims." The section goes on to say:
"If the recovery of money or damages is demanded, the amount demanded must be stated. If
special relief, such as an order for the special restitution of property, etc., the ground of
demanding such relief must be stated and the special relief prayed for. But there may be added
to the statement of the specific relief demanded a general prayer for such further or other relief
as shall be deemed equitable."

In the complaint of the case at bar the provisions of paragraph 2 of the said section 89 [90] are
complied with by setting forth in its paragraphs 4 and 5 the relation of the cause of action, that
is, the contract of the 4th of April, 1899, by which the defendant obligated himself to send to the
plaintiff in Spain a certain amount of money monthly, for her support, and the failure to comply
with this obligation after the month of August, 1899. Paragraph 6, as a consequence of the
promise established in 4 and 5, says as follows: "That the defendant Gabriel Fuster y Fuster
actually owes the plaintiff the sum of 36,100 Spanish pesetas, that is, 7,220 dollars, which,
reduced at the present rate of exchange, amounts to the sum of P12,959.90, Philippine
currency." (B. of E., p. 2.) In the case of default on the part of the defendant "the court shall
proceed to hear the plaintiff and his witnesses and assess the damages or determine the other
relief to which the plaintiff may be entitled, including the costs of the action, and render final
judgment for the plaintiff to recover such sum or to receive such other relief as the pleadings
and the facts warrant." The pleadings, not the prayer of the complaint.

This court has recently decided that the pleadings, not the prayer, exactly, are the essential part
of a complaint.

It is not a question of alimony for the present, nor for the future, which constitutes the first cause
of action, but of certain sums stipulated in a contract. This contract is a law for the contracting
parties, a law which rises superior to those general laws which regulate the nature of the subject
matter of the contract (in the present case an entirely voluntary one) and which govern judicial
action.

An action arising out of a contract of this nature does not prescribe like all personal ones, but,
by the provisions of article 1964 of the Civil Code, after fifteen years. But even though the
provisions of article 1966 were applicable, by which an action to compel the fulfillment of an
agreement to pay alimony prescribes in five years, yet by section 50 of the Code of Civil
Procedure, "when payment has been made upon any demand founded upon contract . . . an
action may be brought . . . after such payment. . . ." And the parties admit that on the 18th of
August, 1908, the plaintiff secured the payment of 6,365.68 pesetas by virtue of the contract of
April 4, 1899. So that from August, 1908, until March, 1909, the date of the complaint, the said
period of five years had not elapsed.

The ninth assignment of error consists in that the court below erred in empowering the receiver
to proceed to the separation of the property and in appointing commissioners to make the
partition and distribution between the spouses, since the principal question in this action hinges
upon the classification of the property; that it was erroneously classified as conjugal property,
whereas all of it pertained to the husband alone and should be adjudicated to him for the reason
that, as it reiterated in the tenth assignment of error, the conjugal partnership was not subject to
the provisions of the law governing conjugal property, because such provision are totally foreign
to the Foral Law of the Balearic Islands.

The action of the trial court, by the terms of section 184 of the Code of Civil Procedure, was in
accordance with law. The only question before this court is the partition of real property. All that
referred to in the second decision appealed from, dated September 9, 1911, is urban real
estate. Its classification as conjugal property is in accordance with law, as is shown in the
foregoing reasoning, and that no consideration of the Foral Law enters into the question has
also been demonstrated.

II. PLAINTIFF'S APPEAL.

As the trial court rendered judgment ordering the defendant to pay to the plaintiff only
P5,010.17, the petitioner here prays that the judgment be reversed and that in its place this
court order the defendant to pay to the plaintiff her claim of P12,959.90, plus the additional sum
which the alimony amounts to at the rate of P107.70 per month, dating from the 1st of August,
1909, until the date of payment, with legal interest upon the said P12,959.90 from the date of
the filing of the complaint until the date of payment, and, furthermore, legal interest upon each of
the monthly payments due after the filing of the complaint, and which will continue to become
due until the close of this litigation.
The trial court made the following findings: First, that the total amount of the alimony owing to
the plaintiff amounted to 34,200 pesetas; second, that of this sum the plaintiff had collected in
Madrid 6,365.68; third, that the remainder, that is, 27,834.32, was equivalent to $5,566.86
Mexican currency; fourth, that the Mexican peso was worth 90 centavos Philippine currency;
fifth, that therefore the sum of $5,566.86 Mexican currency was equivalent to P5,010 Philippine
currency; and finally, as there was no evidence as to the kind of pesetas agreed upon, it was to
be presumed that it was that current at the time and place where the agreement was made,
which was Mexicanpesetas.

In her appeal, the plaintiff contends that these findings are erroneous in that, firstly, the parties
had admitted that the pesetas referred to in the contract of the 4th of April, 1899, were Spanish,
and in view of this admission the court was not empowered to define them as being different
from the kind admitted by the parties; secondly, if he were so empowered, his interpretation
should be governed by the terms of the law.

With regard to the first error, the plaintiff says that the statement is made in her complaint that
the defendant had obligated himself to pay her a "monthly pension for her support of 300
Spanish pesetas, that is, 60 Spanish dollars, which, reduced to Philippine currency, amounts to
P107.70;" that the defendant had admitted this in hi answer to the complaint, and that by his
finding in a sense other than that accepted and not refuted in the answer of the defendant, the
court violated the provisions of section 94 of the Code of Civil Procedure.

The court has not incurred this error, because it does not appear that the defendant in his
answer accepted the fact in the manner alleged in the complaint. The defendant said that he
admitted having made the agreement referred to in paragraph 4 of the complaint, and that
he stood upon its contents. The contents of the document to which he refers is of the following
tenor: "Mr. Fuster binds and obligates himself to pay to his said wife the sum of 300 pesetas,
monthly, payable de su cuenta in the city and capital of Madrid, for her support. . . ." He did not
therefore admit the matter of the Spanish pesetas; that does not appear in the contents of the
document — the only thing he admitted in his answer.

As to the second error, the court did not commit it in applying the rule contained in article 1287
of the Civil Code. "The usages or customs of the country shall be taken into consideration in
interpreting ambiguity in contracts. . . ." If in the contract the word " pesetas," not being specific,
was ambiguous, then it was in harmony with this precept to interpret it as being the peseta
then in use or current when and where the agreement was made, Mexican being then the usual
and current money in the Philippines. Furthermore, the phrase de su cuenta clearly means that
it was not "Spanish pesetas" that the contracting parties had in mind, because if the agreement
had been a specific one to pay 300 Spanish pesetas in Madrid, everyone would of course
understand that the expense of following the fluctuations of change and of the differences in
value between the money current in the country, and the Spanishpesetas, would have to be
defrayed by the obligated party; whereas, if nothing more than pesetas was mentioned, it was
necessary to decide which party should pay for the difference in value so that the
300 pesetas stipulated here should be 300 Spanish pesetas paid in Madrid. Against the reasons
of the court below for his decision this court can offer no legal grounds. The rule of interpretation
cited is the one applicable and it supports the reasoning of the decision appealed from.

The appellant also alleges as error that the court did not adjudicate to her the 30,000 Spanish
dollars which the commissioners proposed in their report. First she characterizes this sum of
30,000 dollars as the dowry of the wife delivered to the husband, then, later, as paraphernal
property brought to the marriage.

According to the last instructions of the court to the commissioners, this amount of 30,000
dollars could not enter into the partition, and with reason. If, as was claimed, it was inherited by
the plaintiff from her uncle, it really constitutes paraphernal property under article 1381.
"Paraphernal property is that which the wife brings to the marriage without being included in the
dowry and that she may acquire after the creation of the same without being added thereto." But
it is a provision of article 1384 that "The wife shall have the management of the paraphernal
property unless she has delivered the same to her husband, before a notary, in order that he
may administer said property. In such case the husband is obliged to create a mortgage for the
value of the personal property he may receive, or to secure said property, in the manner
established for the dowry property." Not even was there offered in evidence the public deed of
delivery, nor the equally public mortgage deed that is required by law. So that, therefore, the
necessary proof of the obligation to return paraphernal property as here demanded does not
exist.lawphil.net

The partition of property decreed in the judgment appealed from of the 9th of September, 1911,
should be and is hereby confirmed.

The two judgments appealed from are hereby affirmed, without special pronouncement of costs
in this instance.

Arellano, C.J., Torres, Carson and Trent, JJ., concur.

MERCADO VS ESPIRITU

Republic of the Philippines


SUPREME COURT
Manila

EN BANC

DECISION

December 1, 1917

G.R. No. L-11872


DOMINGO MERCADO and JOSEFA MERCADO, plaintiffs-appellants,
vs.
JOSE ESPIRITU, administrator of the estate of the deceased Luis Espiritu,
defendant-appellee.

Perfecto Salas Rodriguez for appellants.


Vicente Foz for appellee.

Torres, J.:

This is an appeal by bill of exceptions, filed by the counsel for the plaintiffs from the
judgment of September 22, 1914, in which the judge of the Seventh Judicial District
dismissed the complaint filed by the plaintiffs and ordered them to keep perpetual
silence in regard to the litigated land, and to pay the costs of the suit.

By a complaint dated April 9, 1913, counsel for Domingo and Josefa Mercado
brought suit in the Court of First Instance of Bulacan, against Luis Espiritu, but, as
the latter died soon thereafter, the complaint was amended by being directed against
Jose Espiritu in his capacity of his administrator of the estate of the deceased Luis
Espiritu. The plaintiffs alleged that they and their sisters Concepcion and Paz, all
surnamed Mercado, were the children and sole heirs of Margarita Espiritu, a sister of
the deceased Luis Espiritu; that Margarita Espiritu died in 1897, leaving as her
paraphernal property a tract of land of 48 hectares in area situated in the barrio of
Panducot, municipality of Calumpit, Bulacan, and bounded as described in
paragraph 4 of the amended complaint, which hereditary portion had since then been
held by the plaintiffs and their sisters, through their father Wenceslao Mercado,
husband of Margarita Espiritu; that, about the year 1910, said Luis Espiritu, by
means of cajolery, induced, and fraudulently succeeded in getting the plaintiffs
Domingo and Josefa Mercado to sign a deed of sale of the land left by their mother,
for the sum of P400, which amount was divided among the two plaintiffs and their
sisters Concepcion and Paz, notwithstanding the fact that said land, according to its
assessment, was valued at P3,795; that one-half of the land in question belonged to
Margarita Espiritu, and one-half of this share, that is, one-fourth of said land , to the
plaintiffs, and the other one-fourth, to their two sisters Concepcion and Paz; that the
part of the land belonging to the two plaintiffs could produce 180 cavanes of rice per
annum, at P2.50 per cavan, was equivalent to P450 per annum; and that Luis
Espiritu had received said products from 1901 until the time of his death. Said
counsel therefore asked that judgment be rendered in plaintiffs' favor by holding to
be null and void the sale they made of their respective shares of their land, to Luis
Espiritu, and that the defendant be ordered to deliver and restore to the plaintiffs the
shares of the land that fell to the latter in the partition of the estate of their deceased
mother Margarita Espiritu, together with the products thereof, uncollected since
1901, or their equivalent, to wit, P450 per annum, and to pay the costs of the suit.

In due season the defendant administrator answered the aforementioned complaint,


denying each and all of the allegations therein contained, and in special defense
alleged that the land, the subject-matter of the complaint, had an area of only 21
cavanes of seed rice; that, on May 25, 1894, its owner, the deceased Margarita
Espiritu y Yutoc, the plaintiffs' mother, with the due authorization of her husband
Wenceslao Mercado y Arnedo Cruz sold to Luis Espiritu for the sum of P2,000 a
portion of said land, to wit, an area such as is usually required for fifteen cavanes of
seed; that subsequently, on May 14, 1901, Wenceslao Mercado y Arnedo Cruz, the
plaintiffs' father, in his capacity as administrator of the property of his children sold
under pacto de retro to the same Luis Espiritu at the price of P375 the remainder of
the said land, to wit, an area covered by six cavanes of seed to meet the expenses
of the maintenance of his (Wenceslao's) children, and this amount being still
insufficient the successively borrowed from said Luis Espiritu other sums of money
aggregating a total of P600; but that later, on May 17,1910, the plaintiffs, alleging
themselves to be of legal age, executed, with their sisters Maria del Consejo and
Maria dela Paz, the notarial instrument inserted integrally in the 5th paragraph of the
answer, by which instrument, ratifying said sale under pacto de retro of the land that
had belonged to their mother Margarita Espiritu, effected by their father Wenceslao
Mercado in favor of Luis Espiritu for the sum of P2,600, they sold absolutely and
perpetually to said Luis Espiritu, in consideration of P400, the property that had
belonged to their deceased mother and which they acknowledged having received
from the aforementioned purchaser. In this cross-complaint the defendant alleged
that the complaint filed by the plaintiffs was unfounded and malicious, and that
thereby losses and damages in the sum of P1,000 had been caused to the intestate
estate of the said Luis Espiritu. He therefore asked that judgment be rendered by
ordering the plaintiffs to keep perpetual silence with respect to the land in litigation
and, besides, to pay said intestate estate P1,000 for losses and damages, and that
the costs of the trial be charged against them.

In reply to the cross-complaint, the plaintiffs denied each and all of the facts therein
set forth, and in special defense alleged that at the time of the execution of the deed
of sale inserted in the cross-complaint the plaintiffs were still minors, and that since
they reached their majority the four years fixed by law for the annulment of said
contract had not yet elapsed. They therefore asked that they be absolved from the
defendant's cross-complaint.

After trial and the introduction of evidence by both parties, the court rendered the
judgment aforementioned, to which the plaintiffs excepted and in writing moved for a
reopening of the case and a new trial. This motion was overruled, exception was
taken by the petitioners, and the proper bill of exceptions having been presented, the
same was approved and transmitted to the clerk of this court.

As the plaintiffs assailed the validity of the deed of sale, Exhibit 3, executed by them
on May 17, 1910, on the ground that they were minors when they executed it, the
questions submitted to the decision of this court consist in determining whether it is
true that the plaintiffs were then minors and therefore incapable of selling their
property on the date borne by the instrument Exhibit 3; and in case they then were
such, whether a person who is really and truly a minor and, notwithstanding, attests
that he is of legal age, can, after the execution of the deed and within legal period,
ask for the annulment of the instrument executed by him, because of some defect
that invalidates the contract, in accordance with the law (Civ. Code, arts. 1263 and
1300), so that he may obtain the restitution of the land sold hnJsYw1zg.

The records shows it to have been fully proven that in 1891 Lucas Espiritu obtained
title by composition with the State, to three parcels of land, adjoining each other, in
the sitio of Panducot of the pueblo of Calumpit, Bulacan, containing altogether an
area of 75 hectares, 25 ares, and 59 centares, which facts appear in the title Exhibit
D; that, upon Luis Espiritu's death, his said lands passed by inheritance to his four
children named Victoria, Ines, Margarita, and Luis; and that, in the partition of said
decedent's estate, the parcel of land described in the complaint as containing forty-
seven and odd hectares was allotted to the brother and sister Luis and Margarita, in
equal shares. Margarita Espiritu, married to Wenceslao Mercado y Ardeno Cruz, had
by this husband five children, Maria Consejo, Maria de la Paz, Domingo, Josefa, and
Amalia, all surnamed Mercado y Espiritu, who, at the death of their mother in 1896
inherited, by operation of law, one-half of the land described in the complaint.

The plaintiffs' petition for annulment of the sale and the consequent restitution to
them of two-fourths of the land left by their mother, that is, of one-fourth of all the
land described in the complaint, and which, they stated, amounts to 11 hectares, 86
ares and 37 centares. To this claim the defendant excepted, alleging that the land in
question comprised only an area such as is customarily covered by 21 cavanes of
seed.

It was also duly proven that, by a notarial instrument of May 25, 1894, the plaintiffs'
mother conveyed by actual and absolute sale for the sum of P2,000, to her brother
Luis Espiritu a portion of the land now on litigation, or an area such as is usually
covered by about 15 cavanes of seed; and that, on account of the loss of the original
of said instrument, which was on the possession of the purchaser Luis Espiritu, and
furthermore because, during the revolution, the protocols or registers of public
documents of the Province of Bulacan were burned, Wenceslao Mercado y Arnedo
Cruz, the widower of the vendor and father of the plaintiffs, executed, at the instance
of the interested party Luis Espiritu, the notarial instrument Exhibit 1, of the date of
May 20, 1901, in his own name and those of his minor children Maria Consejo, Maria
de la Paz, Domingo, Josefa, and Amalia, and therein set forth that it was true that the
sale of said portion of land had been made by his aforementioned wife, then
deceased, to Luis Espiritu in 1894.

However, even prior to said date, to wit, on May 14th of the same year, 1901, the
widower Wenceslao Mercado, according to the private document Exhibit 2, pledged
or mortgaged to the same man, Luis Espiritu, for P375, a part, or an area covered by
six cavanes of seed, of the land that had belonged to this vendor's deceased wife, to
the said Luis Espiritu and which now forms a part of the land in question — a
transaction which Mercado was obliged to make in order to obtain funds with which
"to cover his children's needs." Wenceslao Mercado, the plaintiffs' father, having
died, about the year 1904, the plaintiffs Domingo and Josefa Mercado, together with
their sisters Consejo and Paz, declaring themselves to be of legal age and in
possession of the required legal status to contract, executed and subscribed before a
notary the document Exhibit 3, on May 17, 1910, in which referring to the previous
sale of the land, effected by their deceased mother for the sum of P2,600 and with
her husband's permission and authorization, they sold absolutely and in perpetuity to
Luis Espiritu, for the sum of P400 "as an increase" of the previous purchase price,
the land described in said instrument and situated in Panducot, pueblo of Calumpit,
Bulacan, of an area equal to that usually sown with 21 cavanes of seed bounded on
the north by the lands of Flaviano Abreu and the heirs of Pedro Espiritu, on the east
by those of Victoria Espiritu and Ines Espiritu, on the south by those of Luis Espiritu,
and on the west by those of Hermogenes Tan-Toco and by the Sapang-Maitu
stream.

In this status of the case the plaintiffs seek the annulment of the deed Exhibit 3, on
the ground that on the date of its execution they were minors without legal capacity
to contract, and for the further reason that the deceased purchaser Luis Espiritu
availed himself of deceit and fraud in obtaining their consent for the execution of said
deed.

As it was proven by the testimony of the clerk of the parochial church of Apalit
(plaintiffs were born in Apalit) that the baptismal register books of that parish
pertaining to the years 1890-1891, were lost or burned, the witness Maria Consejo
Mercado recognized and identified the book Exhibit A, which she testified had been
kept and taken care of by her deceased father Wenceslao Mercado, pages 396 and
397 of which bear the attestation that the plaintiff Domingo Mercado was born on
August 4, 1890, and Josefa Mercado, on July 14, 1891. Furthermore, this witness
corroborated the averment of the plaintiffs' minority, by the personal registration
certificate of said Domingo Mercado, of the year 1914, Exhibit C, by which it appears
that in 1910 he was only 23 years old, whereby it would also be appear that Josefa
Mercado was 22 years of age in 1910, and therefore, on May 17,1910, when the
instrument of purchase and sale, Exhibit 3, was executed, the plaintiffs must have
been, respectively, 19 and 18 years of age.

The witness Maria Consejo Mercado also testified that after her father's death her
brother and sisters removed to Manila to live there, although her brother Domingo
used to reside with his uncle Luis Espiritu, who took charge of the administration of
the property left by his predecessors in interest; that it was her uncle Luis who got for
her brother Domingo the other cedula, Exhibit B, pertaining to the year 1910, where
in it appears that the latter was then already 23 years of age; that she did not know
why her uncle did so; that she and her brother and sisters merely signed the deed of
May 17, 1910; and that her father Wenceslao Mercado, prior to his death had
pledged the land to her uncle Luis Espiritu.
The witness Ines Espiritu testified that after the death of the plaintiffs' father, it was
Luis Espiritu who directed the cultivation of the land in litigation. This testimony was
corroborated by her sister Victoria Espiritu, who added that her nephew, the plaintiff
Domingo, had lived for some time, she did not know just how long, under the control
of Luis Espiritu.

Roque Galang, married to a sister of Luis Espiritu, stated that the land that fell to his
wife and to his sister-in-law Victoria, and which had an area of about 8 hectares less
than that of the land allotted to the aforementioned Luis and Margarita produced for
his wife and his sister-in-law Victoria a net and minimum yield of 507 cavanes in
1907, in spite of its being high land and of inferior quality, as compared with the land
in dispute, and that its yield was still larger in 1914, when the said two sisters' share
was 764 cavanes.

Patricio Tanjucto, the notary before whom the deed Exhibit 3 was ratified, was a
witness for the defendant. He testified that this deed was drawn up by him at the
request of the plaintiff Josefa Mercado; that the grantors of the instrument assured
him that they were all of legal age; that said document was signed by the plaintiffs
and the other contracting parties, after it had been read to them and had been
translated into the Pampangan dialect for those of them who did not understand
Spanish. On cross-examination, witness added that ever since he was 18 years of
age and began to court, he had known the plaintiff Josefa Mercado, who was then a
young maiden, although she had not yet commenced to attend social gatherings,
and that all this took place about the year 1898, for witness said that he was then [at
the time of his testimony, 1914,] 34 years of age.

Antonio Espiritu, 60 years of age, who knew Lucas Espiritu and the properties owned
by the latter, testified that Espiritu's land contained an area of 84 cavanes, and after
its owner's death, was under witness' administration during to harvest two harvest
seasons; that the products yielded by a portion of this land, to wit, an area such as is
sown by about 15 cavanes of seed, had been, since 1894, utilized by Luis Espiritu,
by reason of his having acquired the land; and that, after Margarita Espiritu's death,
her husband Wenceslao Mercado took possession of another portion of the land,
containing an area of six cavanes of seed and which had been left by this deceased,
and that he held same until 1901, when he conveyed it to Luis Espiritu.

The defendant-administrator, Jose Espiritu, son of the deceased Luis Espiritu,


testified that the plaintiff Domingo Mercado used to live off and on in the house of his
deceased father, about the year 1909 or 1910, and used to go back and forth
between his father's house and those of his other relatives. He denied that his father
had at any time administered the property belonging to the Mercado brother and
sisters.

In rebuttal, Antonio Mercado, a cousin of Wenceslao, father of the plaintiffs, testified


that he mediate in several transactions in connection with a piece of land belonging
to Margarita Espiritu. When shown the deed of purchase and sale Exhibit 1, he
stated that he was not acquainted with its contents. This same witness also testified
that he mediated in a transaction had between Wenceslao Mercado and Luis Espiritu
(he did not remember the year), in which the former sold to the latter a parcel of land
situated in Panducot. He stated that as he was a witness of the deed of sale he could
identify this instrument were it exhibited to him; but he did not do so, for no
instrument whatever was presented to him for identification. The transaction
mentioned must have concerned either the ratification of the sale of the land of 15
cavanes, in 1901, attested in Exhibit 1, or the mortgage or pledge of the other parcel
of 6 cavanes, given on May 14, 1901, by Wenceslao Mercado to Luis Espiritu, as
may be seen by the private document Exhibit 2. In rebuttal, the plaintiff Josefa
Mercado denied having gone to the house of the notary Tanjutco for the purpose of
requesting him to draw up any document whatever. She stated that she saw the
document Exhibit 3 for the first time in the house of her uncle Luis Espiritu on the day
she signed it, on which occasion and while said document was being signed said
notary was not present, nor were the witnesses thereto whose names appear
therein; and that she went to her said uncle's house, because he had sent for her, as
well as her brother and sisters, sending a carromata to fetch them. Victoria Espiritu
denied ever having been in the house of her brother. Luis Espiritu in company with
the plaintiffs, for the purpose of giving her consent to the execution of any deed in
behalf of her brother.

The evidence adduced at the trial does not show, even circumstantially, that the
purchaser Luis Espiritu employed fraud, deceit, violence, or intimidation, in order to
effect the sale mentioned in the document Exhibit 3, executed on May 17, 1910. In
this document the vendors, the brother and the sisters Domingo, Maria del Consejo,
Paz and, Josefa surnamed Mercado y Espiritu, attested the certainty of the previous
sale which their mother, during her lifetime, had made in behalf of said purchaser
Luis Espiritu, her brother with the consent of her husband Wenceslao Mercado,
father of the vendors of the portion of land situated in the barrio of Panducot, pueblo
of Calumpit, Bulacan; and in consideration of the fact that the said vendor Luis
Espiritu paid them, as an increase, the sum of P400, by virtue of the contract made
with him, they declare having sold to him absolutely and in perpetuity said parcel of
the land, waive and thenceforth any and all rights they may have, inasmuch as said
sum constitutes the just price of the property JiQuqTKkti.

So that said document Exhibit 3 is virtually an acknowledgment of the contract of


sale of the parcel or portion of land that would contain 15 cavanes of seed rice made
by the vendors' mother in favor of the purchaser Luis Espiritu, their uncle, and
likewise an acknowledgment of the contract of pledge or mortgage of the remainder
of said land, an area of six cavanes, made with the same purchaser, at an increase
of P400 over the price of P2,600, making an aggregate sum of P3,000, decomposed
as follows: P2,000, collected during her lifetime, by the vendors' father; and the said
increase of P400, collected by the plaintiffs pEGNtPU.

In the aforementioned sale, according to the deed of May 25, 1894, Margarita
Espiritu conveyed to her brother Luis the parcel of 15 cavanes of seed, Exhibit 1, and
after her death the plaintiffs' widowed father mortgaged or pledged the remaining
parcel or portion of 6 cavanes of seed to her brother-in-law, Luis Espiritu, in May,
1901 (Exhibit 2). So it is that the notarial instrument Exhibit 3, which was assailed by
the plaintiffs, recognized the validity of the previous contracts, and the totality of the
land, consisting of an area containing 21 cavanes of seed rice, was sold absolutely
and in perpetuity, the vendors receiving in exchange P400 more; and there is no
conclusive proof in the record that this last document was false and simulated on
account of the employment of any violence, intimidation, fraud, or deceit, in the
procuring of the consent of the vendors who executed it.

Considering the relation that exists between the document Exhibit 3 and those of
previous dates, Exhibits 1 and 2, and taking into the account the relationship
between the contracting parties, and also the general custom that prevails in many
provinces of these Islands for the vendor or debtor to obtain an increase in the price
of the sale or of the pledge, or an increase in the amount loaned, without proof to the
contrary, it would be improper and illegal to hold, in view of the facts hereinabove set
forth, that the purchaser Luis Espiritu, now deceased, had any need to forge or
simulate the document Exhibit 3 inasmuch as, since May, 1894, he has held in the
capacity of owner by virtue of a prior acquisition, the parcel of land of 15 cavanes of
seed, and likewise, since May, 1901, according to the contract of mortgage or
pledge, the parcel of 6 cavanes, or the remainder of the total area of 21 cavanes
e6hatlmMQB.

So that Luis Espiritu was, during his lifetime, and now, after his death, his testate or
intestate estate is in lawful possession of the parcel of land situated in Panducot that
contains 21 cavanes of seed, by virtue of the title of conveyance of ownership of the
land measuring 15 cavanes, and, in consequence of the contract of pledge or
mortgage in security for the sum of P600, is likewise in lawful possession of the
remainder of the land, or an area containing 6 cavanes of seed.

The plaintiffs have absolutely no right whatever to recover said first parcel of land, as
its ownership was conveyed to the purchaser by means of a singular title of purchase
and sale; and as to the other portion of 6 cavanes of seed, they could have
redeemed it before May 17, 1910, upon the payment or the return of the sum which
their deceased father Wenceslao Mercado had, during his lifetime, received as a
loan under security of the pledged property; but, after the execution of the document
Exhibit 3, the creditor Luis Espiritu definitely acquired the ownership of said parcel of
6 cavanes. It is therefore a rash venture to attempt to recover this latter parcel by
means of the contract of final and absolute sale, set forth in the deed Exhibit 3.
Moreover, the notarial document Exhibit 1, are regards the statements made therein,
is of the nature of a public document and is evidence of the fact which gave rise to its
execution and of the date of the latter, even against a third person and his
predecessors in interest such as are the plaintiffs. (Civ. Code, art. 1218.)

The plaintiffs' father, Wenceslao Mercado, recognizing it to be perfectly true that his
wife Margarita Espiritu sold said parcel of land which she inherited from her father, of
an area of about "15 cavanes of seed," to her brother Luis Espiritu, by means of an
instrument executed by her on May 25,1894 — an instrument that disappeared or
was burned — and likewise recognizing that the protocols and register books
belonging to the Province of Bulacan were destroyed as a result of the past
revolution, at the request of his brother-in-law Luis Espiritu he had no objection to
give the testimony recorded in said notarial instrument, as it was the truth regarding
what had occurred, and in so doing he acted as the plaintiffs' legitimate father in the
exercise of his parental authority, inasmuch as he had personal knowledge of said
sale, he himself being the husband who authorized said conveyance,
notwithstanding that his testimony affected his children's interest and prejudiced his
own, as the owner of any fruits that might be produced by said real property.

The signature and handwriting of the document Exhibit 2 were identified as authentic
by one of the plaintiffs, Consejo Mercado, and as the record shows no evidence
whatever that this document is false, and it does not appear to have been assailed
as such, and as it was signed by the plaintiffs' father, there is no legal ground or well-
founded reason why it should be rejected. It was therefore properly admitted as
evidence of the certainty of the facts therein set forth.

The principal defect attributed by the plaintiffs to the document Exhibit 3 consists in
that, on the date of May 17, 1910, when it was executed that they signed it, they
were minors, that is, they had not yet attained the age of 21 years fixed by Act No.
1891, though no evidence appears in the record that the plaintiffs Josefa and
Domingo Mercado were in fact minors, for no certified copies were presented of their
baptismal certificates, nor did the plaintiffs adduce any supplemental evidence
whatever to prove that Domingo was actually 19 and Josefa 18 years of age when
they signed the document Exhibit 3, on May 17, 1910, inasmuch as the copybook,
Exhibit A, notwithstanding the testimony of the plaintiff Consejo Mercado, does not
constitute sufficient proof of the dates of births of the said Domingo and Josefa.

However, even in the doubt whether they certainly were of legal age on the date
referred to, it cannot be gainsaid that in the document Exhibit 3 they stated that they
were of legal age at the time they executed and signed it, and on that account the
sale mentioned in said notarial deed Exhibit 3 is perfectly valid — a sale that is
considered as limited solely to the parcel of land of 6 cavanes of seed, pledged by
the deceased father of the plaintiffs in security for P600 received by him as a loan
from his brother-in-law Luis Espiritu, for the reason that the parcel of 15 cavanes had
been lawfully sold by its original owner, the plaintiffs' mother.

The courts, in their interpretation of the law, have laid down the rule that the sale of
real estate, made by minors who pretend to be of legal age, when in fact they are
not, is valid, and they will not be permitted to excuse themselves from the fulfillment
of the obligations contracted by them, or to have them annulled in pursuance of the
provisions of Law 6, title 19, of the 6th Partida; and the judgment that holds such a
sale to be valid and absolves the purchaser from the complaint filed against him
does not violate the laws relative to the sale of minors' property, nor the juridical
rules established in consonance therewith. (Decisions of the supreme court of Spain,
of April 27, 1860, July 11, 1868, and March 1, 1875.)

With respect to the true age of the plaintiffs, no proof was adduced of the fact that it
was Luis Espiritu who took out Domingo Mercado's personal registration certificate
on April 13, 1910, causing the age of 23 years to be entered therein in order to
corroborate the date of the notarial instrument of May 17th of the same year; and the
supposition that he did, would also allow it to be supposed, in order to show the
propriety of the claim, that the cedula Exhibit C was taken out on February 14, 1914,
where in it is recorded that Domingo Mercado was on that date 23 years of age, for
both these facts are not proved; neither was any proof adduced against the
statement made by the plaintiffs Domingo and Josefa in the notarial instrument
Exhibit 3, that, on the date when they executed it, they were already of legal age,
and, besides the annotation contained in the copybook Exhibit A, no supplemental
proof of their true ages was introduced.

Aside from the foregoing, from a careful examination of the record in this case, it
cannot be concluded that the plaintiffs, who claim to have minors when they
executed the notarial instrument Exhibit 3, have suffered positive and actual losses
and damages in their rights and interests as a result of the execution of said
document, inasmuch as the sale effected by the plaintiffs' mother, Margarita Espiritu,
in May, 1894, of the greater part of the land of 21 cavanes of seed, did not occasion
any damage or prejudice to the plaintiffs, inasmuch as their father stated in the
document Exhibit 2 that he was obliged to mortgage or pledge said remaining portion
of the land in order to secure the loan of the P375 furnished by Luis Espiritu and
which was subsequently increased to P600 so as to provide for certain engagements
or perhaps to meet the needs of his children, the plaintiff; and therefore, to judge
from the statements made by their father himself, they received through him, in
exchange for the land of 6 cavanes of seed, which passed into the possession of the
creditor Luis Espiritu, the benefit which must have accrued to them from the sums of
money received as loans; and, finally, on the execution of the impugned document
Exhibit 3, the plaintiffs received and divided between themselves the sum of P400,
which sum, added to that P2,000 received by Margarita Espiritu, and to that of the
P600 collected by Wenceslao Mercado, widower of the latter and father of the
plaintiffs, makes all together the sum of P3,000, the amount paid by the purchaser as
the price of all the land containing 21 cavanes of seed, and is the just price of the
property, was not impugned, and, consequently, should be considered as equivalent
to, and compensatory for, the true value of said land.

For the foregoing reasons, whereby the errors assigned to the judgment appealed
from have been refuted, and deeming said judgment to be in accordance with law
and the evidence of record, we should, and do hereby, affirm the same, with costs
against the appellants. So ordered.

Arellano, C. J., Johnson, Street, and Malcolm, JJ., concur.

Separate Opinions

CARSON, J., concurring:

I concur.

But in order to avoid misunderstanding, I think it well to indicate that the general
statement, in the prevailing opinion to the effect that the making of false
representations as to his age by an infant executing a contract will preclude him from
disaffirming the contract or setting up the defense of infancy, must be understood as
limited to cases wherein, on account of the minor's representations as to his majority,
and because of his near approach thereto, the other party had good reason to
believe, and did in fact believe the minor capable of contracting.

The doctrine set forth in the Partidas, relied upon by the supreme court of Spain in
the cases cited in the prevailing opinion, is substantially similar to the doctrine of
estoppel as applied in like instances by many of the courts in the United States.

For the purposes of convenient comparison, I here insert some citations of authority,
Spanish and American, recognizing the limitations upon the general doctrine to
which I am inviting attention at this time; and in this connection it is worthy of note
that the courts of the United States look with rather less favor than the supreme court
of Spain upon the application of the doctrine, doubtless because the cases wherein it
may properly be applied, are much less likely to occur in a jurisdiction where majority
is reached at the age of 21 than a jurisdiction wherein majority is not ordinarily
attained until the infant reaches the age of 25.
Ley 6, tit. 19, Partida 6. is, in part, as follows:

If he who is minor (1) deceitfully says or sets forth in an instrument that he is over
twenty-five years of age, and this assertion is believed by another person who takes
him to be of about that age, (2) in an action at law he should be deemed to be of the
age he asserted, and should no (3) afterwards be released from liability on the plea
that he was not of said age when he assumed the obligation. The reason for this is
that the law helps the deceived and not the deceivers.

In the glossary to these provisions of the Partidas by Gregorio Lopez, I find the
following:

(1) De tal tiempo. Nota bene hoc verbum, nam si appareret ex aspectu eum esse
minorem, tunc adversarius non potest dicere se deceptum; imo tam ipse, quam
minor videntur esse in dolo, quo casu competit minori restitutio, quia facta doli
compensatione, perinde ast ac si nullus fuiset in dolo, et ideo datur restitutio; et quia
scienti dolus non infertur, l. 1. D. de act. empt. secundum Cyn. Alberic et Salic. in l.
3. C. si minor se major. dixer. adde Albericum tenentem, quabndo per aspectum a
liter constaret, in authent. sacramenta puberum, col. 3. C. si advers vendit.

(2) Engoñosamente. Adde 1. 2. et 3. C. si minor se major. dixer. Et adverte nam per


istam legem Partitarum, que non distinguit, an adultus, vel pupillus talem
assertionem faciat, videtur comprobari dictum Guillielm. de Cun. de quo per Paul. de
Castr. in 1. qui jurasse. in princ. D. de jurejur. quod si pupillus proximus pubertari
juret, cum contrahit, se esse puberem, et postea etiam juret, quod non veniet contra
contractum quod habebit locum dispositio authenticae sacramenta puberum, sicut si
esset pubes: et cum isto dicto transit ibi Paul. de Cast. multum commendans, dicens,
se alibi non legisse; si tamen teneamus illam opinionem, quod etiam pupillus doli
capax obligatur ex juramento, non esset ita miranda dicat, decissio; vide per
Alexand. in dict. 1. qui jurasse, in princ. Item lex ista Partitarum expresse sentit de
adulto, non de pupillo, cum superius dixit, que paresciere de tal tiempo: Doctores
etiam intelligunt de adulto 11. dict. tit. C. si minor. se major. dixer. et patet ex 11.
illius tituli. Quid autem dicemus in dubio, cum non constat de dolo minoris? Azon. in
summa illius tit. in fin. Cynus tamen, et alli, tenent oppositum, quia dolus non
praesumitur, nisi probetur, 1. quotiens, s., qui dolo, D. de probat. Et hoc etiam vult
ista lex Partitarum, cum dicit, si lo faze engoñosamente: et ita tenent Alberic. et
Salicet. in dict. 1. 3. ubi etiam Bart. in fin. Si autem minor sui facilitate asserat se
mojorem, et ita juret, tunc distingue, ut habetur dict. 1. 3 quia aut juravit verbo tenus,
et tunc non restituitur, nisi per instrumentum seu scripturam probet se minorem; et si
juravit corporaliter, nullo modo restituitur, ut ibi; et per quae instrumenta probentur,
cum verbo tenus juravit, vide per Specul. tit. de restit, in integr. s. quis autem, col. 4.
vers. sed cujusmodi erit scriptura, ubi etiam vide per Speculatorem aliquas notabiles
quaestiones in ista materia, in col. 5. videlicet, an praejudicet sibi minor ex tali
juramento in aliis contractibus, et tenet, quod non; et tenet glossa finalis in 1. de
aetate, D. de minor. in fin. gloss. vide ibi per Speculat. ubi etiam de aliis in ista
materia.

In the decision of the supreme court of Spain dated the 27th of April, 1860, I find an
excellent illustration of the conditions under which that court applied the doctrine, as
appears from the following resolution therein set forth.

Sales of real estate made by minors are valid when the latter pretend to be twenty-
five years of age and, due to the circumstances that they are nearly of that age, are
married, or have administration of their property, or on account of other special
circumstances affecting them, the other parties to the contract believe them to be of
legal age.

With these citations compare the general doctrine in the United States as set forth in
22 Cyc. (p. 610), supported by numerous citations of authority.

Estoppel to disaffirm — (I) In General. — The doctrine of estoppel not being as a


general rule applicable to infants, the court will not readily hold that his acts during
infancy have created an estoppel against him to disaffirm his contracts. Certainly the
infant cannot be estopped by the acts or admissions of other persons.

(II) False representations as to age. — According to some authorities the fact that an
infant at the time of entering into a contract falsely represented to the person with
whom he dealt that he had attained the age of majority does not give any validity to
the contract or estop the infant from disaffirming the same or setting up the defense
of infancy against the enforcement of any rights thereunder; but there is also
authority for the view that such false representations will create an estoppel against
the infant, and under the statutes of some states no contract can be disaffirmed
where, on account of the minor's representations as to his majority, the other party
had good reason to believe the minor capable of contracting. Where the infant has
made no representations whatever as to his age, the mere fact that the person with
whom he dealt believed him to be of age, even though his belief was warranted by
the infant's appearance and the surrounding circumstances, and the infant knew of
such belief, will not render the contract valid or estop the infant to disaffirm. .

BAMBALAN VS MARAMBA

Republic of the Philippines


SUPREME COURT
Manila

EN BANC

G.R. No. L-27710 January 30, 1928


ISIDRO BAMBALAN Y PRADO, plaintiff-appellant,
vs.
GERMAN MARAMBA and GENOVEVA MUERONG, defendants-appellants.

Pedro C. Quinto for plaintiff-appellant.


Turner, Rheberg and Sanchez for defendants-appellants.

ROMUALDEZ, J.:

The defendants admit in their amended answer those paragraphs of the complaint wherein it is
alleged that Isidro Bambalan y Colcotura was the owner, with Torrens title, of the land here in
question and that the plaintiff is the sole and universal heir of the said deceased Isidro
Bambalan y Colcotura, as regards the said land. This being so, the fundamental question to be
resolved in this case is whether or not the plaintiff sold the land in question to the defendants.

The defendants affirm they did and as proof of such transfer present document Exhibit 1, dated
July 17, 1922. The plaintiff asserts that while it is true that he signed said document, yet he did
so by intimidation made upon his mother Paula Prado by the defendant Genoveva Muerong,
who threatened the former with imprisonment. While the evidence on this particular point does
not decisively support the plaintiff's allegation, this document, however, is vitiated to the extent
of being void as regards the said plaintiff, for the reason that the latter, at the time he signed it,
was a minor, which is clearly shown by the record and it does not appear that it was his real
intention to sell the land in question.

What is deduced from the record is, that his mother Paula Prado and the latter's second
husband Vicente Lagera, having received a certain sum of money by way of a loan from
Genoveva Muerong in 1915 which, according to Exhibit 3, was P200 and according to the
testimony of Paula Prado, was P150, and Genoveva Muerong having learned later that the land
within which was included that described in said Exhibit 3, had a Torrens title issued in favor of
the plaintiff's father, of which the latter is the only heir and caused the plaintiff to sign a
conveyance of the land.

At any rate, even supposing that the document in question, Exhibit 1, embodies all of the
requisites prescribed by law for its efficacy, yet it does not, according to the provisions of section
50 of Act No. 496, bind the land and would only be a valid contract between the parties and as
evidence of authority to the register of deeds to make the proper registration, inasmuch as it is
the registration that gives validity to the transfer. Therefore, the defendants, by virtue of the
document Exhibit 1 alone, did not acquire any right to the property sold as much less, if it is
taken into consideration, the vendor Isidro Bambalan y Prado, the herein plaintiff, was a minor.

As regards this minority, the doctrine laid down in the case of Mercado and Mercado vs.
Espiritu (37 Phil., 215), wherein the minor was held to be estopped from contesting the contract
executed by him pretending to be age, is not applicable herein. In the case now before us the
plaintiff did not pretend to be of age; his minority was well known to the purchaser, the
defendant, who was the one who purchased the plaintiff's first cedula used in the
acknowledgment of the document.

In regard to the amount of money that the defendants allege to have given the plaintiff and her
son in 1992 as the price of the land, the preponderance of evidence shows that no amount was
given by the defendants to the alleged vendors in said year, but that the sum of P663.40, which
appears in the document Exhibit 1, is arrived at, approximately, by taking the P150 received by
Paula Prado and her husband in 1915 and adding thereto interest at the rate of 50 per cent
annum, then agreed upon, or P75 a year for seven years up to July 31, 1922, the sate of Exhibit
1.

The damages claimed by the plaintiff have not been sufficiently proven, because the witness
Paula Prado was the only one who testified thereto, whose testimony was contradicted by that
of the defendant Genoveva Muerong who, moreover, asserts that she possesses about half of
the land in question. There are, therefore, not sufficient data in the record to award the damages
claimed by the plaintiff.

In view of the foregoing, the dispositive part of the decision appealed from is hereby affirmed,
without any express findings as to the costs in this instance. So ordered.

Johnson, Street, Malcolm, Ostrand, Johns and Villa-Real, JJ., concur.

SUAN VS ALCANTARA

Republic of the Philippines


SUPREME COURT
Manila

EN BANC

G.R. No. L-1720 March 4, 1950

SIA SUAN and GAW CHIAO, Petitioners,


vs.
RAMON ALCANTARA, respondent.

Antonio Barredo for Petitioners.


Zosimo D. Tanalega for Respondents.

PARAS, J.:

On August 3, 1931, a deed of sale WAS Executed by Rufino Alcantara and Sons historical
Damaso Ramon Alcantara Alcantara and Sia Suan Conveying to five parcels of land. Ramon
Alcantara Was Then 17 years, 10 months and 22 days old. On August 27, 1931, Gaw Chiao
(husband of Sia Suan) Received a letter from Francisco Alfonso, attorney of Ramon Alcantara,
Gaw Chiao Informing Ramon Alcantara That Was a minor and disavowing the contract
accordingly. After Being Contacted by Gaw Chiao, pero, Ramon Alcantara Executed an affidavit
in the office of Jose Gomez, attorney of Gaw Chiao, Wherein Ramon Alcantara ratified the deed
of sale. On occasion Said Ramon Alcantara Gaw Chiao Received from the sum of P500. In the
Meantime, Sia Suan one of the lots sold to Nicolas Antonio Azores Azores Whom inherited from
the Same.
On August 8, 1940, WAS institute an action by Ramon Alcantara in the Court of First Instance of
Laguna for the Annulment of the deed of sale as historical Regards undivided share in the two
parcels of land cover certificates of title by Nos. 751 and 752 of Laguna. Said action Sia Suan
WAS Against Gaw Chiao and her husband, Antonio, Azores, Damaso Rufino Alcantara and
Alcantara (the Latter Being two, respectivamente, the brother and father of Ramon Alcantara
appeal to the Court of Appeal reversed the decision Which of the trial court , on the ground That
the deed of sale is Not Against binding Ramon Alcantara in view of historical minority STI on the
date of execution, and accordingly Sia Suan Sentenced to Ramon Alcantara to pay the sum of
P1, 750, with legal Interest from December 17 , 1931, in lieu of historical share in the lot sold to
Antonio Azores (Who Was absolved from the Complaint), and to reconvey to Ramon Alcantara
an undivided one-Fourth Interest in the lot originally Covered by certificate of title NO. 752 of
Laguna plus the cost of the suit. From this Judgement and Gaw Chiao Sia Suan Have come to
us on appeal by certiorari.

It is undeniable That the deed of sale signed by the appellee, Ramon Alcantara, On August 3,
1931, Eichmann That He, Like His co-Signers (father and brother), Was Then of legal age. It is
Not pretend and There is nothing to Indicate That Did not believe the appellants and Rely On
Such recital of fact. This conclusion is Decisive and very Obvious in the decision of the Court of
Appeals It is true That in the resolution on the for reconsideration, the Court of Appeals remark
that "and the Fact That When Informed of Appellant's minority, the appelle too no steps for
nine years to Protect Their Interest Requiring Beyond the Appellant to execute a ratification of
the leaves while still a minor, Strongly Indicates That the appelle Knew Of His minority When
the deed of sale WAS execute. " But the insinuation is sufficiently negative feeble by the
Following positive pronouncements of the Court of Appeals as well as in Said resolution in the
decision.

As to the Complaint That the defendant is guilty of laches, suffice it to say That the appelle
Were Informed of historical minority Within one (1) month after the transaction completed
WAS. (Resolution.)

Finally, the appelle Were Equally Negligent in Not Taking Any Action to Protect Their Interest
Form and after August 27, 1931, When They Were Notified in writing of Appellant's
minority.(Resolution.)

. . . The Fact Remains That Were Advised appelle Within the month the Appellant That Was a
minor, Through the letter of Attorney Alfonso (Exhibit 1) Informing appelle Of His client's desire
to disaffirm the contract. . . (Decision.)

Been HAVING apprise the purchaser of Incapacity of historical vendor WAS Shortly after the
contract made, the delay in Bringing the action of Annulment Will Not unless it serve to bar the
Period fixed by the statute of Limitations expired Before the filing of the
Complaint. . . . (Decision.)
In support of the contend That the deed of sale is binding on the appellee, counsel for the
appellants Invokes the decision in Mercado and Mercado vs.. Spirit (37 Phil., 215), Wherein this
court held:

The courts, in Their interpretation of the law, Have laid down the rule That the out of real estate,
made by minors Who Pretend To Be of legal age, it FACT When They Are not, is valid, and
They Will Not Be permitted to Themselves excused from the fulfillment of the Obligations
Contracted by Them, To Have Them or annulled in pursuance of the Provisions of Law 6 title
19, of the 6th Item, and the Judgement That Holds Such a valid and goes to absolve the
purchaser from the Complaint Filed Against him Does Not Violate the Laws relative to the out of
minors' property, the normal rules Juridical Established in consonance therewith. (Decisions of
the Supreme Court of Spain, of April 27, 1840 July 11, 1868, and March 1, 1875.)

The Court of Appeals has Refuse to Apply this doctrine on the ground That the appellants
Actually Did not Pay Any Amount in cash to the appellee and THEREFORE Did not Suffer Any
Detriment by reason of the deed of sale, it stipulate Being That the consideration THEREFORE
WAS a pre-existing indebtedness of appellee's father, Rufino Alcantara. We are of the opinion
That the Court of Appeals erred. In the first place, in the case cited scientist, the consideration
for sale consist in Greater part of pre-existing obligation. In the second place, under the
doctrine, to bind a minor Who Represents Himself to Be of legal age, not is it NECESSARY for
historical Actually vendee to part with cash, as long as the contract is Supported by a valid
consideration. Since appellee's conveyance to the appellants WAS admittedly for and in Virtue
of a pre-existing indebtedness (unquestionably a valid consideration), it produces STI Should
full force and effect in the Absence of Any Other vice That May Legally invalidate the Same. It is
not here CLAIMED That the deed of sale is null and void on ground and Stock Other Than the
appellee's minority. Appellee's contract has Become Fully Executed efficacious as a contract by
parties with full legal capacity.

The Circumstances That, about one month after the date of the conveyance, the appellee report
The appellants of historical minority, is of no moment, Because appellee's previous
misrepresentation Already Had him estopped from disavowing the contract. Said belated
information Merely leads to the inference That the appellants in fact-Did not Know That the
appellee Was a minor on the date of the contract, and Somewhat Emphasizer appellee's Had
Faith, When it is borne in mind That no sooner Had I Given Said information Than have ratified
historical deed of sale upon receiving from the appellants the sum of P500.

Counsel for the appellants the appelle Argues That Could not Have Been misled as to the real
age of the appellee Because They Were free to make the investigation NECESSARY. The
suggestion, while Perhaps practicable, is conspicuously unbusinesslike and Beside the point,
Because the Findings of the Court of Appeals Do not show the appellants Knew That Could
Suspected or appellee's minority.

The Court of Appeals Seems To Be of the opinion That the letter written by the appellee
Informing the appellants of an Effective establishi historical minority disaffirmance of the leaves,
And That although the choice to disaffirm Itself Will Not Avoid the contract by the Courts Until
adjudge the Agreement to Be invalid, Said notice shielded from the appellee Consequent laches
and estoppel. This position is untenable Since the effect of estoppel in Proper marry is
unaffected by the promptness with a notice to disaffirm Which is made.

The appeal decision of the Court of Appeals is hereby reversed and the appellants absolved
from the Complaint, with Costs Against the appellee, Ramon Alcantara. So ordered.

Ozaeta, Tuason, Montemayor and Torres, JJ., Contests.

Separate Opinions

PADILLA, J., concurring:

I Concur in the result upon the Grounds Not Stated in the Majority opinion But for the Following
Reasons: The deed of sale Executed by Ramon Alcantara on 3 August 1931 Sia Suan
Conveying to five parcels of land is null and void insofar as the Interest, share , or participation
of Ramon Alcantara in two parcels of land is concerned, Because on the date of sale I WAS 17
years, 10 months and 22 days old only. Consent Being one of the essential requisites for the
execution of a valid contract, a minor s, such as Ramon Alcantara WAS, Could not Give His
Consent thereof. The only historical misrepresentation as to age, if any, Was the statement
Appearing in the instrument That He Was of age. On 27 August 1931, or 24 days after the deed
WAS execute, Gaw Chiao, the husband of the vendee Sia Suan, WAS Advised by
Atty.Francisco Alfonso of The Fact That His Client Was a minor Ramon Alcantara. The Fact
That the latte, for and in consideration of P500, execute an affidavit, whereby I have ratified the
deed of sale, is of no moment. I WAS still minor. The Majority opinion Invokes the rule laid down
in the case of Mercado et al. vs. Espiritu, 37 Phil., 215. The rule laid down by this Court in That
case is based on three Judgments rendered by the Supreme Court of Spain on 27 April 1960,
11 July 1868 and 1 March 1875. These Decisions in the Supreme Court of Spain Applied Law 6,
Title 19, of the 6th Item Which expressly provides:

"In saying, or to entrust the whichever is less, it was more of XXV years if ouiesse person
paresciesse such time, if deceptively faze, would be worth the lawsuit that it may date assi e
non EUAs be released later, and that any non Quando old was he made, what, this is because
the laws help the deceived, and non of the deceitful.... " (Alcubilla, Codes Antigous of Spain, p.
613.)

The contract of sale Involved in the case of Mercado vs.. Espiritu, supra, WAS Executed by the
minors on 17 May 1910. The Law in force on this date last-Was Not Mentioned Seven Moves,
Which Was the one in force at the time the married DECIDED by the Supreme Court of Spain
'referred to, But The Civil Code Which Took effect in the Philippines on 8
December 1889. Already As state, the Civil Code Requires the Consent of Both parties for the
valid execution of a contract (art. 1261, Civil Code). As a minor historical Can not Give Consent,
the contract made by him or Executed Validity and has no legal effect. There is no provision in
the Civil Code similar to That of Law 6, Title 19, of the 6th Item Which is equivalent to the
Principle of common law estoppel. If There Be an express provision in the Civil Code 6 similar
law, Title 19, of the 6th Item, I Would Agree To The Reasoning of the Majority. The Absence of
Such provision in the Civil Code is fatal to the Validity of the contract Executed by a minor. It
Would Be Illogical to uphold the Validity of a contract on the ground of estoppel, if the contract
Because Executed by a minor is null and void for Lack of consent and produce no legal effect,
How Could Be Such a minor bound by historical misrepresentation aboutage? If I have not
could be bound by a direct act s, such as the execution of a deed of sale, How Could I Be
bound by an indirect act s, such as historical misrepresentation as to age? The rule laid down in
Young v.. Tecson, 39 O. G. 953, in my opinion, is the correct one.

Nevertheless, as the action in this case WAS Brought on 8 August 1940, barred the Same
WAS, Because It Was Not Brought Within four (4) years after the minor Had Become of age,
pursuant to article 1301 of the Civil Code. Ramon Alcantara Became of age sometime in
September 1934.

Moran, C.J. and Bengzon, J., Concur.

PAUL, M., dissenting:

I do not think Ramon Alcantara is in estoppel to want to recoup their participation in the lots that
Sia Suan ceded to the writing of August 3, 1931. The circumstances surrounding its execution
demonstrate that this conclusion is untenable. Sia Suan was the creditor and the debtor, Rufino
Alcantara per transaction he had with her in the copra business. On the death of the wife of
Rufino, someone may have noticed the difficulty of collecting the loan because Rufino had but
three lots of its exclusive property and two lots, as marital property. Ramon, one of the heirs,
was a minor.Therefore, in seeking the grant of such writing, selling the father (Rufino) and two
children (Damaso and Ramon) in five batches assessor P19, 592.85 for P2, 500, that in reality
was nothing more than a gift in payment of debt. If such writing is not granted, the creditor had
need of a long process of intestacy to obtain payment of the debt as relevant, if it could affect
the community property of Rufino Alcantara and his late wife, or care for someone act in place
of less Ramon. The shorter procedure and less expensive then was to have appeared as the
least competent age to execute the deed of sale. And so it happened: the deed. The child did
not receive a single penny. With the inheritance he had received from his late mother, paid the
debt of his father.

After Sia Suan notified of the claim of invalidity of the document, management of Gaw Chiao,
Ramon Alcantara still underage, signed an affidavit confirming the sale in the office of Attorney
Gaw Chiao. This action by Gar Chiao, Sia Suan husband complains that it was Ramon who
made them believe that was an adult and informal and has voluntarily requested the execution
of the deed of sale. If Gaw Chiao, Sia Suan husband was the one who manages the
otorgamientodel affidavit of ratification, why do not we conclude that the one who manages
indication perhaps of some lawyer, Ramon Alcantara his signature in the writing of 3 August
1931? But the signing of a minor is worthless and should appear then that Ramon was
older. Why had the least be interested in giving a deed of sale of such lands? Is not it more
likely that the creditor or her husband or any staff member who was interested was that Ramon
took part in the execution of the deed?

That benefit had the lowest in the execution of the deed? Nothing, however, the creditor got to
own the five lots in exchange for credit.Is it less than the firms favored his affidavit of
ratification? Nor, however, Sia Suan claims that the child was the one who misled. If someone
cheats to someone, there will have been Ramon. Had to be the creditor or someone who helped
her get the credit payment, but was not, could not be the lowest.

Given all these circumstances, we can not conclude that Ramon Alcantara has misled Sia
Suan. Not applicable, therefore, the decision of this Court and the Internal Market from Spirit (37
Jur Phil., 227), nor the Supreme Court of Spain, since in such cases the child and pretended
pretended to buyers that was of age: it was not just that which prompted buyers to buy land
desprendiendosedel purchase price claim is allowed after his minority to override the acting
done by. That is true estoppel, but in this case it is not.

Laches is the other foundation on which rests the majority to revoke the decision
appealed. Laches is a measure of equity, and is not applicable to this case. Must be accepted
as a defense only when application and need to make use of equity. Should not apply to foster
injustice, but to minimize its effects and should only be used as a defense when the application
of a real injustice is committed law (30 CJS, 531). In this case Ramon Alcantara has ten years
of time to count from August 3, 1931, within which to request the cancellation of the sale. And
demand that started this case occurred within this period, is not required for even the action (art.
43, Code Proc. Civ.)

Assuming Ramon Alcantara had submitted his application before the sale of a lot to Nicolas
Azores sentence had been passed?The execution of a deed of transfer of a quarter of the two
lots, but sold after a batch would be ordered, as decided by the Court of Appeal, the transfer of
a quarter of the remaining lot and pay a quarter of sale amount of the lot sold to Ramon. In
either case there is no damage to Sia Suan, but is forced to transfer to the party Ramon in the
community property inheritance left by her late mother, is responsible. No harm disproportionate
equity Sia Suan authority to invoke the defense of laches. If Sia Suan before the filing of the
lawsuit, had constructed buildings on the lots in the amount of P3, 000.000, demos by case,
maybe it would be equitable to Sia Suan invoke the defense of laches, because the silence of
Ramon Alcantara she has made valuable improvements that such a decision would be
impaired. The Ramon Alcantara live up to a quarter of each of the two lots would put her in the
alternative of buying that one quarter of the lots with overpriced or down part of the buildings. In
this case you have not been put in that difficult situation, on the contrary, she was enjoying
those two lots without making dramatic improvements, and after selling the second batch, I use
the money received, and no evidence has been caused her injury claim for failing to appear
earlier.
Vote for the confirmation of the decision of the Court of Apalacion.

QUITA VS DANDAN
SECOND DIVISION

[G.R. No. 124862. December 22, 1998]

FE D. QUITA, petitioner, vs. COURT OF APPEALS and BLANDINA


DANDAN,* respondents.

DECISION
BELLOSILLO, J .:

FE D. QUITA and Arturo T. Padlan, both Filipinos, were married in the Philippines on 18
May 1941. They were not however blessed with children. Somewhere along the way their
relationship soured. Eventually Fe sued Arturo for divorce in San Francisco, California,
U.S.A. She submitted in the divorce proceedings a private writing dated 19 July 1950
evidencing their agreement to live separately from each other and a settlement of their conjugal
properties. On 23 July 1954 she obtained a final judgment of divorce. Three (3) weeks
thereafter she married a certain Felix Tupaz in the same locality but their relationship also
ended in a divorce. Still in the U.S.A., she married for the third time, to a certain Wernimont.
On 16 April 1972 Arturo died. He left no will. On 31 August 1972 Lino Javier Inciong filed a
petition with the Regional Trial Court of Quezon City for issuance of letters of administration
concerning the estate of Arturo in favor of the Philippine Trust Company. Respondent Blandina
Dandan (also referred to asBlandina Padlan), claiming to be the surviving spouse of Arturo
Padlan, and Claro, Alexis, Ricardo, Emmanuel, Zenaida and Yolanda, all surnamed Padlan,
named in the petition as surviving children of Arturo Padlan, opposed the petition and prayed for
the appointment instead of Atty. Leonardo Cabasal, which was resolved in favor of the
latter. Upon motion of the oppositors themselves, Atty. Cabasal was later replaced by Higino
Castillon. On 30 April 1973 the oppositors (Blandina and the Padlan children) submitted
certified photocopies of the 19 July 1950 private writing and the final judgment of divorce
between petitioner and Arturo. Later Ruperto T. Padlan, claiming to be the sole surviving
brother of the deceased Arturo, intervened.
On 7 October 1987 petitioner moved for the immediate declaration of heirs of the decedent
and the distribution of his estate. At the scheduled hearing on 23 October 1987, private
respondent as well as the six (6) Padlan children and Ruperto failed to appear despite due
notice. On the same day, the trial court required the submission of the records of birth of the
Padlan children within ten (10) days from receipt thereof, after which, with or without the
documents, the issue on the declaration of heirs would be considered submitted for
resolution. The prescribed period lapsed without the required documents being submitted.
The trial court invoking Tenchavez v. Escaño[1] which held that "a foreign divorce between
Filipino citizens sought and decreed after the effectivity of the present Civil Code (Rep. Act
386) was not entitled to recognition as valid in this jurisdiction," [2] disregarded the divorce
between petitioner and Arturo. Consequently, it expressed the view that their marriage
subsisted until the death of Arturo in 1972. Neither did it consider valid their extrajudicial
settlement of conjugal properties due to lack of judicial approval. [3] On the other hand, it opined
that there was no showing that marriage existed between private respondent and Arturo, much
less was it shown that the alleged Padlan children had been acknowledged by the deceased as
his children with her. As regards Ruperto, it found that he was a brother of Arturo. On 27
November 1987[4] only petitioner and Ruperto were declared the intestate heirs of
Arturo. Accordingly, equal adjudication of the net hereditary estate was ordered in favor of the
two intestate heirs.[5]
On motion for reconsideration, Blandina and the Padlan children were allowed to present
proofs that the recognition of the children by the deceased as his legitimate children, except
Alexis who was recognized as his illegitimate child, had been made in their respective records
of birth. Thus on 15 February 1988[6]partial reconsideration was granted declaring the Padlan
children, with the exception of Alexis, entitled to one-half of the estate to the exclusion of
Ruperto Padlan, and petitioner to the other half. [7] Private respondent was not declared an
heir. Although it was stated in the aforementioned records of birth that she and Arturo were
married on 22 April 1947, their marriage was clearly void since it was celebrated during the
existence of his previous marriage to petitioner.
In their appeal to the Court of Appeals, Blandina and her children assigned as one of the
errors allegedly committed by the trial court the circumstance that the case was decided without
a hearing, in violation of Sec. 1, Rule 90, of the Rules of Court, which provides that if there is a
controversy before the court as to who are the lawful heirs of the deceased person or as to the
distributive shares to which each person is entitled under the law, the controversy shall be heard
and decided as in ordinary cases.
Respondent appellate court found this ground alone sufficient to sustain the appeal; hence,
on 11 September 1995 it declared null and void the 27 November 1987 decision and 15
February 1988 order of the trial court, and directed the remand of the case to the trial court for
further proceedings.[8] On 18 April 1996 it denied reconsideration. [9]
Should this case be remanded to the lower court for further proceedings? Petitioner insists
that there is no need because, first, no legal or factual issue obtains for resolution either as to
the heirship of the Padlan children or as to their respective shares in the intestate estate of the
decedent; and, second, the issue as to who between petitioner and private respondent is the
proper heir of the decedent is one of law which can be resolved in the present petition based on
established facts and admissions of the parties.
We cannot sustain petitioner. The provision relied upon by respondent court is clear: If
there is a controversy before the court as to who are the lawful heirs of the deceased person or
as to the distributive shares to which each person is entitled under the law, the controversy shall
be heard and decided as in ordinary cases.
We agree with petitioner that no dispute exists either as to the right of the six (6) Padlan
children to inherit from the decedent because there are proofs that they have been duly
acknowledged by him and petitioner herself even recognizes them as heirs of Arturo
Padlan;[10] nor as to their respective hereditary shares. But controversy remains as to who is the
legitimate surviving spouse of Arturo. The trial court, after the parties other than petitioner failed
to appear during the scheduled hearing on 23 October 1987 of the motion for immediate
declaration of heirs and distribution of estate, simply issued an order requiring the submission of
the records of birth of the Padlan children within ten (10) days from receipt thereof, after which,
with or without the documents, the issue on declaration of heirs would be deemed submitted for
resolution.
We note that in her comment to petitioner's motion private respondent raised, among
others, the issue as to whether petitioner was still entitled to inherit from the decedent
considering that she had secured a divorce in the U.S.A. and in fact had twice remarried. She
also invoked the above quoted procedural rule. [11] To this, petitioner replied that Arturo was a
Filipino and as such remained legally married to her in spite of the divorce they
obtained.[12] Reading between the lines, the implication is that petitioner was no longer a Filipino
citizen at the time of her divorce from Arturo. This should have prompted the trial court to
conduct a hearing to establish her citizenship. The purpose of a hearing is to ascertain the truth
of the matters in issue with the aid of documentary and testimonial evidence as well as the
arguments of the parties either supporting or opposing the evidence. Instead, the lower court
perfunctorily settled her claim in her favor by merely applying the ruling in Tenchavez v.
Escaño.
Then in private respondent's motion to set aside and/or reconsider the lower court's
decision she stressed that the citizenship of petitioner was relevant in the light of the ruling
in Van Dorn v. Romillo Jr.[13] that aliens may obtain divorces abroad, which may be recognized
in the Philippines, provided they are valid according to their national law. She prayed therefore
that the case be set for hearing. [14] Petitioner opposed the motion but failed to squarely address
the issue on her citizenship.[15] The trial court did not grant private respondent's prayer for a
hearing but proceeded to resolve her motion with the finding that both petitioner and Arturo were
"Filipino citizens and were married in the Philippines." [16] It maintained that their divorce obtained
in 1954 in San Francisco, California, U.S.A., was not valid in Philippine jurisdiction. We deduce
that the finding on their citizenship pertained solely to the time of their marriage as the trial
court was not supplied with a basis to determine petitioner's citizenship at the time of
their divorce. The doubt persisted as to whether she was still a Filipino citizen when their
divorce was decreed. The trial court must have overlooked the materiality of this aspect. Once
proved that she was no longer a Filipino citizen at the time of their divorce,Van Dorn would
become applicable and petitioner could very well lose her right to inherit from Arturo.
Respondent again raised in her appeal the issue on petitioner's citizenship; [17] it did not
merit enlightenment however from petitioner. [18] In the present proceeding, petitioner's
citizenship is brought anew to the fore by private respondent. She even furnishes the Court
with the transcript of stenographic notes taken on 5 May 1995 during the hearing for the
reconstitution of the original of a certain transfer certificate title as well as the issuance of new
owner's duplicate copy thereof before another trial court. When asked whether she was an
American citizen petitioner answered that she was since 1954. [19] Significantly, the decree of
divorce of petitioner and Arturo was obtained in the same year. Petitioner however did not
bother to file a reply memorandum to erase the uncertainty about her citizenship at the time of
their divorce, a factual issue requiring hearings to be conducted by the trial
court. Consequently, respondent appellate court did not err in ordering the case returned to the
trial court for further proceedings.
We emphasize however that the question to be determined by the trial court should be
limited only to the right of petitioner to inherit from Arturo as his surviving spouse. Private
respondent's claim to heirship was already resolved by the trial court. She and Arturo were
married on 22 April 1947 while the prior marriage of petitioner and Arturo was subsisting
thereby resulting in a bigamous marriage considered void from the beginning under Arts. 80 and
83 of the Civil Code. Consequently, she is not a surviving spouse that can inherit from him as
this status presupposes a legitimate relationship. [20]
As regards the motion of private respondent for petitioner and her counsel to be declared in
contempt of court and that the present petition be dismissed for forum shopping, [21] the same
lacks merit. For forum shopping to exist the actions must involve the same transactions and
same essential facts and circumstances. There must also be identical causes of action, subject
matter and issue.[22] The present petition deals with declaration of heirship while the subsequent
petitions filed before the three (3) trial courts concern the issuance of new owner's duplicate
copies of titles of certain properties belonging to the estate of Arturo. Obviously, there is no
reason to declare the existence of forum shopping.
WHEREFORE, the petition is DENIED. The decision of respondent Court of Appeals
ordering the remand of the case to the court of origin for further proceedings and declaring null
and void its decision holding petitioner Fe D. Quita and Ruperto T. Padlan as intestate heirs
is AFFIRMED. The order of the appellate court modifying its previous decision by granting
one-half (1/2) of the net hereditary estate to the Padlan children, namely, Claro, Ricardo,
Emmanuel, Zenaida and Yolanda, with the exception of Alexis, all surnamed Padlan, instead of
Arturo's brother Ruperto Padlan, is likewise AFFIRMED. The Court however emphasizes that
the reception of evidence by the trial court should be limited to the hereditary rights of petitioner
as the surviving spouse of Arturo Padlan.
The motion to declare petitioner and her counsel in contempt of court and to dismiss the
present petition for forum shopping is DENIED.
SO ORDERED.
Puno, Mendoza, and Martinez, JJ., concur.

*
The name of private respondent Blandina Dandan appears as Blandina Padlan in the
proceedings before the lower courts.
[1]
No. L-19671, 29 November 1965, 15 SCRA 355.
[2]
Id., p. 367.
[3]
Then Art. 190 of the Civil Code provided that in the absence of an express declaration in the
marriage settlement, the separation of property between spouses during the marriage shall not
take place save in virtue of a judicial order. Quite in relation thereto, then Art. 191, par. 4 of the
same Code provided that the husband and the wife may agree upon the dissolution of the
conjugal partnership during the marriage, subject to judicial approval.
[4]
Decision penned by Judge Tomas V. Tadeo Jr. of RTC-Br. 105, Quezon City; Appendix "A" of
Brief for the Oppositors-Appellants; CA Rollo, p. 15.
[5]
Article 1001 of the Civil Code provides that should brothers and sisters or their children
survive with the widow or widower, the latter shall be entitled to one-half of the inheritance and
the brothers and sisters or their children to the other half.
[6]
Appendix "B" of Brief for the Oppositors-Appellants; See Note 4.
[7]
Article 998 of the Civil Code provides that if a widow or widower survives with illegitimate
children, such widow or widower shall be entitled to one-half of the inheritance, and the
illegitimate children or their descendants, whether legitimate or illegitimate, to the other half.
[8]
Decision penned by Justice Pacita Cañizares-Nye with the concurrence of Justices Romeo J.
Callejo Jr. and Delilah Vidallon-Magtolis; Rollo, p. 39.
[9]
Id., p. 42.
[10]
Id., p. 180.
[11]
Rollo, p. 196.
[12]
CA Rollo, p. 29.
[13]
G. R. No. 68470, 8 October 1985, 139 SCRA 139.
[14]
CA Rollo, p. 30.
[15]
Record on Appeal, pp. 24-26.
[16]
Rollo, p. 206.
[17]
Brief of Oppositors-Appellants, p. 13; CA Rollo, p. 15.
[18]
Brief of Appellee; Id., p. 17.
[19]
Rollo, pp. 225-226.
[20]
Arturo M. Tolentino, Commentaries and Jurisprudence on the Civil Code of the Philippines,
1979 Ed., Vol. III, p. 264.
[21]
Rollo, pp. 129-132.
[22]
Professional Regulation Commission v. Court of Appeals, G. R. No. 117817, 9 July 1998.

LLORENTE VS CA

FIRST DIVISION

[G.R. No. 124371. November 23, 2000]

PAULA T. LLORENTE, petitioner, vs. COURT OF APPEALS and ALICIA F.


LLORENTE, respondents.

DECISION
PARDO, J.:

The Case

The case raises a conflict of laws issue.


What is before us is an appeal from the decision of the Court of Appeals[1] modifying that of
the Regional Trial Court, Camarines Sur, Branch 35, Iriga City [2] declaring respondent Alicia F.
Llorente (herinafter referred to as ―Alicia‖), as co-owners of whatever property she and the
deceased Lorenzo N. Llorente (hereinafter referred to as ―Lorenzo‖) may have acquired during
the twenty-five (25) years that they lived together as husband and wife.
The Facts

The deceased Lorenzo N. Llorente was an enlisted serviceman of the United States Navy
from March 10, 1927 to September 30, 1957. [3]
On February 22, 1937, Lorenzo and petitioner Paula Llorente (hereinafter referred to as
―Paula‖) were married before a parish priest, Roman Catholic Church, in Nabua, Camarines
Sur.[4]
Before the outbreak of the Pacific War, Lorenzo departed for the United States and Paula
stayed in the conjugal home in barrio Antipolo, Nabua, Camarines Sur. [5]
On November 30, 1943, Lorenzo was admitted to United States citizenship and Certificate
of Naturalization No. 5579816 was issued in his favor by the United States District Court,
Southern District of New York.[6]
Upon the liberation of the Philippines by the American Forces in 1945, Lorenzo was
granted an accrued leave by the U. S. Navy, to visit his wife and he visited the Philippines. [7] He
discovered that his wife Paula was pregnant and was ―living in‖ and having an adulterous
relationship with his brother, Ceferino Llorente. [8]
On December 4, 1945, Paula gave birth to a boy registered in the Office of the Registrar of
Nabua as ―Crisologo Llorente,‖ with the certificate stating that the child was not legitimate and
the line for the father‘s name was left blank. [9]
Lorenzo refused to forgive Paula and live with her. In fact, on February 2, 1946, the couple
drew a written agreement to the effect that (1) all the family allowances allotted by the United
States Navy as part of Lorenzo‘s salary and all other obligations for Paula‘s daily maintenance
and support would be suspended; (2) they would dissolve their marital union in accordance with
judicial proceedings; (3) they would make a separate agreement regarding their conjugal
property acquired during their marital life; and (4) Lorenzo would not prosecute Paula for her
adulterous act since she voluntarily admitted her fault and agreed to separate from Lorenzo
peacefully. The agreement was signed by both Lorenzo and Paula and was witnessed by
Paula‘s father and stepmother. The agreement was notarized by Notary Public Pedro
Osabel.[10]
Lorenzo returned to the United States and on November 16, 1951 filed for
divorce with the Superior Court of the State of California in and for the County of San
Diego. Paula was represented by counsel, John Riley, and actively participated in the
proceedings. On November 27, 1951, the Superior Court of the State of California, for the
County of San Diego found all factual allegations to be true and issued an interlocutory
judgment of divorce.[11]
On December 4, 1952, the divorce decree became final. [12]
In the meantime, Lorenzo returned to the Philippines.
On January 16, 1958, Lorenzo married Alicia F. Llorente in Manila. [13] Apparently, Alicia had
no knowledge of the first marriage even if they resided in the same town as Paula, who did not
oppose the marriage or cohabitation. [14]
From 1958 to 1985, Lorenzo and Alicia lived together as husband and wife. [15] Their twenty-
five (25) year union produced three children, Raul, Luz and Beverly, all surnamed Llorente. [16]
On March 13, 1981, Lorenzo executed a Last Will and Testament. The will was notarized
by Notary Public Salvador M. Occiano, duly signed by Lorenzo with attesting witnesses
Francisco Hugo, Francisco Neibres and Tito Trajano. In the will, Lorenzo bequeathed all his
property to Alicia and their three children, to wit:

―(1) I give and bequeath to my wife ALICIA R. FORTUNO exclusively my residential house and
lot, located at San Francisco, Nabua, Camarines Sur, Philippines, including ALL the personal
properties and other movables or belongings that may be found or existing therein;

―(2) I give and bequeath exclusively to my wife Alicia R. Fortuno and to my children, Raul F.
Llorente, Luz F. Llorente and Beverly F. Llorente, in equal shares, all my real properties
whatsoever and wheresoever located, specifically my real properties located at Barangay Aro-
Aldao, Nabua, Camarines Sur; Barangay Paloyon, Nabua, Camarines Sur; Barangay Baras,
Sitio Puga, Nabua, Camarines Sur; and Barangay Paloyon, Sitio Nalilidong, Nabua, Camarines
Sur;

―(3) I likewise give and bequeath exclusively unto my wife Alicia R. Fortuno and unto my
children, Raul F. Llorente, Luz F. Llorente and Beverly F. Llorente, in equal shares, my real
properties located in Quezon City Philippines, and covered by Transfer Certificate of Title No.
188652; and my lands in Antipolo, Rizal, Philippines, covered by Transfer Certificate of Title
Nos. 124196 and 165188, both of the Registry of Deeds of the province of Rizal, Philippines;

―(4) That their respective shares in the above-mentioned properties, whether real or personal
properties, shall not be disposed of, ceded, sold and conveyed to any other persons, but could
only be sold, ceded, conveyed and disposed of by and among themselves;

―(5) I designate my wife ALICIA R. FORTUNO to be the sole executor of this my Last Will and
Testament, and in her default or incapacity of the latter to act, any of my children in the order of
age, if of age;

―(6) I hereby direct that the executor named herein or her lawful substitute should served (sic)
without bond;

―(7) I hereby revoke any and all my other wills, codicils, or testamentary dispositions heretofore
executed, signed, or published, by me;

―(8) It is my final wish and desire that if I die, no relatives of mine in any degree in the Llorente‘s
Side should ever bother and disturb in any manner whatsoever my wife Alicia R. Fortunato and
my children with respect to any real or personal properties I gave and bequeathed respectively
to each one of them by virtue of this Last Will and Testament.‖ [17]

On December 14, 1983, Lorenzo filed with the Regional Trial Court, Iriga, Camarines Sur, a
petition for the probate and allowance of his last will and testament wherein Lorenzo moved that
Alicia be appointed Special Administratrix of his estate. [18]
On January 18, 1984, the trial court denied the motion for the reason that the testator
Lorenzo was still alive.[19]
On January 24, 1984, finding that the will was duly executed, the trial court admitted the will
to probate.[20]
On June 11, 1985, before the proceedings could be terminated, Lorenzo died. [21]
On September 4, 1985, Paula filed with the same court a petition [22] for letters of
administration over Lorenzo‘s estate in her favor. Paula contended (1) that she was Lorenzo‘s
surviving spouse, (2) that the various property were acquired during their marriage, (3) that
Lorenzo‘s will disposed of all his property in favor of Alicia and her children, encroaching on her
legitime and 1/2 share in the conjugal property.[23]
On December 13, 1985, Alicia filed in the testate proceeding (Sp. Proc. No. IR-755), a
petition for the issuance of letters testamentary. [24]
On October 14, 1985, without terminating the testate proceedings, the trial court gave due
course to Paula‘s petition in Sp. Proc. No. IR-888.[25]
On November 6, 13 and 20, 1985, the order was published in the newspaper ―Bicol Star‖. [26]
On May 18, 1987, the Regional Trial Court issued a joint decision, thus:

―Wherefore, considering that this court has so found that the divorce decree granted to the late
Lorenzo Llorente is void and inapplicable in the Philippines, therefore the marriage he
contracted with Alicia Fortunato on January 16, 1958 at Manila is likewise void. This being so
the petition of Alicia F. Llorente for the issuance of letters testamentary is denied. Likewise, she
is not entitled to receive any share from the estate even if the will especially said so her
relationship with Lorenzo having gained the status of paramour which is under Art. 739 (1).

―On the other hand, the court finds the petition of Paula Titular Llorente, meritorious, and so
declares the intrinsic disposition of the will of Lorenzo Llorente dated March 13, 1981 as void
and declares her entitled as conjugal partner and entitled to one-half of their conjugal properties,
and as primary compulsory heir, Paula T. Llorente is also entitled to one-third of the estate and
then one-third should go to the illegitimate children, Raul, Luz and Beverly, all surname (sic)
Llorente, for them to partition in equal shares and also entitled to the remaining free portion in
equal shares.

―Petitioner, Paula Llorente is appointed legal administrator of the estate of the deceased,
Lorenzo Llorente. As such let the corresponding letters of administration issue in her favor upon
her filing a bond in the amount (sic) of P100,000.00 conditioned for her to make a return to the
court within three (3) months a true and complete inventory of all goods, chattels, rights, and
credits, and estate which shall at any time come to her possession or to the possession of any
other person for her, and from the proceeds to pay and discharge all debts, legacies and
charges on the same, or such dividends thereon as shall be decreed or required by this court; to
render a true and just account of her administration to the court within one (1) year, and at any
other time when required by the court and to perform all orders of this court by her to be
performed.

―On the other matters prayed for in respective petitions for want of evidence could not be
granted.

―SO ORDERED.‖[27]

In time, Alicia filed with the trial court a motion for reconsideration of the aforequoted
decision.[28]
On September 14, 1987, the trial court denied Alicia‘s motion for reconsideration but
modified its earlier decision, stating that Raul and Luz Llorente are not children ―legitimate or
otherwise‖ of Lorenzo since they were not legally adopted by him. [29] Amending its decision of
May 18, 1987, the trial court declared Beverly Llorente as the only illegitimate child of Lorenzo,
entitling her to one-third (1/3) of the estate and one-third (1/3) of the free portion of the estate. [30]
On September 28, 1987, respondent appealed to the Court of Appeals. [31]
On July 31, 1995, the Court of Appeals promulgated its decision, affirming with modification
the decision of the trial court in this wise:

―WHEREFORE, the decision appealed from is hereby AFFIRMED with the MODIFICATION that
Alicia is declared as co-owner of whatever properties she and the deceased may have acquired
during the twenty-five (25) years of cohabitation.

―SO ORDERED.‖[32]

On August 25, 1995, petitioner filed with the Court of Appeals a motion for reconsideration
of the decision.[33]
On March 21, 1996, the Court of Appeals, [34] denied the motion for lack of merit.
Hence, this petition.[35]

The Issue

Stripping the petition of its legalese and sorting through the various arguments
raised,[36] the issue is simple. Who are entitled to inherit from the late Lorenzo N. Llorente?
We do not agree with the decision of the Court of Appeals. We remand the case to the trial
court for ruling on the intrinsic validity of the will of the deceased.

The Applicable Law

The fact that the late Lorenzo N. Llorente became an American citizen long before and at
the time of: (1) his divorce from Paula; (2) marriage to Alicia; (3) execution of his will; and (4)
death, is duly established, admitted and undisputed.
Thus, as a rule, issues arising from these incidents are necessarily governed by foreign
law.
The Civil Code clearly provides:

―Art. 15. Laws relating to family rights and duties, or to the status, condition and legal capacity
of persons are binding upon citizens of the Philippines, even though living abroad.

―Art. 16. Real property as well as personal property is subject to the law of the country where it
is situated.
―However, intestate and testamentary succession, both with respect to the order of succession
and to the amount of successional rights and to the intrinsic validity of testamentary
provisions, shall be regulated by the national law of the person whose succession is
under consideration, whatever may be the nature of the property and regardless of the country
wherein said property may be found.‖ (emphasis ours)

True, foreign laws do not prove themselves in our jurisdiction and our courts are not
authorized to take judicial notice of them. Like any other fact, they must be alleged and
proved.[37]
While the substance of the foreign law was pleaded, the Court of Appeals did not admit the
foreign law. The Court of Appeals and the trial court called to the fore the renvoi doctrine,
where the case was ―referred back‖ to the law of the decedent‘s domicile, in this case, Philippine
law.
We note that while the trial court stated that the law of New York was not sufficiently
proven, in the same breath it made the categorical, albeit equally unproven statement that
―American law follows the ‗domiciliary theory‘ hence, Philippine law applies when determining
the validity of Lorenzo‘s will.[38]
First, there is no such thing as one American law. The "national law" indicated in Article 16
of the Civil Code cannot possibly apply to general American law. There is no such law
governing the validity of testamentary provisions in the United States. Each State of the union
has its own law applicable to its citizens and in force only within the State. It can therefore refer
to no other than the law of the State of which the decedent was a resident. [39] Second, there is
no showing that the application of the renvoi doctrine is called for or required by New York State
law.
The trial court held that the will was intrinsically invalid since it contained dispositions in
favor of Alice, who in the trial court‘s opinion was a mere paramour. The trial court threw the will
out, leaving Alice, and her two children, Raul and Luz, with nothing.
The Court of Appeals also disregarded the will. It declared Alice entitled to one half (1/2) of
whatever property she and Lorenzo acquired during their cohabitation, applying Article 144 of
the Civil Code of the Philippines.
The hasty application of Philippine law and the complete disregard of the will, already
probated as duly executed in accordance with the formalities of Philippine law, is
fatal, especially in light of the factual and legal circumstances here obtaining.

Validity of the Foreign Divorce

In Van Dorn v. Romillo, Jr.[40] we held that owing to the nationality principle embodied in
Article 15 of the Civil Code, only Philippine nationals are covered by the policy against absolute
divorces, the same being considered contrary to our concept of public policy and morality. In the
same case, the Court ruled that aliens may obtain divorces abroad, provided they are valid
according to their national law.
Citing this landmark case, the Court held in Quita v. Court of Appeals,[41] that once proven
that respondent was no longer a Filipino citizen when he obtained the divorce from petitioner,
the ruling in Van Dorn would become applicable and petitioner could ―very well lose her right to
inherit‖ from him.
In Pilapil v. Ibay-Somera,[42] we recognized the divorce obtained by the respondent in his
country, the Federal Republic of Germany. There, we stated that divorce and its legal effects
may be recognized in the Philippines insofar as respondent is concerned in view of the
nationality principle in our civil law on the status of persons.
For failing to apply these doctrines, the decision of the Court of Appeals must be
reversed.[43] We hold that the divorce obtained by Lorenzo H. Llorente from his first wife Paula
was valid and recognized in this jurisdiction as a matter of comity. Now, the effects of this
divorce (as to the succession to the estate of the decedent) are matters best left to the
determination of the trial court.

Validity of the Will

The Civil Code provides:

―Art. 17. The forms and solemnities of contracts, wills, and other public instruments shall be
governed by the laws of the country in which they are executed.

―When the acts referred to are executed before the diplomatic or consular officials of the
Republic of the Philippines in a foreign country, the solemnities established by Philippine laws
shall be observed in their execution.‖ (underscoring ours)

The clear intent of Lorenzo to bequeath his property to his second wife and children by her
is glaringly shown in the will he executed. We do not wish to frustrate his wishes, since he was
a foreigner, not covered by our laws on ―family rights and duties, status, condition and legal
capacity.‖[44]
Whether the will is intrinsically valid and who shall inherit from Lorenzo are issues best
proved by foreign law which must be pleaded and proved. Whether the will was executed in
accordance with the formalities required is answered by referring to Philippine law. In fact, the
will was duly probated.
As a guide however, the trial court should note that whatever public policy or good customs
may be involved in our system of legitimes, Congress did not intend to extend the same to the
succession of foreign nationals. Congress specifically left the amount of successional rights to
the decedent's national law.[45]
Having thus ruled, we find it unnecessary to pass upon the other issues raised.

The Fallo

WHEREFORE, the petition is GRANTED. The decision of the Court of Appeals in CA-G. R.
SP No. 17446 promulgated on July 31, 1995 is SET ASIDE.
In lieu thereof, the Court REVERSES the decision of the Regional Trial Court and
RECOGNIZES as VALID the decree of divorce granted in favor of the deceased Lorenzo N.
Llorente by the Superior Court of the State of California in and for the County of San Diego,
made final on December 4, 1952.
Further, the Court REMANDS the cases to the court of origin for determination of the
intrinsic validity of Lorenzo N. Llorente‘s will and determination of the parties‘ successional
rights allowing proof of foreign law with instructions that the trial court shall proceed with all
deliberate dispatch to settle the estate of the deceased within the framework of the Rules of
Court.
No costs.
SO ORDERED.
Davide, Jr., C.J., (Chairman), Puno, Kapunan, and Ynares-Santiago, JJ., concur.

[1]
In CA-G. R. SP. No. 17446, promulgated on July 31, 1995, Lipana-Reyes+, J., ponente,
Torres, Jr. and Hofilena, JJ., concurring.
[2]
In Spec. Proc. No. IR-755 (In the Matter of the Probate and Allowance of the Last Will and
Testament of Lorenzo N. Llorente, Lorenzo N. Llorente, Petitioner) and Spec. Proc. No. IR-
888 (Petition for the Grant of Letters of Administration for the Estate of Lorenzo N. Llorente,
Paula T. Llorente, Petitioner), dated May 18, 1987, Judge Esteban B. Abonal, presiding.
[3]
Decision, Court of Appeals, Rollo, p. 51.
[4]
Exh. ―B‖, Trial Court Folder of Exhibits, p. 61.
[5]
Ibid.
[6]
This was issued pursuant to Lorenzo‘s petition, Petition No. 4708849, filed with the U.S.
Court. Exhs. ―H‖ and ―H-3‖ Trial Court Folder of Exhibits, p. 157, 159.
[7]
Decision, Court of Appeals, Rollo, p. 51; Exh. ―B‖, Trial Court Folder of Exhibits, p. 61.
[8]
Ibid.
[9]
Exh. ―A‖, Trial Court Folder of Exhibits, p. 60.
[10]
Exh. ―B-1‖ Trial Court Folder of Exhibits, p. 62.
[11]
Exh. ―D‖, Trial Court Folder of Exhibits, pp. 63-64.
[12]
Exh. ―E‖, Trial Court Folder of Exhibits, p. 69.
[13]
Exh. ―F‖, Trial Court Folder of Exhibits, p. 148.
[14]
Decision, Court of Appeals, Rollo, p. 52.
[15]
Comment, Rollo, p. 147.
[16]
Decision, Court of Appeals, Rollo, p. 52.
[17]
Exh. ―A‖, Trial Court Folder of Exhibits, pp. 3-4; Decision, Court of Appeals, Rollo, p. 52.
[18]
Docketed as Spec. Proc. No. IR-755.
[19]
Decision, RTC, Rollo, p. 37.
[20]
Ibid.
[21]
Ibid.
[22]
Docketed as Spec. Proc. No. IR-888.
[23]
Decision, RTC, Rollo, p. 38.
[24]
Decision, Court of Appeals, Rollo, p. 52.
[25]
Ibid., pp. 52-53.
[26]
Ibid., p. 53.
[27]
RTC Decision, Rollo, p. 37.
[28]
Order, Regional Trial Court in Spec. Proc. Nos. IR-755 and 888, Rollo, p. 46.
[29]
Citing Article 335 of the Civil Code, which states, ―The following cannot adopt: xxx
(3) a married person, without the consent of the other spouse; xxx‖, the trial court reasoned that
since the divorce obtained by Lorenzo did not dissolve his first marriage with Paula, then the
adoption of Raul and Luz was void, as Paula did not give her consent to it.
[30]
Order, Regional Trial Court, Rollo, p. 47.
[31]
Docketed as CA-G. R. SP No. 17446.
[32]
Decision, Court of Appeals, Rollo, p. 56.
[33]
On August 31, 1995, petitioner also filed with this Court a verified complaint against the
members of the Special Thirteenth Division, Court of Appeals, Associate Justices Justo P.
Torres, Jr., Celia Lipana-Reyes + and Hector Hofilena for ―gross ignorance of the
law, manifest incompetence and extreme bias (Rollo, p. 15).‖
[34]
Again with Associate Justice Celia Lipana-Reyes+, ponente, concurred in by Associate
Justices Justo P. Torres, Jr. and Hector Hofilena (Former Special Thirteenth Division).
[35]
Filed on May 10, 1996, Rollo, pp. 9-36.
[36]
Petitioner alleges (1) That the Court of Appeals lost its jurisdiction over the case when it
issued the resolution denying the motion for reconsideration; (2) That Art. 144 of the Civil Case
has been repealed by Arts. 253 and 147 of the Family Code and (3) That Alicia and her children
not are entitled to any share in the estate of the deceased (Rollo, p. 19).
[37]
Collector of Internal Revenue v. Fisher, 110 Phil. 686 (1961).
[38]
Joint Record on Appeal, p. 255; Rollo, p. 40.
[39]
In Re: Estate of Edward Christensen, Aznar v. Helen Garcia, 117 Phil. 96 (1963).
[40]
139 SCRA 139 (1985).
[41]
300 SCRA 406 (1998).
[42]
174 SCRA 653 (1989).
[43]
The ruling in the case of Tenchavez v. Escano (122 Phil. 752 [1965]) that provides that ―a
foreign divorce between Filipino citizens sought and decreed after the effectivity of the present
civil code is not entitled to recognition as valid in this jurisdiction‖ is NOT applicable in the case
at bar as Lorenzo was no longer a Filipino citizen when he obtained the divorce.
[44]
Article 15, Civil Code provides ―Laws relating to family rights and duties, or to the status,
condition and legal capacity of persons are binding upon citizens of the Philippines, even though
living abroad.‖ (Underscoring ours)
[45]
Bellis v. Bellis, 126 Phil. 726 (1967).

BAYOT VS CA

Republic of the Philippines


SUPREME COURT
Manila

SECOND DIVISION

G.R. No. 155635 November 7, 2008

MARIA REBECCA MAKAPUGAY BAYOT, petitioner,


vs.
THE HONORABLE COURT OF APPEALS and VICENTE MADRIGAL BAYOT, respondents.

x-------------------------------------------x

G.R. No. 163979 November 7, 2008

MARIA REBECCA MAKAPUGAY BAYOT, petitioner,


vs.
VICENTE MADRIGAL BAYOT, respondent.

DECISION

VELASCO, JR., J.:

The Case

Before us are these two petitions interposed by petitioner Maria Rebecca Makapugay Bayot
impugning certain issuances handed out by the Court of Appeals (CA) in CA-G.R. SP No.
68187.

In the first, a petition for certiorari1 under Rule 65 and docketed as G.R. No.
155635, Rebecca assails and seeks to nullify the April 30, 2002 Resolution 2 of the CA, as
reiterated in another Resolution of September 2, 2002, 3 granting a writ of preliminary injunction
in favor of private respondent Vicente Madrigal Bayot staving off the trial court's grant of
support pendente lite to Rebecca.

The second, a petition for review under Rule 45, 4 docketed G.R. No. 163979, assails the March
25, 2004 Decision5 of the CA, (1) dismissing Civil Case No. 01-094, a suit for declaration of
absolute nullity of marriage with application for support commenced by Rebecca against Vicente
before the Regional Trial Court (RTC) in Muntinlupa City; and (2) setting aside certain orders
and a resolution issued by the RTC in the said case.

Per its Resolution of August 11, 2004, the Court ordered the consolidation of both cases.
The Facts

Vicente and Rebecca were married on April 20, 1979 in Sanctuario de San Jose, Greenhills,
Mandaluyong City. On its face, the Marriage Certificate 6 identified Rebecca, then 26 years old,
to be an American citizen7 born in Agaña, Guam, USA to Cesar Tanchiong Makapugay,
American, and Helen Corn Makapugay, American.

On November 27, 1982 in San Francisco, California, Rebecca gave birth to Marie Josephine
Alexandra or Alix. From then on, Vicente and Rebecca's marital relationship seemed to have
soured as the latter, sometime in 1996, initiated divorce proceedings in the Dominican Republic.
Before the Court of the First Instance of the Judicial District of Santo Domingo, Rebecca
personally appeared, while Vicente was duly represented by counsel. On February 22, 1996,
the Dominican court issued Civil Decree No. 362/96,8ordering the dissolution of the couple's
marriage and "leaving them to remarry after completing the legal requirements," but giving them
joint custody and guardianship over Alix. Over a year later, the same court would issue Civil
Decree No. 406/97,9 settling the couple's property relations pursuant to an Agreement 10 they
executed on December 14, 1996. Said agreement specifically stated that the "conjugal property
which they acquired during their marriage consist[s] only of the real property and all the
improvements and personal properties therein contained at 502 Acacia Avenue, Alabang,
Muntinlupa."11

Meanwhile, on March 14, 1996, or less than a month from the issuance of Civil Decree No.
362/96, Rebecca filed with the Makati City RTC a petition 12 dated January 26, 1996, with
attachments, for declaration of nullity of marriage, docketed as Civil Case No. 96-378. Rebecca,
however, later moved13and secured approval14 of the motion to withdraw the petition.

On May 29, 1996, Rebecca executed an Affidavit of Acknowledgment 15 stating under oath that
she is an American citizen; that, since 1993, she and Vicente have been living separately; and
that she is carrying a child not of Vicente.

On March 21, 2001, Rebecca filed another petition, this time before the Muntinlupa City RTC,
for declaration of absolute nullity of marriage 16 on the ground of Vicente's alleged psychological
incapacity. Docketed as Civil Case No. 01-094 and entitled as Maria Rebecca Makapugay
Bayot v. Vicente Madrigal Bayot, the petition was eventually raffled to Branch 256 of the court.
In it, Rebecca also sought the dissolution of the conjugal partnership of gains with application
for support pendente lite for her and Alix. Rebecca also prayed that Vicente be ordered to pay a
permanent monthly support for their daughter Alix in the amount of PhP 220,000.

On June 8, 2001, Vicente filed a Motion to Dismiss 17 on, inter alia, the grounds of lack of cause
of action and that the petition is barred by the prior judgment of divorce. Earlier, on June 5,
2001, Rebecca filed and moved for the allowance of her application for support pendente lite.

To the motion to dismiss, Rebecca interposed an opposition, insisting on her Filipino citizenship,
as affirmed by the Department of Justice (DOJ), and that, therefore, there is no valid divorce to
speak of.

Meanwhile, Vicente, who had in the interim contracted another marriage, and Rebecca
commenced several criminal complaints against each other. Specifically, Vicente filed adultery
and perjury complaints against Rebecca. Rebecca, on the other hand, charged Vicente with
bigamy and concubinage.
Ruling of the RTC on the Motion to Dismiss
and Motion for Support Pendente Lite

On August 8, 2001, the RTC issued an Order18 denying Vicente's motion to dismiss Civil Case
No. 01-094 and granting Rebecca's application for support pendente lite, disposing as follows:

Wherefore, premises considered, the Motion to Dismiss filed by the respondent is


DENIED. Petitioner's Application in Support of the Motion for Support Pendente Lite is
hereby GRANTED. Respondent is hereby ordered to remit the amount of TWO
HUNDRED AND TWENTY THOUSAND PESOS (Php 220,000.00) a month to Petitioner
as support for the duration of the proceedings relative to the instant Petition.

SO ORDERED.19

The RTC declared, among other things, that the divorce judgment invoked by Vicente as bar to
the petition for declaration of absolute nullity of marriage is a matter of defense best taken up
during actual trial. As to the grant of support pendente lite, the trial court held that a mere
allegation of adultery against Rebecca does not operate to preclude her from receiving legal
support.

Following the denial20 of his motion for reconsideration of the above August 8, 2001 RTC order,
Vicente went to the CA on a petition for certiorari, with a prayer for the issuance of a temporary
restraining order (TRO) and/or writ of preliminary injunction. 21 His petition was docketed as CA-
G.R. SP No. 68187.

Grant of Writ of Preliminary Injunction by the CA

On January 9, 2002, the CA issued the desired TRO. 22 On April 30, 2002, the appellate court
granted, via a Resolution, the issuance of a writ of preliminary injunction, the decretal portion of
which reads:

IN VIEW OF ALL THE FOREGOING, pending final resolution of the petition at bar, let
the Writ of Preliminary Injunction be ISSUED in this case, enjoining the respondent court
from implementing the assailed Omnibus Order dated August 8, 2001 and the Order
dated November 20, 2001, and from conducting further proceedings in Civil Case No.
01-094, upon the posting of an injunction bond in the amount of P250,000.00.

SO ORDERED.23

Rebecca moved24 but was denied reconsideration of the aforementioned April 30, 2002
resolution. In the meantime, on May 20, 2002, the preliminary injunctive writ 25 was issued.
Rebecca also moved for reconsideration of this issuance, but the CA, by Resolution dated
September 2, 2002, denied her motion.

The adverted CA resolutions of April 30, 2002 and September 2, 2002 are presently being
assailed in Rebecca's petition for certiorari, docketed under G.R. No. 155635.

Ruling of the CA
Pending resolution of G.R. No. 155635, the CA, by a Decision dated March 25, 2004, effectively
dismissed Civil Case No. 01-094, and set aside incidental orders the RTC issued in relation to
the case. The fallo of the presently assailed CA Decision reads:

IN VIEW OF THE FOREGOING, the petition is GRANTED. The Omnibus Order dated
August 8, 2001 and the Order dated November 20, 2001 are REVERSED and SET
ASIDE and a new one entered DISMISSING Civil Case No. 01-094, for failure to state a
cause of action. No pronouncement as to costs.

SO ORDERED.26

To the CA, the RTC ought to have granted Vicente's motion to dismiss on the basis of the
following premises:

(1) As held in China Road and Bridge Corporation v. Court of Appeals, the hypothetical-
admission rule applies in determining whether a complaint or petition states a cause of
action.27 Applying said rule in the light of the essential elements of a cause of action, 28 Rebecca
had no cause of action against Vicente for declaration of nullity of marriage.

(2) Rebecca no longer had a legal right in this jurisdiction to have her marriage with Vicente
declared void, the union having previously been dissolved on February 22, 1996 by the foreign
divorce decree she personally secured as an American citizen. Pursuant to the second
paragraph of Article 26 of the Family Code, such divorce restored Vicente's capacity to contract
another marriage.

(3) Rebecca's contention about the nullity of a divorce, she being a Filipino citizen at the time
the foreign divorce decree was rendered, was dubious. Her allegation as to her alleged Filipino
citizenship was also doubtful as it was not shown that her father, at the time of her birth, was still
a Filipino citizen. The Certification of Birth of Rebecca issued by the Government of Guam also
did not indicate the nationality of her father.

(4) Rebecca was estopped from denying her American citizenship, having professed to have
that nationality status and having made representations to that effect during momentous events
of her life, such as: (a) during her marriage; (b) when she applied for divorce; and (c) when she
applied for and eventually secured an American passport on January 18, 1995, or a little over a
year before she initiated the first but later withdrawn petition for nullity of her marriage (Civil
Case No. 96-378) on March 14, 1996.

(5) Assuming that she had dual citizenship, being born of a purportedly Filipino father in Guam,
USA which follows the jus soli principle, Rebecca's representation and assertion about being an
American citizen when she secured her foreign divorce precluded her from denying her
citizenship and impugning the validity of the divorce.

Rebecca seasonably filed a motion for reconsideration of the above Decision, but this recourse
was denied in the equally assailed June 4, 2004 Resolution. 29 Hence, Rebecca's Petition for
Review on Certiorari under Rule 45, docketed under G.R. No. 163979.

The Issues
In G.R. No. 155635, Rebecca raises four (4) assignments of errors as grounds for the
allowance of her petition, all of which converged on the proposition that the CA erred in
enjoining the implementation of the RTC's orders which would have entitled her to support
pending final resolution of Civil Case No. 01-094.

In G.R. No. 163979, Rebecca urges the reversal of the assailed CA decision submitting as
follows:

THE COURT OF APPEALS GRAVELY ERRED IN NOT MENTIONING AND NOT


TAKING INTO CONSIDERATION IN ITS APPRECIATION OF THE FACTS THE FACT
OF PETITIONER'S FILIPINO CITIZENSHIP AS CATEGORICALLY STATED AND
ALLEGED IN HER PETITION BEFORE THE COURT A QUO.

II

THE COURT OF APPEALS GRAVELY ERRED IN RELYING ONLY ON ANNEXES TO


THE PETITION IN RESOLVING THE MATTERS BROUGHT BEFORE IT.

III

THE COURT OF APPEALS GRAVELY ERRED IN FAILING TO CONSIDER THAT


RESPONDENT IS ESTOPPED FROM CLAIMING THAT HIS MARRIAGE TO
PETITIONER HAD ALREADY BEEN DISSOLVED BY VIRTUE OF HIS SUBSEQUENT
AND CONCURRENT ACTS.

IV

THE COURT OF APPEALS GRAVELY ERRED IN RULING THAT THERE WAS ABUSE
OF DISCRETION ON THE PART OF THE TRIAL COURT, MUCH LESS A GRAVE
ABUSE.30

We shall first address the petition in G.R. No. 163979, its outcome being determinative of the
success or failure of the petition in G.R. No. 155635.

Three legal premises need to be underscored at the outset. First, a divorce obtained abroad by
an alien married to a Philippine national may be recognized in the Philippines, provided the
decree of divorce is valid according to the national law of the foreigner. 31 Second, the reckoning
point is not the citizenship of the divorcing parties at birth or at the time of marriage, but their
citizenship at the time a valid divorce is obtained abroad. And third, an absolute divorce secured
by a Filipino married to another Filipino is contrary to our concept of public policy and morality
and shall not be recognized in this jurisdiction. 32

Given the foregoing perspective, the determinative issue tendered in G.R. No. 155635, i.e., the
propriety of the granting of the motion to dismiss by the appellate court, resolves itself into the
questions of: first, whether petitioner Rebecca was a Filipino citizen at the time the divorce
judgment was rendered in the Dominican Republic on February 22, 1996; and second, whether
the judgment of divorce is valid and, if so, what are its consequent legal effects?
The Court's Ruling

The petition is bereft of merit.

Rebecca an American Citizen in the Purview of This Case

There can be no serious dispute that Rebecca, at the time she applied for and obtained her
divorce from Vicente, was an American citizen and remains to be one, absent proof of an
effective repudiation of such citizenship. The following are compelling circumstances indicative
of her American citizenship: (1) she was born in Agaña, Guam, USA; (2) the principle of jus
soli is followed in this American territory granting American citizenship to those who are born
there; and (3) she was, and may still be, a holder of an American passport.33

And as aptly found by the CA, Rebecca had consistently professed, asserted, and represented
herself as an American citizen, particularly: (1) during her marriage as shown in the marriage
certificate; (2) in the birth certificate of Alix; and (3) when she secured the divorce from the
Dominican Republic. Mention may be made of the Affidavit of Acknowledgment 34 in which she
stated being an American citizen.

It is true that Rebecca had been issued by the Bureau of Immigration (Bureau) of Identification
(ID) Certificate No. RC 9778 and a Philippine Passport. On its face, ID Certificate No. RC 9778
would tend to show that she has indeed been recognized as a Filipino citizen. It cannot be over-
emphasized, however, that such recognition was given only on June 8, 2000 upon the
affirmation by the Secretary of Justice of Rebecca's recognition pursuant to the Order of
Recognition issued by Bureau Associate Commissioner Edgar L. Mendoza.

For clarity, we reproduce in full the contents of ID Certificate No. RC 9778:

To Whom It May Concern:

This is to certify that *MARIA REBECCA MAKAPUGAY BAYOT* whose photograph and
thumbprints are affixed hereto and partially covered by the seal of this Office, and whose
other particulars are as follows:

Place of Birth: Guam, USA Date of Birth: March 5, 1953

Sex: female Civil Status: married Color of Hair:


brown

Color of Eyes: brown Distinguishing marks on face: none

was - r e c o g n i z e d - as a citizen of the Philippines as per pursuant to Article IV,


Section 1, Paragraph 3 of the 1935 Constitution per order of Recognition JBL 95-213
signed by Associate Commissioner Jose B. Lopez dated October 6, 1995, and duly
affirmed by Secretary of Justice Artemio G. Tuquero in his 1st Indorsement dated June 8,
2000.

Issued for identification purposes only. NOT VALID for travel purposes.
Given under my hand and seal this 11th day of October, 1995

(SGD) EDGAR L. MENDOZA


ASSO. COMMISSIONER

Official Receipt No. 5939988


issued at Manila
dated Oct. 10, 1995 for P 2,000

From the text of ID Certificate No. RC 9778, the following material facts and dates may be
deduced: (1) Bureau Associate Commissioner Jose B. Lopez issued the Order of Recognition
on October 6, 1995; (2) the 1st Indorsement of Secretary of Justice Artemio G. Tuquero
affirming Rebecca's recognition as a Filipino citizen was issued on June 8, 2000 or almost five
years from the date of the order of recognition; and (3) ID Certificate No. RC 9778 was
purportedly issued on October 11, 1995 after the payment of the PhP 2,000 fee on October 10,
1995 per OR No. 5939988.

What begs the question is, however, how the above certificate could have been issued by the
Bureau on October 11, 1995 when the Secretary of Justice issued the required affirmation only
on June 8, 2000. No explanation was given for this patent aberration. There seems to be no
error with the date of the issuance of the 1 st Indorsement by Secretary of Justice Tuquero as
this Court takes judicial notice that he was the Secretary of Justice from February 16, 2000 to
January 22, 2001. There is, thus, a strong valid reason to conclude that the certificate in
question must be spurious.

Under extant immigration rules, applications for recognition of Filipino citizenship require the
affirmation by the DOJ of the Order of Recognition issued by the Bureau. Under Executive
Order No. 292, also known as the 1987 Administrative Code, specifically in its Title III, Chapter
1, Sec. 3(6), it is the DOJ which is tasked to "provide immigration and naturalization regulatory
services and implement the laws governing citizenship and the admission and stay of
aliens." Thus, the confirmation by the DOJ of any Order of Recognition for Filipino citizenship
issued by the Bureau is required.

Pertinently, Bureau Law Instruction No. RBR-99-00235 on Recognition as a Filipino Citizen


clearly provides:

The Bureau [of Immigration] through its Records Section shall automatically furnish the
Department of Justice an official copy of its Order of Recognition within 72 days from its
date of approval by the way of indorsement for confirmation of the Order by the
Secretary of Justice pursuant to Executive Order No. 292. No Identification Certificate
shall be issued before the date of confirmation by the Secretary of Justice and any
Identification Certificate issued by the Bureau pursuant to an Order of Recognition shall
prominently indicate thereon the date of confirmation by the Secretary of Justice.
(Emphasis ours.)

Not lost on the Court is the acquisition by Rebecca of her Philippine passport only on June 13,
2000, or five days after then Secretary of Justice Tuquero issued the 1 st Indorsement confirming
the order of recognition. It may be too much to attribute to coincidence this unusual sequence of
close events which, to us, clearly suggests that prior to said affirmation or confirmation,
Rebecca was not yet recognized as a Filipino citizen. The same sequence would also imply that
ID Certificate No. RC 9778 could not have been issued in 1995, as Bureau Law Instruction No.
RBR-99-002 mandates that no identification certificate shall be issued before the date of
confirmation by the Secretary of Justice. Logically, therefore, the affirmation or confirmation of
Rebecca's recognition as a Filipino citizen through the 1st Indorsement issued only on June 8,
2000 by Secretary of Justice Tuquero corresponds to the eventual issuance of Rebecca's
passport a few days later, or on June 13, 2000 to be exact.

When Divorce Was Granted Rebecca, She Was not a


Filipino Citizen and Was not Yet Recognized as One

The Court can assume hypothetically that Rebecca is now a Filipino citizen. But from the
foregoing disquisition, it is indubitable that Rebecca did not have that status of, or at least was
not yet recognized as, a Filipino citizen when she secured the February 22, 1996 judgment of
divorce from the Dominican Republic.

The Court notes and at this juncture wishes to point out that Rebecca voluntarily withdrew her
original petition for declaration of nullity (Civil Case No. 96-378 of the Makati City RTC)
obviously because she could not show proof of her alleged Filipino citizenship then. In fact, a
perusal of that petition shows that, while bearing the date January 26, 1996, it was only filed
with the RTC on March 14, 1996 or less than a month after Rebecca secured, on February 22,
1996, the foreign divorce decree in question. Consequently, there was no mention about said
divorce in the petition. Significantly, the only documents appended as annexes to said original
petition were: the Vicente-Rebecca Marriage Contract (Annex "A") and Birth Certificate of Alix
(Annex "B"). If indeed ID Certificate No. RC 9778 from the Bureau was truly issued on October
11, 1995, is it not but logical to expect that this piece of document be appended to form part of
the petition, the question of her citizenship being crucial to her case?

As may be noted, the petition for declaration of absolute nullity of marriage under Civil Case No.
01-094, like the withdrawn first petition, also did not have the ID Certificate from the Bureau as
attachment. What were attached consisted of the following material documents: Marriage
Contract (Annex "A") and Divorce Decree. It was only through her Opposition (To Respondent's
Motion to Dismiss dated 31 May 2001)36 did Rebecca attach as Annex "C" ID Certificate No. RC
9778.

At any rate, the CA was correct in holding that the RTC had sufficient basis to dismiss the
petition for declaration of absolute nullity of marriage as said petition, taken together with
Vicente's motion to dismiss and Rebecca's opposition to motion, with their respective
attachments, clearly made out a case of lack of cause of action, which we will expound later.

Validity of Divorce Decree

Going to the second core issue, we find Civil Decree Nos. 362/96 and 406/97 valid.

First, at the time of the divorce, as above elucidated, Rebecca was still to be recognized,
assuming for argument that she was in fact later recognized, as a Filipino citizen, but
represented herself in public documents as an American citizen. At the very least, she chose,
before, during, and shortly after her divorce, her American citizenship to govern her marital
relationship. Second, she secured personally said divorce as an American citizen, as is evident
in the text of the Civil Decrees, which pertinently declared:
IN THIS ACTION FOR DIVORCE in which the parties expressly submit to the jurisdiction
of this court, by reason of the existing incompatibility of temperaments x x x. The parties
MARIA REBECCA M. BAYOT, of United States nationality, 42 years of age, married,
domiciled and residing at 502 Acacia Ave., Ayala Alabang, Muntin Lupa, Philippines, x x
x, who personally appeared before this court, accompanied by DR. JUAN ESTEBAN
OLIVERO, attorney, x x x and VICENTE MADRIGAL BAYOT, of Philippine nationality, of
43 years of age, married and domiciled and residing at 502 Acacia Ave., Ayala Alabang,
Muntin Lupa, Filipino, appeared before this court represented by DR. ALEJANDRO
TORRENS, attorney, x x x, revalidated by special power of attorney given the 19 th of
February of 1996, signed before the Notary Public Enrico L. Espanol of the City of
Manila, duly legalized and authorizing him to subscribe all the acts concerning this
case.37 (Emphasis ours.)

Third, being an American citizen, Rebecca was bound by the national laws of the United States
of America, a country which allows divorce. Fourth, the property relations of Vicente and
Rebecca were properly adjudicated through their Agreement 38 executed on December 14, 1996
after Civil Decree No. 362/96 was rendered on February 22, 1996, and duly affirmed by Civil
Decree No. 406/97 issued on March 4, 1997. Veritably, the foreign divorce secured by Rebecca
was valid.

To be sure, the Court has taken stock of the holding in Garcia v. Recio that a foreign divorce
can be recognized here, provided the divorce decree is proven as a fact and as valid under the
national law of the alien spouse. 39 Be this as it may, the fact that Rebecca was clearly an
American citizen when she secured the divorce and that divorce is recognized and allowed in
any of the States of the Union,40 the presentation of a copy of foreign divorce decree duly
authenticated by the foreign court issuing said decree is, as here, sufficient.

It bears to stress that the existence of the divorce decree has not been denied, but in fact
admitted by both parties. And neither did they impeach the jurisdiction of the divorce court nor
challenge the validity of its proceedings on the ground of collusion, fraud, or clear mistake of
fact or law, albeit both appeared to have the opportunity to do so. The same holds true with
respect to the decree of partition of their conjugal property. As this Court explained in Roehr v.
Rodriguez:

Before our courts can give the effect of res judicata to a foreign judgment [of divorce] x x
x, it must be shown that the parties opposed to the judgment had been given ample
opportunity to do so on grounds allowed under Rule 39, Section 50 of the Rules of Court
(now Rule 39, Section 48, 1997 Rules of Civil Procedure), to wit:

SEC. 50. Effect of foreign judgments.--The effect of a judgment of a tribunal of a


foreign country, having jurisdiction to pronounce the judgment is as follows:

(a) In case of a judgment upon a specific thing, the judgment is conclusive upon
the title to the thing;

(b) In case of a judgment against a person, the judgment is presumptive


evidence of a right as between the parties and their successors in interest by a
subsequent title; but the judgment may be repelled by evidence of a want of
jurisdiction, want of notice to the party, collusion, fraud, or clear mistake of law or
fact.
It is essential that there should be an opportunity to challenge the foreign judgment, in
order for the court in this jurisdiction to properly determine its efficacy. In this jurisdiction,
our Rules of Court clearly provide that with respect to actions in personam, as
distinguished from actions in rem, a foreign judgment |merely constitutes prima
facie evidence of the justness of the claim of a party and, as such, is subject to proof to
the contrary.41

As the records show, Rebecca, assisted by counsel, personally secured the foreign divorce
while Vicente was duly represented by his counsel, a certain Dr. Alejandro Torrens, in said
proceedings. As things stand, the foreign divorce decrees rendered and issued by the
Dominican Republic court are valid and, consequently, bind both Rebecca and Vicente.

Finally, the fact that Rebecca may have been duly recognized as a Filipino citizen by force of
the June 8, 2000 affirmation by Secretary of Justice Tuquero of the October 6, 1995 Bureau
Order of Recognition will not, standing alone, work to nullify or invalidate the foreign divorce
secured by Rebecca as an American citizen on February 22, 1996. For as we stressed at the
outset, in determining whether or not a divorce secured abroad would come within the pale of
the country's policy against absolute divorce, the reckoning point is the citizenship of the parties
at the time a valid divorce is obtained. 42

Legal Effects of the Valid Divorce

Given the validity and efficacy of divorce secured by Rebecca, the same shall be given a res
judicataeffect in this jurisdiction. As an obvious result of the divorce decree obtained, the
marital vinculumbetween Rebecca and Vicente is considered severed; they are both freed from
the bond of matrimony. In plain language, Vicente and Rebecca are no longer husband and wife
to each other. As the divorce court formally pronounced: "[T]hat the marriage between MARIA
REBECCA M. BAYOT and VICENTE MADRIGAL BAYOT is hereby dissolved x x x leaving
them free to remarry after completing the legal requirements."43

Consequent to the dissolution of the marriage, Vicente could no longer be subject to a


husband's obligation under the Civil Code. He cannot, for instance, be obliged to live with,
observe respect and fidelity, and render support to Rebecca. 44

The divorce decree in question also brings into play the second paragraph of Art. 26 of the
Family Code, providing as follows:

Art. 26. x x x x

Where a marriage between a Filipino citizen and a foreigner is validly celebrated and a
divorce is thereafter validly obtained abroad by the alien spouse capacitating him or her
to remarry, the Filipino spouse shall likewise have capacity to remarry under Philippine
law. (As amended by E.O. 227)

In Republic v. Orbecido III, we spelled out the twin elements for the applicability of the second
paragraph of Art. 26, thus:

x x x [W]e state the twin elements for the application of Paragraph 2 of Article 26 as
follows:
1. There is a valid marriage that has been celebrated between a Filipino citizen and a
foreigner; and

2. A valid divorce is obtained abroad by the alien spouse capacitating him or her to
remarry.

The reckoning point is not the citizenship of the parties at the time of the celebration of
the marriage, but their citizenship at the time a valid divorce is obtained abroad by the
alien spouse capacitating the latter to remarry. 45

Both elements obtain in the instant case. We need not belabor further the fact of marriage of
Vicente and Rebecca, their citizenship when they wed, and their professed citizenship during
the valid divorce proceedings.

Not to be overlooked of course is the fact that Civil Decree No. 406/97 and the Agreement
executed on December 14, 1996 bind both Rebecca and Vicente as regards their property
relations. The Agreement provided that the ex-couple's conjugal property consisted only their
family home, thus:

9. That the parties stipulate that the conjugal property which they acquired during
their marriage consists only of the real property and all the improvements and
personal properties therein contained at 502 Acacia Avenue, Ayala Alabang, Muntinlupa,
covered by TCT No. 168301 dated Feb. 7, 1990 issued by the Register of Deeds of
Makati, Metro Manila registered in the name of Vicente M. Bayot, married to Rebecca M.
Bayot, x x x.46 (Emphasis ours.)

This property settlement embodied in the Agreement was affirmed by the divorce court which,
per its second divorce decree, Civil Decree No. 406/97 dated March 4, 1997, ordered that,
"THIRD: That the agreement entered into between the parties dated 14 th day of December 1996
in Makati City, Philippines shall survive in this Judgment of divorce by reference but not merged
and that the parties are hereby ordered and directed to comply with each and every
provision of said agreement."47

Rebecca has not repudiated the property settlement contained in the Agreement. She is thus
estopped by her representation before the divorce court from asserting that her and Vicente's
conjugal property was not limited to their family home in Ayala Alabang. 48

No Cause of Action in the Petition for Nullity of Marriage

Upon the foregoing disquisitions, it is abundantly clear to the Court that Rebecca lacks, under
the premises, cause of action. Philippine Bank of Communications v. Trazo explains the
concept and elements of a cause of action, thus:

A cause of action is an act or omission of one party in violation of the legal right of the
other. A motion to dismiss based on lack of cause of action hypothetically admits the
truth of the allegations in the complaint. The allegations in a complaint are sufficient to
constitute a cause of action against the defendants if, hypothetically admitting the facts
alleged, the court can render a valid judgment upon the same in accordance with the
prayer therein. A cause of action exists if the followingelements are present, namely: (1)
a right in favor of the plaintiff by whatever means and under whatever law it arises or is
created; (2) an obligation on the part of the named defendant to respect or not to violate
such right; and (3) an act or omission on the part of such defendant violative of the right
of the plaintiff or constituting a breach of the obligation of the defendant to the plaintiff for
which the latter may maintain an action for recovery of damages. 49

One thing is clear from a perusal of Rebecca's underlying petition before the RTC, Vicente's
motion to dismiss and Rebecca's opposition thereof, with the documentary evidence attached
therein: The petitioner lacks a cause of action for declaration of nullity of marriage, a suit which
presupposes the existence of a marriage.

To sustain a motion to dismiss for lack of cause of action, the movant must show that the claim
for relief does not exist rather than that a claim has been defectively stated or is ambiguous,
indefinite, or uncertain.50 With the valid foreign divorce secured by Rebecca, there is no more
marital tie binding her to Vicente. There is in fine no more marriage to be dissolved or nullified.

The Court to be sure does not lose sight of the legal obligation of Vicente and Rebecca to
support the needs of their daughter, Alix. The records do not clearly show how he had
discharged his duty, albeit Rebecca alleged that the support given had been insufficient. At any
rate, we do note that Alix, having been born on November 27, 1982, reached the majority age
on November 27, 2000, or four months before her mother initiated her petition for declaration of
nullity. She would now be 26 years old. Hence, the issue of back support, which allegedly had
been partly shouldered by Rebecca, is best litigated in a separate civil action for reimbursement.
In this way, the actual figure for the support of Alix can be proved as well as the earning
capacity of both Vicente and Rebecca. The trial court can thus determine what Vicente owes, if
any, considering that support includes provisions until the child concerned shall have finished
her education.

Upon the foregoing considerations, the Court no longer need to delve into the issue tendered in
G.R. No. 155635, that is, Rebecca's right to support pendente lite. As it were, her entitlement to
that kind of support hinges on the tenability of her petition under Civil Case No. 01-094 for
declaration of nullity of marriage. The dismissal of Civil Case No. 01-094 by the CA veritably
removed any legal anchorage for, and effectively mooted, the claim for support pendente lite.

WHEREFORE, the petition for certiorari in G.R. No. 155635 is hereby DISMISSED on the
ground of mootness, while the petition for review in G.R. No. 163979 is hereby DENIED for lack
of merit. Accordingly, the March 25, 2004 Decision and June 4, 2004 Resolution of the CA in
CA-G.R. SP No. 68187 are hereby AFFIRMED. Costs against petitioner.

SO ORDERED.

PRESBITERO J. VELASCO, JR.


Associate Justice

WE CONCUR:
LEONARDO A. QUISUMBING
Associate Justice
Chairperson

CONCHITA CARPIO MORALES DANTE O. TINGA


Associate Justice Associate Justice

ARTURO D. BRION
Associate Justice

ATTESTATION

I attest that the conclusions in the above Decision had been reached in consultation before the
case was assigned to the writer of the opinion of the Court's Division.

LEONARDO A. QUISUMBING
Associate Justice
Chairperson

CERTIFICATION

Pursuant to Section 13, Article VIII of the Constitution, and the Division Chairperson's
Attestation, it is hereby certified that the conclusions in the above Decision were reached in
consultation before the case was assigned to the writer of the opinion of the Court's Division.

REYNATO S. PUNO
Chief Justice

Footnotes

1
Rollo (G.R. No. 155635), pp. 3-34.
2
Id. at 36-38. Penned by Associate, now Presiding, Justice Conrado M. Vasquez, Jr.
and concurred in by Associate Justices Andres B. Reyes, Jr. and Mario L. Guariña III.
3
Id. at 40-41.
4
Rollo (G.R. No. 163979), pp. 10-43.
5
Id. at 575-583.
6
Id. at 145.
7
See Certification of Birth from the Government of Guam issued on June 1, 2000; rollo
(G.R. No. 155635), p. 213.
8
Rollo (G.R. No. 163979), pp. 146-150.
9
Id. at 214-217.
10
Rollo (G.R. No. 155635), pp. 151-158.
11
Id. at 154.
12
Rollo (G.R. No. 163979), pp. 206-212.
13
Id. at 305-306. Per a motion to withdraw dated November 8, 1996.
14
Id. at 213. Per Order of Judge Josefina Guevara Salonga dated November 14, 1996.
15
Id. at 236-237.
16
Id. at 126-144.
17
Id. at 156-204.
18
Id. at 123-124. Penned by Presiding Judge Alberto L. Lerma.
19
Id. at 338.
20
Id. at 125. Per Order dated November 20, 2001.
21
Rollo (G.R. No. 155635), pp. 512-590.
22
Id. at 592-593.
23
Id. at 38.
24
Id. at 852-869.
25
Id. at 850-851.
26
Supra note 5, at 583.
27
G.R. No. 137898, December 15, 2000, 348 SCRA 401, 409.
28
Enumerated in San Lorenzo Village Association, Inc. v. Court of Appeals, G.R. No.
116825 March 26, 1998, 288 SCRA 115, 125: (1) the legal right of the plaintiff, (2) the
correlative obligation of the defendant, and (3) the act or omission of the defendant in
violation of said legal right.
29
Rollo (G.R. No. 163979), p. 597.
30
Id. at 22-23.
31
Garcia v. Recio, G.R. No. 138322, October 2, 2001, 366 SCRA 437, 447.
32
Llorente v. Court of Appeals, G.R. No. 124371, November 23, 2000, 345 SCRA 592,
600.
33
Rollo (G.R. No. 155635), pp. 388-389, issued on January 18, 1995 with expiration
date on January 17, 2005.
34
Supra note 15.
35
Adopted on April 15, 1999.
36
Rollo (G.R. No. 163979), pp. 268-292.
37
Id. at 147, 214-215.
38
Supra note 10.
39
Supra note 31.
40
Van Dorn v. Romillo, Jr., No. L-68470, October 8, 1985, 139 SCRA 139, 143.
41
G.R. No. 142820, June 20, 2003, 404 SCRA 495, 502-503.
42
Id. at 501-502.
43
Rollo (G.R. No. 163979), pp. 148, 216.
44
Van Dorn, supra note 40, at 144.
45
G.R. No. 154380, October 5, 2005, 472 SCRA 114, 122.
46
Rollo (G.R. No. 155635), p. 154.
47
Rollo (G.R. No. 163979), p. 215.
48
Van Dorn, supra note 44.
49
G.R. No. 165500, August 30, 2006, 500 SCRA 242, 251-252; citations omitted.
50
Azur v. Provincial Board, No. L-22333, February 27, 1969, 27 SCRA 50, 57-58.
V. Marriage

Art. II

Section 12. The State recognizes the sanctity of family life and shall protect and strengthen
the family as a basic autonomous social institution. It shall equally protect the life of the
mother and the life of the unborn from conception. The natural and primary right and duty of
parents in the rearing of the youth for civic efficiency and the development of moral
character shall receive the support of the Government.

Art. XV

Section 2. Marriage, as an inviolable social institution, is the foundation of the family and shall
be protected by the State.

GOITIA VS CAMPOS RUEDA

Digest:

Goitia vs. Campos Rueda

Facts:

In January 1915, Eloisa Goitia and Jose Campos Rueda were legally married in Manila
and thereafter lived together in Calle San Marcelino for about a month. Petitioner went
back to her parents because of the following reasons:
1) Defendant demanded her to perform unchaste and lascivious acts on her genitals
2) Defendant made other lewd demands
3) Defendant maltreated petitioner by word and by deed on the ground that the
latter refused to do any of defendant‘s demands other than legal and valid cohabitation
Petitioner filed an action against her husband for support outside their conjugal domicile.

Issue:
WON defendant may be compelled to render support to his wife

Held:
Yes. Campos Rueda was held liable to support his wife.

Ratio Decidendi:
Upon termination of the marriage ceremony, a conjugal partnership is formed between
the spouses. The reciprocal rights arising from this partnership is governed by the law
and controlled by the state or government, upon principles of public policy for the
benefit of society as well as the parties.
Because marriage is a social institution vested with public interest, the contracting
parties may not terminate it at any shorter period by virtue of any contract they make.
Campos Rueda cannot, by his wrongful acts, be relieved of the duty to support Goitia as
imposed by law. Where a husband, through is wrongful, illegal and unbearable conduct
drives his wife away from the domicile fixed by him, he cannot take advantage of her
departure to sever the law concerning marital relations and repudiate his duties
thereunder.

Full Case:

Republic of the Philippines


SUPREME COURT
Manila

EN BANC

G.R. No. 11263 November 2, 1916

ELOISA GOITIA DE LA CAMARA, plaintiff-appellant,


vs.
JOSE CAMPOS RUEDA, defendant-appellee.

Eduardo Gutierrez Repide and Felix Socias for appellant.


Sanz, Opisso and Luzuriaga for appellee.

TRENT, J.:

This is an action by the wife against her husband for support outside of the conjugal domicile.
From a judgment sustaining the defendant's demurrer upon the ground that the facts alleged in
the complaint do not state a cause of action, followed by an order dismissing the case after the
plaintiff declined to amend, the latter appealed.

It was urged in the first instance, and the court so held, that the defendant cannot be compelled
to support the plaintiff, except in his own house, unless it be by virtue of a judicial decree
granting her a divorce or separation from the defendant.

The parties were legally married in the city of Manila on January 7, 1915, and immediately
thereafter established their residence at 115 Calle San Marcelino, where they lived together for
about a month, when the plaintiff returned to the home of her parents. The pertinent allegations
of the complaint are as follows:

That the defendant, one month after he had contracted marriage with the plaintiff,
demanded of her that she perform unchaste and lascivious acts on his genital organs;
that the plaintiff spurned the obscene demands of the defendant and refused to perform
any act other than legal and valid cohabitation; that the defendant, since that date had
continually on other successive dates, made similar lewd and indecorous demands on
his wife, the plaintiff, who always spurned them, which just refusals of the plaintiff
exasperated the defendant and induce him to maltreat her by word and deed and inflict
injuries upon her lips, her face and different parts of her body; and that, as the plaintiff
was unable by any means to induce the defendant to desist from his repugnant desires
and cease from maltreating her, she was obliged to leave the conjugal abode and take
refuge in the home of her parents.

Marriage in this jurisdiction is a contract entered into in the manner and with the solemnities
established by General Orders No. 68, in so far as its civil effects are concerned requiring the
consent of the parties. (Garcia vs. Montague, 12 Phil. Rep., 480, citing article 1261 of Civil
Code.) Upon the termination of the marriage ceremony, a conjugal partnership is formed
between the parties. (Sy Joc Lieng vs. Encarnacion, 16 Phil. Rep., 137.) To this extent a
marriage partakes of the nature of an ordinary contract. But it is something more than a mere
contract. It is a new relation, the rights, duties, and obligations of which rest not upon the
agreement of the parties but upon the general law which defines and prescribes those rights,
duties, and obligations .Marriage is an institution, in the maintenance of which in its purity the
public is deeply interested. It is a relation for life and the parties cannot terminate it at any
shorter period by virtue of any contract they may make .The reciprocal rights arising from this
relation, so long as it continues, are such as the law determines from time to time, and none
other. When the legal existence of the parties is merged into one by marriage, the new relation
is regulated and controlled by the state or government upon principles of public policy for the
benefit of society as well as the parties. And when the object of a marriage is defeated by
rendering its continuance intolerable to one of the parties and productive of no possible good to
the community, relief in some way should be obtainable. With these principles to guide us, we
will inquire into the status of the law touching and governing the question under consideration.

Articles 42 to 107 of the Civil Code are not in force in the Philippine Islands (Benedicto vs. De la
Rama, 3 Phil .Rep., 34). Articles 44 to 78 of the Law of Civil Marriage of 1870, in force in the
Peninsula, were extended to the Philippine Islands by royal decree on April 13, 1883 (Ebreo vs.
Sichon, 4 Phil. Rep., 705). Articles 44, 45, and 48 of this law read:

ART. 44. The spouses are obliged to be faithful to each other and to mutually assist
each other.

ART. 45. The husband must live with and protect his wife. (The second paragraph deals
with the management of the wife's property.)

ART. 48. The wife must obey her husband, live with him, and follow him when he
charges his domicile or residence.

Notwithstanding the provisions of the foregoing paragraph, the court may for just cause
relieve her from this duty when the husband removes his residence to a foreign country.

And articles 143 and 149 of the Civil Code are as follows:

ART. 143. The following are obliged to support each other reciprocally to the whole
extent specified in the preceding article.
1. The consorts.

xxxxxxxxx

ART. (149) 49. The person obliged to give support may, at his option, satisfy it, either by
paying the pension that may be fixed or by receiving and maintaining in his own home
the person having the right to the same.

Article 152 of the Civil Code gives the instances when the obligation to give support shall cease.
The failure of the wife to live with her husband is not one of them.

The above quoted provisions of the Law of Civil Marriage and the Civil Code fix the duties and
obligations of the spouses. The spouses must be faithful to, assist, and support each other. The
husband must live with and protect his wife. The wife must obey and live with her husband and
follow him when he changes his domicile or residence, except when he removes to a foreign
country. But the husband who is obliged to support his wife may, at his option, do so by paying
her a fixed pension or by receiving and maintaining her in his own home. May the husband, on
account of his conduct toward his wife, lose this option and be compelled to pay the pension? Is
the rule established by article 149 of the Civil Code absolute? The supreme court of Spain in its
decision of December 5, 1903, held:.

That in accordance with the ruling of the supreme court of Spain in its decisions dated
May 11, 1897, November 25, 1899, and July 5, 1901, the option which article 149 grants
the person, obliged to furnish subsistence, between paying the pension fixed or
receiving and keeping in his own house the party who is entitled to the same, is not so
absolute as to prevent cases being considered wherein, either because this right would
be opposed to the exercise of a preferential right or because of the existence of some
justifiable cause morally opposed to the removal of the party enjoying the maintenance,
the right of selection must be understood as being thereby restricted.

Whereas the only question discussed in the case which gave rise to this appeal was
whether there was any reason to prevent the exercise of the option granted by article
149 of the Civil Code to the person obliged to furnish subsistence, to receive and
maintain in his own house the one who is entitled to receive it; and inasmuch as nothing
has been alleged or discussed with regard to the parental authority of Pedro Alcantara
Calvo, which he ha not exercised, and it having been set forth that the natural father
simply claims his child for the purpose of thus better attending to her maintenance, no
action having been taken by him toward providing the support until, owing to such
negligence, the mother was obliged to demand it; it is seen that these circumstances,
together with the fact of the marriage of Pedro Alcantara, and that it would be difficult for
the mother to maintain relations with her daughter, all constitute an impediment of such
a nature as to prevent the exercise of the option in the present case, without prejudice to
such decision as may be deemed proper with regard to the other questions previously
cited in respect to which no opinion should be expressed at this time.

The above was quoted with approval in United States and De Jesus vs. Alvir (9 Phil. Rep., 576),
wherein the court held that the rule laid down in article 149 of the Civil Code "is not absolute."
but it is insisted that there existed a preexisting or preferential right in each of these cases which
was opposed to the removal of the one entitled to support. It is true that in the first the person
claiming the option was the natural father of the child and had married a woman other than the
child's mother, and in the second the right to support had already been established by a final
judgment in a criminal case. Notwithstanding these facts the two cases clearly established the
proposition that the option given by article 149 of the Civil Code may not be exercised in any
and all cases.

Counsel for the defendant cite, in support of their contention, the decision of the supreme court
of Spain, dated November 3, 1905. In this case Don Berno Comas, as a result of certain
business reverses and in order no to prejudice his wife, conferred upon her powers to
administer and dispose of her property. When she left him he gave her all the muniments of title,
mortgage credits, notes, P10,000 in accounts receivable, and the key to the safe in which he
kept a large amount of jewels, thus depriving himself of all his possessions and being reduced
in consequence to want. Subsequently he instituted this civil action against his wife, who was
then living in opulence, for support and the revocation of the powers heretofore granted in
reference to the administration and disposal of her property. In her answer the wife claimed that
the plaintiff (her husband) was not legally in a situation to claim support and that the powers
voluntarily conferred and accepted by her were bilateral and could not be canceled by the
plaintiff. From a judgment in favor of the plaintiff the defendant wife appealed to theAudencia
Territorial wherein, after due trial, judgment was rendered in her favor dismissing the action
upon the merits. The plaintiff appealed to the supreme court and that high tribunal, in affirming
the judgment of theAudencia Territorial, said:

Considering that article 143, No. 1, of the Civil Code, providing that the spouses are
mutually obliged to provide each other with support, cannot but be subordinate to the
other provisions of said Code which regulates the family organization and the duties of
spouses not legally separated, among which duties are those of their living together and
mutually helping each other, as provided in article 56 of the aforementioned code; and
taking this for granted, the obligation of the spouse who has property to furnish support
to the one who has no property and is in need of it for subsistence, is to be understood
as limited to the case where, in accordance with law, their separation has been decreed,
either temporarily or finally and this case, with respect to the husband, cannot occur until
a judgment of divorce is rendered, since, until then, if he is culpable, he is not deprived
of the management of his wife's property and of the product of the other property
belonging to the conjugal partnership; and

Considering that, should the doctrine maintained in the appeal prevail, it would allow
married persons to disregard the marriage bond and separate from each other of their
own free will, thus establishing, contrary to the legal provision contained in said article 56
of the Civil Code, a legal status entirely incompatible with the nature and effects of
marriage in disregard of the duties inherent therein and disturbing the unity of the family,
in opposition to what the law, in conformity with good morals, has established; and.

Considering that, as the spouses D. Ramon Benso and Doña Adela Galindo are not
legally separated, it is their duty to live together and afford each other help and support;
and for this reason, it cannot be held that the former has need of support from his wife
so that he may live apart from her without the conjugal abode where it is his place to be,
nor of her conferring power upon him to dispose even of the fruits of her property in
order therewith to pay the matrimonial expenses and, consequently, those of his own
support without need of going to his wife; wherefore the judgment appealed from,
denying the petition of D. Ramon Benso for support, has not violated the articles of the
Civil Code and the doctrine invoked in the assignments of error 1 and 5 of the appeal.
From a careful reading of the case just cited and quoted from it appears quite clearly that the
spouses separated voluntarily in accordance with an agreement previously made. At least there
are strong indications to this effect, for the court says, "should the doctrine maintained in the
appeal prevail, it would allow married persons to disregard the marriage bond and separate
from each other of their own free will." If this be the true basis upon which the supreme court of
Spain rested its decision, then the doctrine therein enunciated would not be controlling in cases
where one of the spouses was compelled to leave the conjugal abode by the other or where the
husband voluntarily abandons such abode and the wife seeks to force him to furnish support.
That this is true appears from the decision of the same high tribunal, dated October 16, 1903. In
this case the wife brought an action for support against her husband who had willfully and
voluntarily abandoned the conjugal abode without any cause whatever. The supreme court,
reversing the judgment absolving the defendant upon the ground that no action for divorce, etc.,
had been instituted, said:

In the case at bar, it has been proven that it was Don Teodoro Exposito who left the
conjugal abode, although he claims, without however proving his contention, that the
person responsible for this situation was his wife, as she turned him out of the house.
From this state of affairs it results that it is the wife who is party abandoned, the husband
not having prosecuted any action to keep her in his company and he therefore finds
himself, as long as he consents to the situation, under the ineluctable obligation to
support his wife in fulfillment of the natural duty sanctioned in article 56 of the Code in
relation with paragraph 1 of article 143. In not so holding, the trial court, on the mistaken
ground that for the fulfillment of this duty the situation or relation of the spouses should
be regulated in the manner it indicates, has made the errors of law assigned in the first
three grounds alleged, because the nature of the duty of affording mutual support is
compatible and enforcible in all situations, so long as the needy spouse does not create
any illicit situation of the court above described.lawphil.net

If we are in error as to the doctrine enunciated by the supreme court of Spain in its decision of
November 3, 1905, and if the court did hold, as contended by counsel for the defendant in the
case under consideration, that neither spouse can be compelled to support the other outside of
the conjugal abode, unless it be by virtue of a final judgment granting the injured one a divorce
or separation from the other, still such doctrine or holding would not necessarily control in this
jurisdiction for the reason that the substantive law is not in every particular the same here as it is
in Spain. As we have already stated, articles 42 to 107 of the Civil Code in force in the
Peninsula are not in force in the Philippine Islands. The law governing the duties and obligations
of husband and wife in this country are articles 44 to 78 of the Law of Civil Marriage of 1870 .In
Spain the complaining spouse has, under article 105 of the Civil Code, various causes for
divorce, such as adultery on the part of the wife in every case and on the part of the husband
when public scandal or disgrace of the wife results therefrom; personal violence actually inflicted
or grave insults: violence exercised by the husband toward the wife in order to force her to
change her religion; the proposal of the husband to prostitute his wife; the attempts of the
husband or wife to corrupt their sons or to prostitute their daughters; the connivance in their
corruption or prostitution; and the condemnation of a spouse to perpetual chains or hard labor,
while in this jurisdiction the only ground for a divorce is adultery. (Benedicto vs. De la Rama, 3
Phil .Rep., 34, 45.) This positive and absolute doctrine was announced by this court in the case
just cited after an exhaustive examination of the entire subject. Although the case was appealed
to the Supreme Court of the United States and the judgment rendered by this court was there
reversed, the reversal did not affect in any way or weaken the doctrine in reference to adultery
being the only ground for a divorce. And since the decision was promulgated by this court in that
case in December, 1903, no change or modification of the rule has been announced. It is,
therefore, the well settled and accepted doctrine in this jurisdiction.

But it is argued that to grant support in an independent suit is equivalent to granting divorce or
separation, as it necessitates a determination of the question whether the wife has a good and
sufficient cause for living separate from her husband; and, consequently, if a court lacks power
to decree a divorce, as in the instant case, power to grant a separate maintenance must also be
lacking. The weakness of this argument lies in the assumption that the power to grant support in
a separate action is dependent upon a power to grant a divorce. That the one is not dependent
upon the other is apparent from the very nature of the marital obligations of the spouses. The
mere act of marriage creates an obligation on the part of the husband to support his wife. This
obligation is founded not so much on the express or implied terms of the contract of marriage as
on the natural and legal duty of the husband; an obligation, the enforcement of which is of such
vital concern to the state itself that the laws will not permit him to terminate it by his own
wrongful acts in driving his wife to seek protection in the parental home. A judgment for
separate maintenance is not due and payable either as damages or as a penalty; nor is it a debt
in the strict legal sense of the term, but rather a judgment calling for the performance of a duty
made specific by the mandate of the sovereign. This is done from necessity and with a view to
preserve the public peace and the purity of the wife; as where the husband makes so base
demands upon his wife and indulges in the habit of assaulting her. The pro tanto separation
resulting from a decree for separate support is not an impeachment of that public policy by
which marriage is regarded as so sacred and inviolable in its nature; it is merely a stronger
policy overruling a weaker one; and except in so far only as such separation is tolerated as a
means of preserving the public peace and morals may be considered, it does not in any respect
whatever impair the marriage contract or for any purpose place the wife in the situation of
a feme sole.

The foregoing are the grounds upon which our short opinion and order for judgment, heretofore
filed in this case, rest.

Torres, Johnson and Carson, JJ., concur.

Separate Opinions

MORELAND, J., concurring:

I based my vote in this case upon the ground that a husband cannot, by his own wrongful acts,
relieve himself from the duty to support his wife imposed by law; and where a husband, by
wrongful, illegal, and unbearable conduct, drives his wife from the domicile fixed by him, he
cannot take advantage of her departure to abrogate the law applicable to the marital relation
and repudiate his duties thereunder. In law and for all purposes within its purview, the wife still
remains an inmate of the conjugal domicile; for I regard it as a principle of law universally
recognized that where a person by his wrongful and illegal acts creates a condition which under
ordinary circumstances would produce the loss of rights or status pertaining to another, the law
will, whenever necessary to protect fully the rights or status of the person affected by such acts,
regard the condition by such acts created as not existing and will recur to and act upon the
original situation of the parties to determine their relative rights or the status of the person
adversely affected.
I do not believe, therefore, that the case is properly conceived by defendant, when the
consideration thereof proceeds solely on the theory that the wife is outside the domicile fixed by
the husband. Under the facts alleged in the complainant the wife is legally still within the
conjugal domicile.

WONG WOO YIU VS VIVO

Digest:

G.R. No. L-21076 March 31, 1965


WONG WOO YIU alias NG YAO, petitioner-appellee,
vs.
HON. MARTINIANO P. VIVO, ETC., ET AL., respondents-appellants.

Issue:
Whether or not WONG WOO YIU‘s marriage to PERFECTO BLAS is valid in the Philippines,
and her admission into the country is legal.

Facts:
In proceedings held before the Board of Special Inquiry sometime in June, 1961, Wong Woo Yiu
(petitioner) declared that

1) she came to the Philippines in 1961 for the first time to join her husband Perfecto Blas, a
Filipino Citizen, to whom she was married in Chingkang, China on January 15, 1929,
2) that they had several children all of whom are not in the Philippines;
3) that their marriage was celebrated by one Chua Tio, a village leader.

On June 28, 1961 the Board of Special Inquiry No. 3 rendered a decision finding petitioner to be
legally married to Perfecto Blas, thus declaring legal her admission into the country. This
decision was affirmed by the Board of Commissioners on July 12, 1961 of which petitioner was
duly informed in a letter sent on the same date by the Secretary of the Board.

However, on June 28, 1962, the same Board of Commissioners, but composed entirely of a new
set of members, rendered a new decision contrary to that of the Board of Special Inquiry No. 3
and ordering petitioner to be excluded from the country, after discrepancies were found in the
statements made by petitioner and her alleged husband during several investigations conducted
by the immigration authorities concerning the alleged marriage before a village leader in China
in 1929, thus concluding that the petitioner‘s claim that she is the lawful wife of Perfecto Blas
was without basis in evidence as it was "bereft of substantial proof of husband-wife
relationship."

Held:
The above revocation of decision cannot be disputed, it finding support in the record and
investigation. Indeed, not only is there no documentary evidence to support the alleged
marriage of petitioner to Perfecto Blas but the record is punctured with so many inconsistencies
which cannot but lead one to doubt their veracity concerning the said marriage in China on
1929.

Even if we assume, therefore, that the marriage of petitioner to Perfecto Blas before a village
leader is valid in China, the same is not one of those authorized in our country. (In order that a
marriage celebrated in the Philippines may be valid it must be solemnized either by a judge of
any court inferior to the Supreme Court, a justice of the peace, or a priest or minister of the
gospel of any denomination duly registered in the Philippine Library and Museum – Public Act
3412, Section 2) Since our law only recognizes a marriage celebrated before any of the officers
mentioned therein, and a village leader is not one of them, it is clear that petitioner's marriage
cannot be recognized in this jurisdiction.

Also, since no proof was presented relative to the law on marriage in China, we should apply
the general rule that in the absence of proof of the law of a foreign country, it should be
presumed that it is the same as our own, and Article 71 of the Civil Code (a marriage contracted
outside of the Philippines which is valid under the law of the country in which it was celebrated
is also valid in the Philippines) cannot be invoked.

WHEREFORE, the decision appealed from is reversed. As a corollary, the petition for
mandamus filed before the court a quo is hereby dismissed. No costs.

Republic of the Philippines


SUPREME COURT
Manila

EN BANC

G.R. No. L-21076 March 31, 1965

WONG WOO YIU alias NG YAO, petitioner-appellee,


vs.
HON. MARTINIANO P. VIVO, ETC., ET AL., respondents-appellants.

Platon A. Baysa for petitioner-appellee.


Office of the Solicitor General for respondents-appellants.

BAUTISTA ANGELO, J.:

On June 28, 1961, the Board of Special Inquiry No. 3 rendered a decision finding petitioner to
be legally married to Perfecto Blas and admitting her into the country as a non-quota immigrant.
This decision was affirmed by the Board of Commissioners on July 12, 1961 of which petitioner
was duly informed in a letter sent on the same date by the Secretary of the Board. However, on
June 28, 1962, the same Board of Commissioners, but composed entirely of a new set of
members, rendered a new decision reversing that of the Board of Special Inquiry No. 3 and
ordering petitioner to be excluded from the country. On August 9, 1962, petitioner filed a motion
for new trial requesting an opportunity to clarify certain points taken in the decision, but the
same was denied for lack of merit. Whereupon, on September 14, 1962, petitioner initiated the
instant petition for mandamus with preliminary injunction before the Court of First Instance of
Manila which incidentally was considered by it as a petition forcertiorari.

In due time, respondents filed their answer, and, after the parties had submitted a written
stipulation of facts, attaching thereto some documentary evidence, the court a quo rendered a
decision granting in, toto the relief prayed for. Thus, the court declared valid the decision
rendered by the Board of Special Inquiry No. 3 while it restrained respondents from excluding
petitioner from the country. Respondents interposed the present appeal.

It appears that in the proceedings held before the Board of Special Inquiry sometime in June,
1961, petitioner declared that she came to the Philippines in 1961 for the first time to join her
husband Perfecto Blas to whom she was married in Chingkang, China on January 15, 1929;
that they had several children all of whom are not in the Philippines; that their marriage was
celebrated by one Chua Tio, a village leader; that on June 28, 1961 the Board of Special Inquiry
No. 3 rendered a decision finding, among others, that petitioner is legally married to Perfecto
Blas, a Filipino Citizen, and admitted her into the country as a non-quota immigrant; that this
decision was affirmed by the Board of Commissioners of which petitioner was duly notified by
the Secretary of said Board in a letter dated July 12, 1961; that in a motu proprio decision
rendered by the Board of Commissioners composed of a new set of members dated June 28,
1962 the latter found that petitioner's claim that she is the lawful wife of Perfecto Blas was
without basis in evidence as it was "bereft of substantial proof of husband-wife relationship"; that
said Board further held that, it appearing that in the entry proceedings of Perfecto Blas had on
January 23, 1947 he declared that he first visited China in 1935 and married petitioner in 1936,
it could not possibly sustain her claim that she married Perfecto Blas in 1929; that in an affidavit
dated August 9, 1962 Perfecto Blas claimed that he went to China in 1929, 1935 and 1941,
although in his re-entry declaration he admitted that he first went to China in 1935, then in 1937,
then in 1939, and lastly in 1941; and that Perfecto Blas in the same affidavit likewise claimed
that he first went to China when he was merely four years old so that computed from his date of
birth in 1908 it must have been in 1912.1äwphï1.ñët

In view of the discrepancies found in the statements made by petitioner and her alleged
husband Perfecto Blas in the several investigations conducted by the immigration authorities
concerning their alleged marriage before a village leader in China in 1929, coupled with the fact
that the only basis in support of petitioner's claim that she is the lawful wife of Perfecto Blas is "a
mass of oral and documentary evidence bereft of substantial proof of husband-wife
relationship," the Board of Commissioners motu proprio reviewed the record concerning the
admission of petitioner into the country resulting in its finding that she was improperly admitted.
Thus, said Board made the following comment:

The only basis in support of the claim that she is the wife of Perfecto Blas is a mass of
oral and documentary evidence bereft of substantial proof of husband-wife relationship.
She relies on the records of Perfecto Blas in connection with his cancellation case and
the testimony of the supposed children in the previous admission proceeding. But this
claim is belied by the admission of Perfecto Blas himself, in the hearing conducted by a
Board of special inquiry in connection with his entry on January 23, 1947, that he was
married to one Ng Yo in Ki Say, Chingkang, China in 1936, his first visit there being in
1935; he could not therefore have been married to herein applicant in 1929.
The above comment cannot be disputed, it finding support in the record. Indeed, not only is
there no documentary evidence to support the alleged marriage of petitioner to Perfecto Blas
but the record is punctured with so many inconsistencies which cannot but lead one to doubt
their veracity concerning the pretended marriage in China in 1929. This claim cannot also be
entertained under our law on family relations. Thus, Article 15 of our new Civil Code provides
that laws relating to family rights or to the status of persons are binding upon citizens of the
Philippines, even though living abroad, and it is well-known that in 1929 in order that a marriage
celebrated in the Philippines may be valid it must be solemnized either by a judge of any court
inferior to the Supreme Court, a justice of the peace, or a priest or minister of the gospel of any
denomination duly registered in the Philippine Library and Museum (Public Act 3412, Section 2).
Even if we assume, therefore, that the marriage of petitioner to Perfecto Blas before a village
leader is valid in China, the same is not one of those authorized in our country.

But it may be contended that under Section 4 of General orders No. 68, as reproduced in
Section 19 of Act No. 3613, which is now Article 71 of our new Civil Code, a marriage
contracted outside of the Philippines which is valid under the law of the country in which it was
celebrated is also valid in the Philippines. But no validity can be given to this contention
because no proof was presented relative to the law of marriage in China. Such being the case,
we should apply the general rule that in the absence of proof of the law of a foreign country it
should be presumed that it is the same as our own.

The statutes of other countries or states must be pleaded and proved the same as any
other fact. Courts cannot take judicial notice of what such laws are. In the absence of
pleading and proof the laws of a foreign country or state will be presumed to be the
same as our own. (Yam Ka Lim v. Collector of Customs, 30 Phil. 46).

In the absence of anything to the contrary as to the character of a foreign law, it will be
presumed to be the same as the domestic law on the same subject. (Lim and Lim vs.
Collector of Customs, 36 Phil. 472).

In the absence of evidence to the contrary foreign laws on a particular subject are
presumed to be the same as those of the Philippines. (Miciano v. Brimo, 50 Phil. 867).

Since our law only recognizes a marriage celebrated before any of the officers mentioned
therein, and a village leader is not one of them, it is clear that petitioner's marriage, even if true,
cannot be recognized in this jurisdiction.

WHEREFORE, the decision appealed from is reversed. As a corollary, the petition


for mandamus filed before the court a quo is hereby dismissed. No costs.

Bengzon, C.J., Concepcion, Reyes, J.B.L., Barrera, Paredes, Dizon, Regala, Makalintal,
Bengzon, J.P., and Zaldivar, JJ., concur.

ADONG VS CHAONG SENG


Digest:
CASE DIGEST:

GR. No. 18081 March 3, 1922


Mora Adong, petitioner and appellant vs. Cheong Seng Gee, opponent and appellant

Agpalo‘s Statutory Construction quoted this case to wit: ―The policy of the law, once ascertained
should be given effect by the judiciary. One way of accomplishing this mandate is to give a
statute of doubtful meaning, a construction that will promote public policy.‖

FACTS: Cheong Boo, a native of China died in Zamboanga, Philippine Islands on August 5,
1919 and left property worth nearly P100,000 which is now being claimed by two parties –

(1) Cheong Seng Gee who alleged that he was a legitimate child by marriage contracted by
Cheong Boo with Tan Bit in China in 1985, and

(2) Mora Adong who alleged that she had been lawfully married to Cheong Boo in 1896 in
Basilan, Philippine Islands and had two daughters with the deceased namely Payang and
Rosalia.

The conflicting claims to Cheong Boo‘s estate were ventilated in the lower court that ruled that
Cheong Seng Gee failed to sufficiently establish the Chinese marriage through a mere letter
testifying that Cheong Boo and Tan Bit married each other but that because Cheong Seng Gee
had been admitted to the Philippine Islands as the son of the deceased, he should share in the
estate as a natural child. With reference to the allegations of Mora Adong and her daughters,
the trial court reached the conclusion that the marriage between Adong and Cheong Boo had
been adequately proved but that under the laws of the Philippine Islands it could not be held to
be a lawful marriage and thus the daughter Payang and Rosalia would inherit as natural
children. The lower court believes that Mohammedan marriages are not valid under the
Philippine Island‘s laws this as an Imam as a solemnizing officer and under Quaranic laws.

ISSUES: Whether or not the Chinese marriage between Cheong Boo and Tan Dit is valid
Whether or not the Mohammedan marriage between Cheong Boo and Mora Adong is valid

RULING: The Supreme Court found the (1) Chinese marriage not proved and Chinaman
Cheong Seng Gee has only the rights of a natural child while (2) it found the Mohammedan
marriage to be proved and to be valid, thus giving to the widow Mora Adong and the legitimate
children Payang and Rosalia the rights accruing to them under the law.

HELD:

(FOR STATCON) The Supreme Court held that marriage in this jurisdiction is not only a civil
contract but it is a new relation, an instruction in the maintenance of which the public is deeply
interested. The presumption as to marriage is that every intendment of the law leans toward
legalizing matrimony. Persons dwelling together in apparent matrimony are presumed, in the
absence of counter-presumption or evidence special to the case, to be in fact married. The
reason is that such is the common order of society, and if the parties were not what they thus
hold themselves out as being, they would be living in the constant violation of decency of the
law. As to retroactive force, marriage laws is in the nature of a curative provision intended to
safeguard society by legalizing prior marriages. Public policy should aid acts intended to
validate marriages and should retard acts intended to invalidate marriages. This as for public
policy, the courts can properly incline the scales of their decision in favor of that solution which
will most effectively promote the public policy. That is the true construction which will best carry
legislative intention into effect.

(FOR PERSONS) Sec. IV of the Marriage law provides that ―all marriages contracted outside
the islands, which would be valid by the laws of the country in which the same were contracted,
are valid in these islands. To establish a valid foreign marriage pursuant to this comity provision,
it is first necessary to prove before the courts ofthe Islands the existence of the foreign law as a
question of fact, and it is then necessary to prove the alleged foreign marriage by convincing
evidence. A Philippine marriage followed by 23 years of uninterrupted marital life, should not be
impugned and discredited, after the death of the husband through an alleged prior Chinese
marriage, ―save upon proof so clear, strong and unequivocal as to produce a moral conviction of
the existence of such impediment.‖ A marriage alleged to have been contracted in China and
proven mainly by a so-called matrimonial letter held not to be valid in the Philippines.

Full Case:

Republic of the Philippines


SUPREME COURT
Manila

EN BANC

G.R. No. 18081 March 3, 1922

IN THE MATTER OF THE ESTATE OF CHEONG BOO, deceased.


MORA ADONG, petitioner-appellant,
vs.
CHEONG SENG GEE, opponent-appellant.

Kincaid, Perkins & Kincaid and P. J. Moore for petitioner-appellant.


Carlos A. Sobral for opponent-appellant.

MALCOLM, J.:

The two question presented for determination by these appeals may be framed as follows: Is a
marriage contracted in China and proven mainly by an alleged matrimonial letter, valid in the
Philippines? Are the marriage performed in the Philippines according to the rites of the
Mohammedan religion valid? As the decision of the Supreme Court on the last point will affect
marriages consummated by not less than one hundred and fifty thousand Moros who profess
the Mohammedan faith, the transcendental importance of the cause can be realized. We
proposed to give to the subject the serious consideration which it deserves.
Cheong Boo, a native of China, died intestate in Zamboanga, Philippine Islands, on August 5,
1919. He left property worth nearly P100,000. The estate of the deceased was claimed, on the
one hand, by Cheong Seng Gee, who alleged that he was a legitimate child by a marriage
contracted by Cheong Boo with Tan Dit in China in 1895. The estate was claimed, on the other
hand, by the Mora Adong who alleged that she had been lawfully married to Cheong Boo in
1896 in Basilan, Philippine Islands, and her daughters, Payang, married to Cheng Bian Chay,
and Rosalia Cheong Boo, unmarried.

The conflicting claims to the estate of Cheong Boo were ventilated in the Court of First Instance
of Zamboanga. The trial judge, the Honorable Quirico Abeto, after hearing the evidence
presented by both sides, reached the conclusion, with reference to the allegations of Cheong
Seng Gee, that the proof did not sufficiently establish the Chinese marriage, but that because
Cheong Seng Gee had been admitted to the Philippine Islands as the son of the deceased, he
should share in the estate as a natural child. With reference to the allegations of the Mora
Adong and her daughters Payang and Rosalia, the trial judge reached the conclusion that the
marriage between the Mora Adong and the deceased had been adequately proved but that
under the laws of the Philippine Islands it could not be held to be a lawful marriage; accordingly,
the daughters Payang and Rosalia would inherit as natural children. The order of the trial judge,
following these conclusions, was that there should be a partition of the property of the deceased
Cheong Boo between the natural children, Cheong Seng Gee, Payang, and Rosalia.

From the judgment of the Judge of First Instance both parties perfected appeals. As to the facts,
we can say that we agree in substance with the findings of the trial court. As to the legal issues
submitted for decision by the numerous assignments of error, these can best be resolved under
two heads, namely: (1) The validity of the Chinese marriage; and (2) the validity of the
Mohammedan marriage.

1. Validity of the Chinese Marriage

The theory advanced on behalf of the claimant Cheong Seng Gee was that Cheong Boo was
married in the city of Amoy, China, during the second moon of the twenty-first year of the
Emperor Quang Su, or, according to the modern count, on February 16, 1985, to a young lady
named Tan Dit. Witnesses were presented who testified to having been present at the marriage
ceremony. There was also introduced in evidence a document in Chinese which in translation
reads as follows:

One hundred Your nephew, Tan Chao, respecfully


years of life answers the venerable Chiong Ing,
and health for father of the bridegroom, accepting his
both. offer of marriage, and let this document
serve as proof of the acceptance of
said marriage which is to be celebrated
during the merry season of the flowers.

I take advantage of this occasion to


wish for your and the spouses much
happiness, a long life, and prolific
issue, as noble and great as that which
you brought forth. I consider the
marriage of your son Boo with my
sister Lit Chia as a mandate of God
and I hope that they treat each other
with great love and mutual courtesy
and that both they and their parents be
very happy.

Given during the second moon of the


twenty-first year of the reign of the
Emperor Quang Su.

Cheong Boo is said to have remained in China for one year and four months after his marriage
during which time there was born to him and his wife a child named Cheong Seng Gee. Cheong
Boo then left China for the Philippine Islands and sometime thereafter took to himself a
concubine Mora by whom he had two children. In 1910, Cheong Boo was followed to the
Philippines by Cheong Seng Gee who, as appears from documents presented in evidence, was
permitted to land in the Philippine Islands as the son of Cheong Boo. The deceased, however,
never returned to his native hearth and seems never to have corresponded with his Chinese
wife or to have had any further relations with her except once when he sent her P10.

The trial judge found, as we have said, that the proof did not sustain the allegation of the
claimant Cheong Seng Gee, that Cheong Boo had married in China. His Honor noted a strong
inclination on the part of the Chinese witnesses, especially the brother of Cheong Boo, to
protect the interests of the alleged son, Cheong Seng Gee, by overstepping the limits of
truthfulness. His Honor also noted that reliable witnesses stated that in the year 1895, when
Cheong Boo was supposed to have been in China, he was in reality in Jolo, in the Philippine
Islands. We are not disposed to disturb this appreciation of fact by the trial court. The
immigration documents only go to show the relation of parent and child existing between the
deceased Cheong Boo and his son Cheong Seng Gee and do not establish the marriage
between the deceased and the mother of Cheong Seng Gee.

Section IV of the Marriage Law (General Order No. 68) provides that "All marriages contracted
without these Islands, which would be valid by the laws of the country in which the same were
contracted, are valid in these Islands." To establish a valid foreign marriage pursuant to this
comity provision, it is first necessary to prove before the courts of the Islands the existence of
the foreign law as a question of fact, and it is then necessary to prove the alleged foreign
marriage by convincing evidence.

As a case directly in point is the leading one of Sy Joc Lieng vs. Encarnacion ([1910]), 16 Phil.,
137; [1913], 228 U.S., 335). Here, the courts of the Philippines and the Supreme Court of the
United States were called upon to decide, as to the conflicting claims to the estate of a Chinese
merchant, between the descendants of an alleged Chinese marriage and the descendants of an
alleged Philippine marriage. The Supreme Courts of the Philippine Islands and the United
States united in holding that the Chinese marriage was not adequately proved. The legal rule
was stated by the United States Supreme Court to be this: A Philippine marriage, followed by
forty years of uninterrupted marital life, should not be impugned and discredited, after the death
of the husband and administration of his estate, though an alleged prior Chinese marriage,
"save upon proof so clear, strong, and unequivocal as to produce a moral conviction of the
existence of such impediment." Another case in the same category is that of Son
Cui vs. Guepangco ([1912], 22 Phil., 216).

In the case at bar there is no competent testimony as to what the laws of China in the Province
of Amoy concerning marriage were in 1895. As in the Encarnacion case, there is lacking proof
so clear, strong, and unequivocal as to produce a moral conviction of the existence of the
alleged prior Chinese marriage. Substitute twenty-three years for forty years and the two cases
are the same.

The lower court allowed the claimant, Cheong Seng Gee, the testamentary rights of an
acknowledged natural child. This finding finds some support in Exhibit 3, the affidavit of Cheong
Boo before the American Vice-Consul at Sandakan, British North Borneo. But we are not called
upon to make a pronouncement on the question, because the oppositor-appellant indicates
silent acquiescence by assigning no error.

2. Validity of the Mohammedan Marriage

The biographical data relating to the Philippine odyssey of the Chinaman Cheong Boo is fairly
complete. He appears to have first landed on Philippine soil sometime prior to the year 1896. At
least, in the year las mentioned, we find him in Basilan, Philippine Islands. There he was
married to the Mora Adong according to the ceremonies prescribed by the book on marriage of
the Koran, by the Mohammedan Iman (priest) Habubakar. That a marriage ceremony took place
is established by one of the parties to the marriage, the Mora Adong, by the Iman who
solemnized the marriage, and by other eyewitnesses, one of whom was the father of the bride,
and another, the chief of the rancheria, now a municipal councilor. The groom complied with
Quranic law by giving to the bride a dowry of P250 in money and P250 in goods.

The religious rites began with the bride and groom seating themselves in the house of the father
of the bride, Marahadja Sahibil. The Iman read from the Koran. Then the Iman asked the
parents if they had any objection to the marriage. The marital act was consummated by the
groom entering the woman's mosquito net.

From the marriage day until the death of Cheong Boo, twenty-three years later, the Chinaman
and the Mora Adong cohabited as husband and wife. To them were born five children, two of
whom, Payang and Rosalia, are living. Both in his relations with Mora Adong and with third
persons during his lifetime, Cheong Boo treated Adong as his lawful wife. He admitted this
relationship in several private and public documents. Thus, when different legal documents
were executed, including decrees of registration, Cheong Boo stated that he was married to the
Mora Adong while as late as 1918, he gave written consent to the marriage of his minor
daughter, Payang.

Notwithstanding the insinuation of counsel for the Chinese appellant that the custom is
prevalent among the Moros to favor in their testimony, a relative or friend, especially when they
do not swear on the Koran to tell the truth, it seems to us that proof could not be more
convincing of the fact that a marriage was contracted by the Chinaman Cheong Boo and the
Mora Adong, according to the ceremonies of the Mohammedan religion.

It is next incumbent upon us to approach the principal question which we announced in the very
beginning of this decision, namely, Are the marriages performed in the Philippines according to
the rites of the Mohammedan religion valid? Three sections of the Marriage Law (General Order
No. 68) must be taken into consideration.

Section V of the Marriage Law provides that "Marriage may be solemnized by either a judge of
any court inferior to the Supreme Court, justice of the peace, or priest or minister of the Gospel
of any denomination . . ." Counsel, failing to take account of the word "priest," and only
considering the phrase "minister of the Gospel of any denomination" would limit the meaning of
this clause to ministers of the Christian religion. We believe this is a strained interpretation.
"Priest," according to the lexicographers, means one especially consecrated to the service of a
divinity and considered as the medium through whom worship, prayer, sacrifice, or other service
is to be offered to the being worshipped, and pardon, blessing, deliverance, etc., obtained by
the worshipper, as a priest of Baal or of Jehovah; a Buddhist priest. "Minister of the Gospel"
means all clergymen of every denomination and faith. A "denomination" is a religious sect
having a particular name. (Haggin vs. Haggin [1892], 35 Neb., 375; In reReinhart, 9 O. Dec.,
441; Hale vs. Everett [1868], 53 N. H. 9.) A Mohammedan Iman is a "priest or minister of the
Gospel," and Mohammedanism is a "denomination," within the meaning of the Marriage Law.

The following section of the Marriage Law, No. VI, provides that "No particular form for the
ceremony of marriage is required, but the parties must declare, in the presence of the person
solemnizing the marriage, that they take each other as husband and wife." The law is quite
correct in affirming that no precise ceremonial is indispensable requisite for the creation of the
marriage contract. The two essentials of a valid marriage are capacity and consent. The latter
element may be inferred from the ceremony performed, the acts of the parties, and habit or
repute. In this instance, there is no question of capacity. Nor do we think there can exist any
doubt as to consent. While it is true that during the Mohammedan ceremony, the remarks of the
priest were addressed more to the elders than to the participants, it is likewise true that the
Chinaman and the Mora woman did in fact take each other to be husband and wife and did
thereafter live together as husband and wife. (Travers vs. Reinhardt [1907], 205 U.S., 423.

It would be possible to leave out of view altogether the two sections of the Marriage Law which
have just been quoted and discussed. The particular portion of the law which, in our opinion, is
controlling, is section IX, reading as follows: "No marriage heretofore solemnized before any
person professing to have authority therefor shall be invalid for want of such authority or on
account of any informality, irregularity, or omission, if it was celebrated with the belief of the
parties, or either of them, that he had authority and that they have been lawfully married."

The trial judge in construing this provision of law said that he did not believe that the legislative
intention in promulgating it was to validate marriages celebrated between Mohammedans. To
quote the judge:

This provisions relates to marriages contracted by virtue of the provisions of the Spanish
law before revolutionary authorized to solemnized marriages, and it is not to be
presumed that the legislator intended by this law to validate void marriages celebrated
during the Spanish sovereignty contrary to the laws which then governed.

What authority there is for this statement, we cannot conceive. To our mind, nothing could be
clearer than the language used in section IX. Note for a moment the all embracing words found
in this section:
"No marriage" — Could more inclusive words be found? "Heretofore solemnized" — Could any
other construction than that of retrospective force be given to this phrase? "Before any person
professing to have authority therefor shall be invalid for want of such authority" — Could
stronger language than this be invoked to announce legislative intention? "Or on account of any
informality, irregularity, or omission" — Could the legislative mind frame an idea which would
more effectively guard the marriage relation against technicality? "If it was celebrated with the
belief of the parties, or either of them, that he had authority and that they have been lawfully
married" — What was the purpose of the legislator here, if it was not to legalize the marriage, if
it was celebrated by any person who thought that he had authority to perform the same, and if
either of the parties thought that they had been married? Is there any word or hint of any word
which would restrict the curative provisions of section IX of the Marriage Law to Christian
marriages? By what system of mental gymnastics would it be possible to evolve from such
precise language the curious idea that it was restricted to marriages performed under the
Spanish law before the revolutionary authorities?

In view of the importance of the question, we do not desire to stop here but would ascertain
from other sources the meaning and scope of Section IX of General Order No. 68.

The purpose of the government toward the Mohammedan population of the Philippines has,
time and again, been announced by treaty, organic law, statutory law, and executive
proclamation. The Treaty of Paris in its article X, provided that "The inhabitants of the territories
over which Spain relinquishes or cedes her sovereignty shall be secured Instructions to the
Philippine Commission imposed on every branch of the Government of the Philippine Islands
the inviolable rule "that no law shall be made respecting an establishment of religion or
prohibiting the free exercise thereof, and that the free exercise and enjoyment of religious
profession and worship, without discrimination or preference, shall forever be allowed ... That no
form of religion and no minister of religion shall be forced upon any community or upon any
citizen of the Islands; that, upon the other hand, no minister of religion shall be interfered with or
molested in following his calling, and that the separation between state and church shall be real,
entire, and absolute." The notable state paper of President McKinley also enjoined the
Commission, "to bear in mind that the Government which they are establishing is designed . . .
for the happiness, peace, and prosperity of the people of the Philippine Islands" and that,
therefore, "the measures adopted should be made to conform to their customs, their habits, and
even their prejudices. . . . The Philippine Bill and the Jones Law reproduced the main
constitutional provisions establishing religious toleration and equality.

Executive and legislative policy both under Spain and the United States followed in the same
path. For instance, in the Treaty of April 30, 1851, entered into by the Captain General of the
Philippines and the Sultan of Sulu, the Spanish Government guaranteed "with all solemnity to
the Sultan and other inhabitants of Sulu the free exercise of their religion, with which it will not
interfere in the slightest way, and it will also respect their customs." (See furtherDecree of the
Governor-General of January 14, 1881.) For instance, Act No. 2520 of the Philippine
Commission, section 3, provided that "Judges of the Court of First Instance and justices of the
peace deciding civil cases in which the parties are Mohammedans or pagans, when such action
is deemed wise, may modify the application of the law of the Philippine Islands, except laws of
the United States applicable to the Philippine Islands, taking into account local laws and
customs. . . ." (See further Act No. 787, sec. 13 [ j]; Act No. 1283, sec. 6 [b]; Act No. 114 of the
Legislative Council amended and approved by the Philippine Commission;
Cacho vs. Government of the United States [1914], 28 Phil., 616.) Various responsible officials
have so oft announced the purpose of the Government not to interfere with the customs of the
Moros, especially their religious customs, as to make quotation of the same superfluous.

The retrospective provisions of the Philippine Marriage Law undoubtedly were inspired by the
governmental policy in the United States, with regard to the marriages of the Indians, the
Quakers, and the Mormons. The rule as to Indians marriages is, that a marriage between two
Indians entered into according to the customs and laws of the people at a place where such
customs and laws are in force, must be recognized as a valid marriage. The rule as to the
Society of Quakers is, that they will be left to their own customs and that their marriages will be
recognized although they use no solemnization. The rule as to Mormon marriages is that the
sealing ceremony entered into before a proper official by members of that Church competent to
contract marriage constitutes a valid marriage.

The basis of human society throughout the civilized world is that of marriage. Marriage in this
jurisdiction is not only a civil contract, but, it is a new relation, an institution in the maintenance
of which the public is deeply interested. Consequently, every intendment of the law leans toward
legalizing matrimony. Persons dwelling together in apparent matrimony are presumed, in the
absence of any counter-presumption or evidence special to the case, to be in fact married. The
reason is that such is the common order of society, and if the parties were not what they thus
hold themselves out as being, they would be living in the constant violation of decency and of
law. A presumption established by our Code of Civil Procedure is "that a man and woman
deporting themselves as husband and wife have entered into a lawful contract of marriage.:"
(Sec. 334, No. 28.) Semper praesumitur pro matrimonio — Always presume marriage. (U.
S. vs. Villafuerte and Rabano [1905], 4 Phil., 476; Son Cui vs.Guepangco, supra;
U.S. vs. Memoracion and Uri [1916], 34 Phil., 633; Teter vs. Teter [1884], 101 Ind., 129.)

Section IX of the Marriage Law is in the nature of a curative provision intended to safeguard
society by legalizing prior marriages. We can see no substantial reason for denying to the
legislative power the right to remove impediments to an effectual marriage. If the legislative
power can declare what shall be valid marriages, it can render valid, marriages which, when
they took place, were against the law. Public policy should aid acts intended to validate
marriages and should retard acts intended to invalidate marriages. (Coghsen vs. Stonington
[1822], 4 Conn, 209; Baity vs. Cranfill [1884], 91 N. C., 273.)

The courts can properly incline the scales of their decisions in favors of that solution which will
mot effectively promote the public policy. That is the true construction which will best carry
legislative intention into effect. And here the consequences, entailed in holding that the marriage
of the Mora Adong and the deceased Cheong Boo, in conformity with the Mohammedan religion
and Moro customs, was void, would be far reaching in disastrous result. The last census shows
that there are at least one hundred fifty thousand Moros who have been married according to
local custom. We then have it within our power either to nullify or to validate all of these
marriages; either to make all of the children born of these unions bastards or to make them
legitimate; either to proclaim immorality or to sanction morality; either to block or to advance
settled governmental policy. Our duty is a obvious as the law is plain.

In moving toward our conclusion, we have not lost sight of the decisions of this court in the
cases of United Statesvs. Tubban ([1915]), 29 Phil., 434) and United States vs. Verzola ([1916,
33 Phil., 285). We do not, however, believe these decisions to be controlling. In the first place,
these were criminal actions and two Justice dissented.. In the second place, in the Tubban
case, the marriage in question was a tribal marriage of the Kalingas, while in the Verzola case,
the marriage had been performed during the Spanish regime by a lieutenant of the Guardia
Civil. In neither case, in deciding as to whether or not the accused should be given the benefit of
the so-called unwritten law, was any consideration given to the provisions of section IX of
General Order No. 68. We are free to admit that, if necessary, we would unhesitatingly revoke
the doctrine announced in the two cases above mentioned.

We regard the evidence as producing a moral conviction of the existence of the Mohammedan
marriage. We regard the provisions of section IX of the Marriage law as validating marriages
performed according to the rites of the Mohammedan religion.

There are other questions presented in the various assignments of error which it is unnecessary
to decide. Inresume, we find the Chinese marriage not to be proved and that the Chinaman
Cheong Seng Gee has only the rights of a natural child, and we find the Mohammedan marriage
to be proved and to be valid, thus giving to the widow and the legitimate children of this union
the rights accruing to them under the law.

Judgment is reversed in part, and the case shall be returned to the lower court for a partition of
the property in accordance with this decision, and for further proceedings in accordance with
law. Without special findings as to costs in this instance, it is so ordered.

Araullo, C.J., Johnson, Street, Avanceña, Villamor, Ostrand, Johns and Romualdez, JJ., concur.

CHING SUAT VS CON HEONG


PEOPLE VS MORA DUMPO

Republic of the Philippines


SUPREME COURT
Manila

EN BANC

G.R. No. L-42581 October 2, 1935

THE PEOPLE OF THE PHILIPPINE ISLANDS, plaintiff-appellee,


vs.
MORA DUMPO, defendant-appellant.

Arturo M. Tolentino for appellant.


Office of the Solicitor-General Hilado for appellee.

RECTO, J.:

Moro Hassan and Mora Dupo have been legally married according to the rites and
practice of the Mohammedan religion. Without this marriage being dissolved, it is alleged that
Dumpo contracted another marriage with Moro Sabdapal after which the two lived together as
husband and wife. Dumpo was prosecuted for and convicted of the crime of bigamy in the Court
of First Instance of Zamboanga and sentenced to an indeterminate penalty with a maximum of
eight years, and one day of prision mayor and minimum of two years, four months and twenty
one days of prision correccional, with costs. From this judgment the accused interposed an
appeal. The records of the case disclose that it has been established by the defense, without
the prosecution having presented any objection nor evidence to the contrary, that the alleged
second marriage of the accused is null and void according to Mohammedan rites on the ground
that her father had not given his consent thereto.

We formulate no general statement regarding the requisites necessary for the validity of a
marriage between Moros according to Mohammedan rites. This is a fact of which no judicial
notice may be taken and must be subject to proof in every particular case. In the case at bar we
have the uncontradicted testimony of Tahari, an Iman or Mohammedan priest authorized to
solemnize marriages between Mohammedans, to the effect that the consent of the bride's father
or. in the absence thereof, that of the chief of the tribe to which she belongs in an indipensable
requisite for the validity of such contracts. If the absence of this requisite did not make the
marriage contract between Mohammedans void, it was easy for the prosecution to show it by
refuting Iman Tahari's testimony inasmuch as for lack of one there were two other Imans among
the State witnesses in this case. It failed to do so, however, and from such failure we infer that
the Iman's testimony for the defense is in accordance with truth. It is contended that, granting
the absolute necessity of the requisite in question, tacit compliance therewith may be presumed
because it does not appear that Dumpo's father has signified his opposition to this alleged
marriage after he had been informed of its celebration. But this presumption should not be
established over the categorical affirmation of Moro Jalmani, Dumpo's father, that he did not
give his consent to his daughter's alleged second marriage for the reason that he was not
informed thereof and that, at all events, he would not have given it, knowing that Dumpo's first
marriage was not dissolved.

It is an essential element of the crime of bigamy that the alleged second marriage, having
all the essential requisites, would be valid were it not for the subsistence of the first marriage. It
appearing that the marriage alleged to first been contracted by the accused with Sabdapal, her
former marriage with Hassan being undissolved, cannot be considered as such, there is no
justification to hold her guilty of the crime charged in the information.

Wherefore, reversing the appealed judgment, the accused is acquitted of the charges and
if she should be in detention her immediate release is ordered, with the costs of both
instances de oficio. So ordered.

Avanceña, C.J., Malcolm, Villa-Real, Abad Santos, and Imperial, JJ., concur.

Separate Opinions
HULL, J., dissenting:

Under section 25 of the Marriage Law Act No. 3613, "marriages between Mohammedans
may be performed in accordance with the rites or practices of their religion . . .," and as stated in
the majority opinion we cannot take judicial notice of such matters but that they are subject to
proof, as is any foreign law. The person "learned in the law" who was presented as an expert
witness for the defense, did not know his age, but it was estimated by the judgment as fifty-four
years. He gave his occupation as that of a fisherman and stated he had performed two
marriages. He does not know how to read. He was not asked any questions as to whether there
was a difference between the marriage of a young woman living with her parents or a woman
who had been emancipated.

In the evidence of the prosecution, it was shown that the second marriage was
solemnized by an Iman, a Mohammedan religious official, seventy years old, who was living in
the vicinity of the contracting parties.lawphil.net

There was no quotation from the Koran regarding the essentials of a marriage ceremony
according to the Mohammedan religion, and I agree with the trial court that the evidence relied
upon by the majority opinion, is not worthy of serious consideration. If consent were in fact
necessary, it can well be presumed from the subsequent actions of the father of the girl. She left
his house and after the second ceremony lived openly with her second husband, this with her
father's full knowledge and at least tacit consent, for many months.

I therefore believe that the sentence appealed from should be affirmed.

Vickers, Goddard, and Diaz, concurs.

LAO AND LAO VS DEE TIM

Republic of the Philippines


SUPREME COURT
Manila

EN BANC

DECISION

February 25, 1924

G.R. No. L-21017


In re estate of JOSE YAP SIONG, deceased. MARIA LAO and JOSE LAO, petitioners-
appellees,
vs.
DEE TIM, YAP KIM TING, YAP KIM SENG, and YAP HU CHO, respondents-appellants.
Salvador Barrios and Gabino S. Abaya for appellants.
Felix B. Bautista and Jose Gutierrez David for appellees.

Johnson, J.:

JOSE YAP SIONG. MARIA LAO, ET AL. vs. DEE TIM, ET AL.

Republic of the PhilippinesSUPREME COURTManila

EN BANC

G.R. No. L-21017 February 25, 1924

In re estate of JOSE YAP SIONG, deceased. MARIA LAO and JOSE LAO, petitioners-
appellees,

vs.

DEE TIM, YAP KIM TING, YAP KIM SENG, and YAP HU CHO, respondents-appellants.

Salvador Barrios and Gabino S. Abaya for appellants.

Felix B. Bautista and Jose Gutierrez David for appellees.

JOHNSON, J.:

It appears from the record that on the 5th day of September, 1922, Yap Siong died in the
municipality of Angeles, Province of Pampanga, Philippine Islands, leaving a considerable
amount of property to be distributed among his heirs. An administrator was appointed to
administer his estate. During the course of the administration and distribution of the estate there
appeared the petitioners and the respondents, each claiming to be the legitimate heirs of Yap
Siong and entitled to his estate. The petitioner Maria Lao claims to be the legitimate widow of
Yap Siong, having been legally joined to him in holy wedlock on the 24th day of June, 1903, in
the Philippine Islands (Exhibit 1) and that Jose Lao is a legitimate child born of that marriage,
and that they are therefore entitled, as heirs, to the estate of Yap Siong, deceased.

Upon the other hand Dee Tim claims to be the legitimate widow of Yap Siong; that she and Yap
Siong were joined in the holy wedlock on the 14th day of September, 1893, in accordance with
the laws of China (Exhibits A and A-1), and that the said Yap Kim Ting, Yap Kim Seng, and Yap
Hu Cho were her legitimate children born of that wedlock.

In support of the contention of the petitioners, Maria Lao and her son Jose Lao, a great deal of
proof was presented. Exhibits 1 and 1-A, certificates of marriage, were presented to show that
she had been legally married to Yap Siong. A number of other documents (Exhibits 9 to 13)
were presented to show that Yap Siong had admitted that he was a married man. Exhibits 14 to
17 were presented for the purpose of proving that Yap Siong had admitted in a public document
that Maria Lao was his wife.

The respondent Dee Tim presented a great deal of proof to show that she was the legitimate
wife of Yap Siong, lawfully joined to him in holy wedlock in China on the 14th day of September,
1893. To support that contention she presented what she contended was a certificate of
marriage, marked Exhibit A — Exhibit A-1. She contended that Exhibit A was positive proof of
her marriage and that it complied with the custom and practice in China with reference to
marriage ceremonies. To support her contention she presented a number of witnesses. Jan
Peng, a Chinaman of 52 years of age, swore that he knew the forms of ceremonies of marriage
in China, and that Exhibit A was the ordinary and customary document issued to prove that the
ceremony of marriage had taken place. He described in detail the ceremony of marriage
performed in accordance with the customs and practice in China.

Dee Tim also presented a witness, Ty Cong Ting, a Chinaman, 32 years of age and a lawyer,
who testified concerning the laws and customs in China with reference to the forms of marriage
ceremony. He testified that he knew and was well acquainted with the customs and practices of
Chinamen in China with reference to marriages and the manner and form in which they were
celebrated, and the form of proof issued for the purpose of proving that a marriage ceremony
had been performed. He further testified that Exhibit A was the usual proof or certificate issued
for the purpose of proving that a marriage ceremony had taken place. He further testified that
Exhibit A was the usual and ordinary proof, or certificate, if it may be called a certificate, issued
to show that a marriage ceremony had been performed between the persons mentioned therein.
Mr. Ty Cong Ting was, at the time he declared as a witness, the legal attorney of the Chinese
Consul General in the City of Manila.

The respondent Dee Tim presented several witnesses who confirmed her contention that she
was the legitimate wife of Yap Siong and that her three children Yap Kim Ting, Yap Kim Seng,
and Yap Hu Cho were her legitimate children, born of her marriage with Yap Siong. To further
sustain her contention she presented Exhibits B, C, D, E, F, G, H, I, and J, documents in which
Yap Siong had expressly recognized his marriage to her.

To overcome the proof adduced by Dee Tim in support of her marriage to Yap Siong, the
petitioner presented Exhibits 2,3,4,5,6,7, and 8. Said exhibits are alleged letters supposed to
have been written by an uncle in China of Yap Siong during the years 1900 to 1906, urging him
to return to China for the purpose of marrying, thus attempting to establish the fact that Yap
Siong during that period was not a married man. When we first studied the record in this case
we were inclined to give said letters great credit, but upon a further examination of the record
and a further argument by the respective parties, we are now inclined to believe that said letters
were fabricated for the very purpose of defeating the contention of Dee Tim. They were not
identified properly by persons who had reason to know that they were genuine in character and
were actually prepared in China and sent to Yap Siong in the Philippine Islands. We are of the
opinion, and we believe that was the real opinion of the trial court, that said exhibits should not
be admitted as proof to sustain the fact for which they were presented. We are now persuaded
that said letters are pure fabrications.

The petitioner further presents two or three witnesses for the purpose of showing that the
marriage between Dee Tim and Yap Siong never took place for the reason that Yap Siong was
in the Philippine Islands on the 14th day of September, 1893, and that at that time he was living
in the municipality of Bacolor, of the Province of Pampanga, and that he never left that
municipality. A careful reading of their testimony, however, does not convince us that it is
altogether reliable. The testimony which they gave was given in the month of January, 1923,
and they testified positively as to exact dates, times, and places in the year 1897. Their
testimony contains no facts, or data, or peculiar circumstances or conditions which caused them
to remember the particular facts concerning which they testified. They gave no reason why they
were able to remember the exact whereabouts of Yap Siong during the period to which their
testimony referred. Upon the contrary there is much proof in the record that Yap Siong returned
to China a number of times after his first arrival here. The petitioner further presents some proof
to show that Yap Siong had admitted on several occasions that Dee Tim was his querida and
not his wife.

The respondents further attempted to show that Maria Lao and Jose Lao, her son, were not the
legitimate wife and son of Yap Siong, by presenting Exhibits L and LL. Exhibit L is the baptismal
certificate issued by the parish priest of the municipality of Angeles, in which it is made to
appear that on the 5th day of January, 1904, he baptized a child named Jose Martin, a natural
son of Maria Lao, and whose father was unknown. Exhibit LL is a certificate of birth issued by
the secretary of the municipality of Angeles, in which it appears that Jose Martin Lao, a child,
was born on the first day of January, 1904, a natural son of Maria Lao. There is nothing,
however, in Exhibits L or LL, which shows that Maria Lao was responsible for the facts which
they contain. Exhibit LL contains the statement that the facts therein were not obtained from
Maria Lao but from one Isabelo Lao.

There is a notable conflict between Exhibits L and LL. Exhibit LL certifies that Jose Martin Lao
was born on the first day of January, 1904, while Exhibit L certifies that the baptism took place
on the 5th day of January, 1904, and that the child was then 34 days old. It is apparent therefore
that the facts stated in one or the other of said exhibits are untrue. And, moreover, when we
consider the customs of the Filipino people in their relation with the Holy Roman Catholic
Apostolic Church, it is easily understood, in view of the alleged fact that Maria Lao and Yap
Siong had been joined in holy matrimony under the forms of the Protestant Church, why the
parish priest of the municipality of Angeles stated in his certificate that the father of the child,
then Jose Martin, was unknown.

The respondents further attempted to show that Yap Siong and Maria Lao had never been
joined legally in holy wedlock, by the testimony of a number of witnesses to the fact that Yap
Siong had on numerous occassions asserted that Maria Lao was his querida only. It is perhaps
true that Yap Siong did on various occasions, depending upon his interest and convenience at
the particular time, state that Maria Lao was his querida and not his wife. It is also perhaps true,
for the same reason, that he stated that Dee Tim was not his wife but his querida. Evidently he
was attempting to keep the information, that he was quite able to do, until he had passed to that
bourn from which none returns, and until a distribution of his large accumulated earnings among
his heirs became necessary.

From all of the foregoing conflicting facts, and considering all of the facts of the record, we are
forced to the conclusion that a preponderance of the evidence shows the following:

(1) That Dee Tim and Yap Siong were legally married in China in accordance with the laws and
customs in China on the 14th day of September, 1893; that Yap Kim Ting, Yap Kim Seng, and
Yap Hu Cho were the legitimate children born of that wedlock; that Dee Tim and her said
children were ignorant of the fact that Yap Siong had legally married Maria Lao, and that Jose
Lao was born of that wedlock; and that they had no reason to believe, until after the death of
Yap Siong, that he was legally married to the petitioner herein.

(2) That Maria Lao was legally married to Yap Siong on the 24th day of June, 1903, in good
faith believing that Yap Siong was not then a married man, without any knowledge or
information or suspicion to the contrary; and that Jose Lao is the legitimate child born of that
marriage of Yap Siong and Maria Lao.

In other words, we are fully convinced that a preponderance of the evidence shows that both
Dee Tim and Maria Lao were legally married to Yap Siong in good faith, believing that each was
his sole and separate wife, living in absolute ignorance of the fact of his double marriage. They
were each married in good faith and in ignorance of the existence of the other marriage. Yap
Siong up to the time of his death seems to have been successful in keeping each of his two
wives ignorant of the fact that he was married to the other.

Under the foregoing facts, how must the property of Yap Siong be divided between the two
families? Under the Leyes de Partidas (Law 1, title 13, partida 4), where two women innocently
and in good faith are legally united in holy matrimony to the same man, their children born will
be regarded as legitimate children and each family will be entitled to one-half of the estate of the
husband upon distribution of his estate. That provision of the Leyes de Partidas is a very
humane and wise law. It justly protects those who innocently have entered into the solemn
relation of marriage and their descendants. The good faith of all the parties will be presumed
until the contrary is positively proved. (Article 69, Civil Code; Las Leyes de Matrimonio, section
96; Gaines vs. Hennen, 65 U.S., 553.)

A woman who is deceived by a man who represents himself as single and who marries him, she
and her children born while the deception lasted, under the Spanish Law, are entitled to all the
rights of a legitimate wife and children. The common law allowing none of the incidents of a true
marriage to follow another marriage entered into during the continuance of a first, was early
found to work a great injustice upon the innocent parties to the second marriage, and specially
upon the offspring of such second marriage. To remedy that hardship under the common law
and following the wise jurisprudence of Spain, both England and many of the states of the
United States adopted statutes. (Glass vs. Glass, 114 Mass., 563; Spicer vs. Spicer, 16 Abbot's
Practice [N. S.], 114; Dyer vs. Brannock, 66 Mo., 391; Graham vs. Bennet, 2 Cal., 503; Smith
vs. Smith, 1 Tex., 621 [46 Am. Dec., 121]; Clendenning vs. Clendenning, 7 Martin [La.], 587;
Patton vs. Cities of Philadelphia and New Orleans, 1 La. Ann., 98; Abston vs. Abston, 15 La.
Ann., 137; Gaines vs. Hennen, 65 U.S., 553; Ex parte Myra Clarke Whitney, 38 U.S., 404;
Estate of Navarro, 24 La. Ann., 298; In re Taylor, 39 La. Ann., 823.)

The foregoing conclusions in no way conflict with the decision of this court in the case of Sy Joc
Lieng vs. Encarnacion (16 Phil., 137) nor with the decision of Adong vs. Cheong Seng Gee (43
Phil., 43), for the reason that in each of said cases a preponderance of the evidence showed
that no legal marriage had been performed in China, that is, that the alleged Chines wife and
the deceased in each of those cases had never been legally married.

Therefore the conclusion reached in the decision heretofore announced by this court in the
present case is hereby set aside and it is hereby ordered and decreed that the judgment of the
lower court be revoked and that the estate of Yap Siong be divided equally, one-half going to
Maria Lao and her son, Jose Lao, and the other one-half to Dee Tim and her three children. And
without any finding as to costs, it so ordered.

Araullo, C.J., Street, Malcolm, Avanceña, Ostrand, Johns and Romualdez, JJ., concur. .

YAO KEE VS GONZALES

167 scra 737

Applicability of Foreign Marriage Laws

Sy Kiat, a Chinese national, died on January 17, 1977 leaving behind properties here in the
Philippines.

Thereafter, Aida Sy-Gonzales et al filed a petition for the grant of letters of administration
alleging that they are the children of the deceased with Asuncion Gillego. The petition was
opposed by Yao Kee et al alleging that Yao Kee is the lawful wife of the deceased whom he
married in China. The trial court rendered decision in favor of the opposition. On appeal, the
Court of Appeals rendered a decision, modifying the decision declaring the marriage of Sy Kiat
to Yao Kee as not has been proven valid in accordance with the laws of China. Hence, both
parties moved for reconsideration to which the Supreme Court granted.

ISSUE: Whether or not the marriage of Yao Kee and Sy Kiat is valid in accordance with
Philippine laws.
HELD: Well-established in this jurisdiction is the principle that Philippine courts cannot take
judicial notice of foreign laws. They must be alleged and proven as any other fact. To establish
the validity of marriage, the existence of foreign law as a question of fact and the alleged
marriage must be proven by clear and convincing evidence.

For failure to prove the foreign law or custom and consequently of the marriage, the marriage
between Yao Kee and Sy Kiat in China cannot be recognized in the jurisdiction of Philippine
courts.

Full Case:

Republic of the Philippines


SUPREME COURT
Manila

THIRD DIVISION

G.R. No. L-55960 November 24, 1988

YAO KEE, SZE SOOK WAH, SZE LAI CHO, and SY CHUN YEN, petitioners,
vs.
AIDA SY-GONZALES, MANUEL SY, TERESITA SY-BERNABE, RODOLFO SY, and
HONORABLE COURT OF APPEALS, respondents.

Montesa, Albon, & Associates for petitioners.

De Lapa, Salonga, Fulgencio & De Lunas for respondents.

CORTES, J.:

Sy Kiat, a Chinese national. died on January 17, 1977 in Caloocan City where he was then
residing, leaving behind real and personal properties here in the Philippines worth P300,000.00
more or less.

Thereafter, Aida Sy-Gonzales, Manuel Sy, Teresita Sy-Bernabe and Rodolfo Sy filed a petition
for the grant of letters of administration docketed as Special Proceedings Case No. C-699 of the
then Court of First Instance of Rizal Branch XXXIII, Caloocan City. In said petition they alleged
among others that (a) they are the children of the deceased with Asuncion Gillego; (b) to their
knowledge Sy Mat died intestate; (c) they do not recognize Sy Kiat's marriage to Yao Kee nor
the filiation of her children to him; and, (d) they nominate Aida Sy-Gonzales for appointment as
administratrix of the intestate estate of the deceased [Record on Appeal, pp. 4-9; Rollo, p. 107.]
The petition was opposed by Yao Kee, Sze Sook Wah, Sze Lai Cho and Sy Yun Chen who
alleged that: (a) Yao Kee is the lawful wife of Sy Kiat whom he married on January 19, 1931 in
China; (b) the other oppositors are the legitimate children of the deceased with Yao Kee; and,
(c) Sze Sook Wah is the eldest among them and is competent, willing and desirous to become
the administratrix of the estate of Sy Kiat [Record on Appeal, pp. 12-13; Rollo, p. 107.] After
hearing, the probate court, finding among others that:

(1) Sy Kiat was legally married to Yao Kee [CFI decision, pp. 12-27; Rollo, pp.
49-64;]

(2) Sze Sook Wah, Sze Lai Cho and Sze Chun Yen are the legitimate children of
Yao Kee with Sy Mat [CFI decision, pp. 28-31; Rollo. pp. 65-68;] and,

(3) Aida Sy-Gonzales, Manuel Sy, Teresita Sy-Bernabe and Rodolfo Sy are the
acknowledged illegitimate offsprings of Sy Kiat with Asuncion Gillego [CFI
decision, pp. 27-28; Rollo, pp. 64- 65.]

held if favor of the oppositors (petitioners herein) and appointed Sze Sook Wah as the
administratrix of the intestate estate of the deceased [CFI decision, pp. 68-69; Rollo, pp. 105-
106.]

On appeal the Court of Appeals rendered a decision modifying that of the probate court, the
dispositive portion of which reads:

IN VIEW OF THE FOREGOING, the decision of the lower Court is hereby


MODIFIED and SET ASIDE and a new judgment rendered as follows:

(1) Declaring petitioners Aida Sy-Gonzales, Manuel Sy, Teresita Sy- Bernabe
and Rodolfo Sy acknowledged natural children of the deceased Sy Kiat with
Asuncion Gillego, an unmarried woman with whom he lived as husband and wife
without benefit of marriage for many years:

(2) Declaring oppositors Sze Sook Wah, Sze Lai Chu and Sze Chun Yen, the
acknowledged natural children of the deceased Sy Kiat with his Chinese wife
Yao Kee, also known as Yui Yip, since the legality of the alleged marriage of Sy
Mat to Yao Kee in China had not been proven to be valid to the laws of the
Chinese People's Republic of China (sic);

(3) Declaring the deed of sale executed by Sy Kiat on December 7, 1976 in favor
of Tomas Sy (Exhibit "G-1", English translation of Exhibit "G") of the Avenue
Tractor and Diesel Parts Supply to be valid and accordingly, said property should
be excluded from the estate of the deceased Sy Kiat; and

(4) Affirming the appointment by the lower court of Sze Sook Wah as judicial
administratrix of the estate of the deceased. [CA decision, pp. 11-12; Rollo, pp.
36- 37.]

From said decision both parties moved for partial reconsideration, which was however denied
by respondent court. They thus interposed their respective appeals to this Court.
Private respondents filed a petition with this Court docketed as G.R. No. 56045 entitled "Aida
Sy-Gonzales, Manuel Sy, Teresita Sy-Bernabe and Rodolfo Sy v. Court of Appeals, Yao Kee,
Sze Sook Wah, Sze Lai Cho and Sy Chun Yen" questioning paragraphs (3) and (4) of the
dispositive portion of the Court of Appeals' decision. The Supreme Court however resolved to
deny the petition and the motion for reconsideration. Thus on March 8, 1982 entry of judgment
was made in G.R. No. 56045. **

The instant petition, on the other hand, questions paragraphs (1) and (2) of the dispositive
portion of the decision of the Court of Appeals. This petition was initially denied by the Supreme
Court on June 22, 1981. Upon motion of the petitioners the Court in a resolution dated
September 16, 1981 reconsidered the denial and decided to give due course to this petition.
Herein petitioners assign the following as errors:

I. RESPONDENT COURT OF APPEALS SERIOUSLY ERRED IN DECLARING


THE MARRIAGE OF SY KIAT TO YAO YEE AS NOT HAVE (sic) BEEN
PROVEN VALID IN ACCORDANCE WITH LAWS OF THE PEOPLE'S
REPUBLIC OF CHINA.

II. RESPONDENT COURT OF APPEALS GRAVELY ERRED IN DECLARING


AIDA SY-GONZALES, MANUEL SY, TERESITA SY-BERNABE AND RODOLFO
SY AS NATURAL CHILDREN OF SY KIAT WITH ASUNCION GILLEGO.
[Petition, p. 2; Rollo, p. 6.]

I. Petitioners argue that the marriage of Sy Kiat to Yao Kee in accordance with Chinese law and
custom was conclusively proven. To buttress this argument they rely on the following testimonial
and documentary evidence.

First, the testimony of Yao Kee summarized by the trial court as follows:

Yao Kee testified that she was married to Sy Kiat on January 19, 1931 in
Fookien, China; that she does not have a marriage certificate because the
practice during that time was for elders to agree upon the betrothal of their
children, and in her case, her elder brother was the one who contracted or
entered into [an] agreement with the parents of her husband; that the agreement
was that she and Sy Mat would be married, the wedding date was set, and
invitations were sent out; that the said agreement was complied with; that she
has five children with Sy Kiat, but two of them died; that those who are alive are
Sze Sook Wah, Sze Lai Cho, and Sze Chun Yen, the eldest being Sze Sook
Wah who is already 38 years old; that Sze Sook Wah was born on November 7,
1939; that she and her husband, Sy Mat, have been living in FooKien, China
before he went to the Philippines on several occasions; that the practice during
the time of her marriage was a written document [is exchanged] just between the
parents of the bride and the parents of the groom, or any elder for that matter;
that in China, the custom is that there is a go- between, a sort of marriage broker
who is known to both parties who would talk to the parents of the bride-to-be; that
if the parents of the bride-to-be agree to have the groom-to-be their son in-law,
then they agree on a date as an engagement day; that on engagement day, the
parents of the groom would bring some pieces of jewelry to the parents of the
bride-to-be, and then one month after that, a date would be set for the wedding,
which in her case, the wedding date to Sy Kiat was set on January 19, 1931; that
during the wedding the bridegroom brings with him a couch (sic) where the bride
would ride and on that same day, the parents of the bride would give the dowry
for her daughter and then the document would be signed by the parties but there
is no solemnizing officer as is known in the Philippines; that during the wedding
day, the document is signed only by the parents of the bridegroom as well as by
the parents of the bride; that the parties themselves do not sign the document;
that the bride would then be placed in a carriage where she would be brought to
the town of the bridegroom and before departure the bride would be covered with
a sort of a veil; that upon reaching the town of the bridegroom, the bridegroom
takes away the veil; that during her wedding to Sy Kiat (according to said
Chinese custom), there were many persons present; that after Sy Kiat opened
the door of the carriage, two old ladies helped her go down the carriage and
brought her inside the house of Sy Mat; that during her wedding, Sy Chick, the
eldest brother of Sy Kiat, signed the document with her mother; that as to the
whereabouts of that document, she and Sy Mat were married for 46 years
already and the document was left in China and she doubt if that document can
still be found now; that it was left in the possession of Sy Kiat's family; that right
now, she does not know the whereabouts of that document because of the lapse
of many years and because they left it in a certain place and it was already eaten
by the termites; that after her wedding with Sy Kiat, they lived immediately
together as husband and wife, and from then on, they lived together; that Sy Kiat
went to the Philippines sometime in March or April in the same year they were
married; that she went to the Philippines in 1970, and then came back to China;
that again she went back to the Philippines and lived with Sy Mat as husband
and wife; that she begot her children with Sy Kiat during the several trips by Sy
Kiat made back to China. [CFI decision, pp. 13-15; Rollo, pp. 50-52.]

Second, the testimony of Gan Ching, a younger brother of Yao Kee who stated that he was
among the many people who attended the wedding of his sister with Sy Kiat and that no
marriage certificate is issued by the Chinese government, a document signed by the parents or
elders of the parties being sufficient [CFI decision, pp. 15-16; Rollo, pp.
52-53.]

Third, the statements made by Asuncion Gillego when she testified before the trial court to the
effect that (a) Sy Mat was married to Yao Kee according to Chinese custom; and, (b) Sy Kiat's
admission to her that he has a Chinese wife whom he married according to Chinese custom
[CFI decision, p. 17; Rollo, p. 54.]

Fourth, Sy Kiat's Master Card of Registered Alien issued in Caloocan City on October 3, 1972
where the following entries are found: "Marital status—Married"; "If married give name of
spouses—Yao Kee"; "Address-China; "Date of marriage—1931"; and "Place of marriage—
China" [Exhibit "SS-1".]

Fifth, Sy Kiat's Alien Certificate of Registration issued in Manila on January 12, 1968 where the
following entries are likewise found: "Civil status—Married"; and, 'If married, state name and
address of spouse—Yao Kee Chingkang, China" [Exhibit "4".]

And lastly, the certification issued in Manila on October 28, 1977 by the Embassy of the
People's Republic of China to the effect that "according to the information available at the
Embassy Mr. Sy Kiat a Chinese national and Mrs. Yao Kee alias Yui Yip also Chinese were
married on January 19, 1931 in Fukien, the People's Republic of China" [Exhibit "5".]

These evidence may very well prove the fact of marriage between Yao Kee and Sy Kiat.
However, the same do not suffice to establish the validity of said marriage in accordance with
Chinese law or custom.

Custom is defined as "a rule of conduct formed by repetition of acts, uniformly observed
(practiced) as a social rule, legally binding and obligatory" [In the Matter of the Petition for
Authority to Continue Use of the Firm Name "Ozaeta, Romulo, de Leon, Mabanta and Reyes",
July 30, 1979, SCRA 3, 12 citing JBL Reyes & RC Puno, Outline of Phil. Civil Law, Fourth Ed.,
Vol. 1, p. 7.] The law requires that "a custom must be proved as a fact, according to the rules of
evidence" [Article 12, Civil Code.] On this score the Court had occasion to state that "a local
custom as a source of right can not be considered by a court of justice unless such custom is
properly established by competent evidence like any other fact" [Patriarca v. Orate, 7 Phil. 390,
395 (1907).] The same evidence, if not one of a higher degree, should be required of a foreign
custom.

The law on foreign marriages is provided by Article 71 of the Civil Code which states that:

Art. 71. All marriages performed outside the Philippines in accordance with the
laws in force in the country where they were performed and valid there as such,
shall also be valid in this country, except bigamous, Polygamous, or incestuous
marriages, as determined by Philippine law. (Emphasis supplied.) ***

Construing this provision of law the Court has held that to establish a valid foreign marriage two
things must be proven, namely: (1) the existence of the foreign law as a question of fact; and (2)
the alleged foreign marriage by convincing evidence [Adong v. Cheong Seng Gee, 43 Phil. 43,
49 (1922).]

In proving a foreign law the procedure is provided in the Rules of Court. With respect to
an unwritten foreign law, Rule 130 section 45 states that:

SEC. 45. Unwritten law.—The oral testimony of witnesses, skilled therein, is


admissible as evidence of the unwritten law of a foreign country, as are also
printed and published books of reports of decisions of the courts of the foreign
country, if proved to be commonly admitted in such courts.

Proof of a written foreign law, on the other hand, is provided for under Rule 132 section 25,
thus:

SEC. 25. Proof of public or official record.—An official record or an entry therein,
when admissible for any purpose, may be evidenced by an official publication
thereof or by a copy attested by the officer having the legal custody of the record,
or by his deputy, and accompanied, if the record is not kept in the Philippines,
with a certificate that such officer has the custody. If the office in which the record
is kept is in a foreign country, the certificate may be made by a secretary of
embassy or legation, consul general, consul, vice consul, or consular agent or by
any officer in the foreign service of the Philippines stationed in the foreign country
in which the record is kept and authenticated by the seal of his office.
The Court has interpreted section 25 to include competent evidence like the testimony of a
witness to prove the existence of a written foreign law [Collector of Internal Revenue v. Fisher
110 Phil. 686, 700-701 (1961) citing Willamette Iron and Steel Works v. Muzzal, 61 Phil. 471
(1935).]

In the case at bar petitioners did not present any competent evidence relative to the law and
custom of China on marriage. The testimonies of Yao and Gan Ching cannot be considered as
proof of China's law or custom on marriage not only because they are
self-serving evidence, but more importantly, there is no showing that they are competent to
testify on the subject matter. For failure to prove the foreign law or custom, and consequently,
the validity of the marriage in accordance with said law or custom, the marriage between Yao
Kee and Sy Kiat cannot be recognized in this jurisdiction.

Petitioners contend that contrary to the Court of Appeals' ruling they are not duty bound to prove
the Chinese law on marriage as judicial notice thereof had been taken by this Court in the case
of Sy Joc Lieng v. Sy Quia [16 Phil. 137 (1910).]

This contention is erroneous. Well-established in this jurisdiction is the principle that Philippine
courts cannot take judicial notice of foreign laws. They must be alleged and proved as any other
fact [Yam Ka Lim v. Collector of Customs, 30 Phil. 46, 48 (1915); Fluemer v. Hix, 54 Phil. 610
(1930).]

Moreover a reading of said case would show that the party alleging the foreign marriage
presented a witness, one Li Ung Bieng, to prove that matrimonial letters mutually exchanged by
the contracting parties constitute the essential requisite for a marriage to be considered duly
solemnized in China. Based on his testimony, which as found by the Court is uniformly
corroborated by authors on the subject of Chinese marriage, what was left to be decided was
the issue of whether or not the fact of marriage in accordance with Chinese law was duly proven
[Sy Joc Lieng v. Sy Quia, supra., at p. 160.]

Further, even assuming for the sake of argument that the Court has indeed taken judicial notice
of the law of China on marriage in the aforecited case, petitioners however have not shown any
proof that the Chinese law or custom obtaining at the time the Sy Joc Lieng marriage was
celebrated in 1847 was still the law when the alleged marriage of Sy Kiat to Yao Kee took place
in 1931 or eighty-four (84) years later.

Petitioners moreover cite the case of U.S. v. Memoracion [34 Phil. 633 (1916)] as being
applicable to the instant case. They aver that the judicial pronouncement in the Memoracion
case, that the testimony of one of the contracting parties is competent evidence to show the fact
of marriage, holds true in this case.

The Memoracion case however is not applicable to the case at bar as said case did not concern
a foreign marriage and the issue posed was whether or not the oral testimony of a spouse is
competent evidence to prove the fact of marriage in a complaint for adultery.

Accordingly, in the absence of proof of the Chinese law on marriage, it should be presumed that
it is the same as ours *** [Wong Woo Yiu v. Vivo, G.R. No. L-21076, March 31, 1965, 13 SCRA
552, 555.] Since Yao Kee admitted in her testimony that there was no solemnizing officer as is
known here in the Philippines [See Article 56, Civil Code] when her alleged marriage to Sy Mat
was celebrated [CFI decision, p. 14; Rollo, p. 51], it therefore follows that her marriage to Sy
Kiat, even if true, cannot be recognized in this jurisdiction [Wong Woo Yiu v. Vivo, supra., pp.
555-556.]

II. The second issue raised by petitioners concerns the status of private respondents.

Respondent court found the following evidence of petitioners' filiation:

(1) Sy Kiat's Master Card of Registered Alien where the following are entered:
"Children if any: give number of children—Four"; and, "Name—All living in China"
[Exhibit "SS-1";]

(2) the testimony of their mother Yao Kee who stated that she had five children
with Sy Kiat, only three of whom are alive namely, Sze Sook Wah, Sze Lai Chu
and Sze Chin Yan [TSN, December 12, 1977, pp. 9-11;] and,

(3) an affidavit executed on March 22,1961 by Sy Kiat for presentation to the


Local Civil Registrar of Manila to support Sze Sook Wah's application for a
marriage license, wherein Sy Kiat expressly stated that she is his daughter
[Exhibit "3".]

Likewise on the record is the testimony of Asuncion Gillego that Sy Kiat told her he has three
daughters with his Chinese wife, two of whom—Sook Wah and Sze Kai Cho—she knows, and
one adopted son [TSN, December 6,1977, pp. 87-88.]

However, as petitioners failed to establish the marriage of Yao Kee with Sy Mat according to the
laws of China, they cannot be accorded the status of legitimate children but only that of
acknowledged natural children. Petitioners are natural children, it appearing that at the time of
their conception Yao Kee and Sy Kiat were not disqualified by any impediment to marry one
another [See Art. 269, Civil Code.] And they are acknowledged children of the deceased
because of Sy Kiat's recognition of Sze Sook Wah [Exhibit "3"] and its extension to Sze Lai Cho
and Sy Chun Yen who are her sisters of the full blood [See Art. 271, Civil Code.]

Private respondents on the other hand are also the deceased's acknowledged natural children
with Asuncion Gillego, a Filipina with whom he lived for twenty-five (25) years without the benefit
of marriage. They have in their favor their father's acknowledgment, evidenced by a
compromise agreement entered into by and between their parents and approved by the Court of
First Instance on February 12, 1974 wherein Sy Kiat not only acknowleged them as his children
by Asuncion Gillego but likewise made provisions for their support and future inheritance, thus:

xxx xxx xxx

2. The parties also acknowledge that they are common-law husband and wife
and that out of such relationship, which they have likewise decided to definitely
and finally terminate effective immediately, they begot five children, namely: Aida
Sy, born on May 30, 1950; Manuel Sy, born on July 1, 1953; Teresita Sy, born on
January 28, 1955; Ricardo Sy now deceased, born on December 14, 1956; and
Rodolfo Sy, born on May 7, 1958.

3. With respect to the AVENUE TRACTOR AND DIESEL PARTS SUPPLY ... ,
the parties mutually agree and covenant that—
(a) The stocks and merchandize and the furniture and equipments
..., shall be divided into two equal shares between, and distributed
to, Sy Kiat who shall own
one-half of the total and the other half to Asuncion Gillego who
shall transfer the same to their children, namely, Aida Sy, Manuel
Sy, Teresita Sy, and Rodolfo Sy.

(b) the business name and premises ... shall be retained by Sy


Kiat. However, it shall be his obligation to give to the aforenamed
children an amount of One Thousand Pesos ( Pl,000.00 ) monthly
out of the rental of the two doors of the same building now
occupied by Everett Construction.

xxx xxx xxx

(5) With respect to the acquisition, during the existence of the


common-law husband-and-wife relationship between the parties, of the real
estates and properties registered and/or appearing in the name of Asuncion
Gillego ... , the parties mutually agree and covenant that the said real estates and
properties shall be transferred in equal shares to their children, namely, Aida Sy,
Manuel Sy, Teresita Sy, and Rodolfo Sy, but to be administered by Asuncion
Gillego during her lifetime ... [Exhibit "D".] (Emphasis supplied.)

xxx xxx xxx

This compromise agreement constitutes a statement before a court of record by which a child
may be voluntarily acknowledged [See Art. 278, Civil Code.]

Petitioners further argue that the questions on the validity of Sy Mat's marriage to Yao Kee and
the paternity and filiation of the parties should have been ventilated in the Juvenile and
Domestic Relations Court.

Specifically, petitioners rely on the following provision of Republic Act No. 5502, entitled "An Act
Revising Rep. Act No. 3278, otherwise known as the Charter of the City of Caloocan', with
regard to the Juvenile and Domestic Relations Court:

SEC. 91-A. Creation and Jurisdiction of the Court.—

xxx xxx xxx

The provisions of the Judiciary Act to the contrary notwithstanding, the court shall
have exclusive original jurisdiction to hear and decide the following cases:

xxx xxx xxx

(2) Cases involving custody, guardianship, adoption, revocation of adoption,


paternity and acknowledgment;
(3) Annulment of marriages, relief from marital obligations, legal separation of
spouses, and actions for support;

(4) Proceedings brought under the provisions of title six and title seven, chapters
one to three of the civil code;

xxx xxx xxx

and the ruling in the case of Bartolome v. Bartolome [G.R. No. L-23661, 21 SCRA 1324]
reiterated in Divinagracia v. Rovira [G.R. No. L-42615, 72 SCRA 307.]

With the enactment of Batas Pambansa Blg. 129, otherwise known as the Judiciary
Reorganization Act of 1980, the Juvenile and Domestic Relations Courts were abolished. Their
functions and jurisdiction are now vested with the Regional Trial Courts [See Section 19 (7),
B.P. Blg. 129 and Divinagracia v. Belosillo, G.R. No. L-47407, August 12, 1986, 143 SCRA 356,
360] hence it is no longer necessary to pass upon the issue of jurisdiction raised by petitioners.

Moreover, even without the exactment of Batas Pambansa Blg. 129 we find in Rep. Act No.
5502 sec. 91-A last paragraph that:

xxx xxx xxx

If any question involving any of the above matters should arise as an incident in
any case pending in the ordinary court, said incident shall be determined in the
main case.

xxx xxx xxx

As held in the case of Divinagracia v. Rovira [G.R. No. L42615. August 10, 1976, 72 SCRA
307]:

xxx xxx xxx

It is true that under the aforequoted section 1 of Republic Act No.


4834 **** a case involving paternity and acknowledgment may be ventilated as
an incident in the intestate or testate proceeding (See Baluyot vs. Ines Luciano,
L-42215, July 13, 1976). But that legal provision presupposes that such an
administration proceeding is pending or existing and has not been terminated. [at
pp. 313-314.] (Emphasis supplied.)

xxx xxx xxx

The reason for ths rule is not only "to obviate the rendition of conflicting rulings on the same
issue by the Court of First Instance and the Juvenile and Domestic Relations Court" [Vda. de
Baluyut v. Luciano, G.R. No. L-42215, July 13, 1976, 72 SCRA 52, 63] but more importantly to
prevent multiplicity of suits. Accordingly, this Court finds no reversible error committed by
respondent court.

WHEREFORE, the decision of the Court of Appeals is hereby AFFIRMED.


SO ORDERED.

Fernan, C.J., Gutierrez, Jr., Feliciano and Bidin, JJ., concur.

Footnotes

* The petition for review in G.R. No. 56045 was denied for lack of merit on March
9, 1981, Counsel for the petitioners then filed a Motion for Consolidation and for
Extension of Time to File Motion for Reconsideration which was granted on July
8, 1981. On February 17, 1982, however, petitioners' motion for reconsideration
of the resolution of March 9, 1981 was denied.

** Other than the exceptions contained in this article, this provision of law is
derived from Section 19, Act No. 3613 and Section IV, General Order No. 68.

*** The presumption that, in the absence of proof, the foreign law is the same as
the law of the forum, is known as processual presumption which has been
applied by this Court in the cases of Lim v. The Insular Collector of Customs, 36
Phil, 472 (1917); International Harvester Co. in Russia v. Hamburg-American
Line, 42 Phil. 845 (1918); Miciano v. Brimo, 60 Phil, 867 (1924); and Rayray v.
Chae Kyung Lee, G.R. No. L-18176, October 26, 1966,18 SCRA 450.

**** Rep. Act 4834 created the Juvenile and Domestic Relations Court of Iloilo.
Section 1 of said Act is the exact copy of section 19-A of Rep. Act 5502.

REPUBLIC VS ORBECIDO

Digest:
REPUBLIC VS ORBECIDO

G.R. No. 154380

October 5, 2005

FACTS:

In 1981, Cipriano Orbecido III married Lady Myro Villanueva in Lam-an, Ozamis City.

In 1986, Orbecido discovered that his wife had had been naturalized as an American
citizen.
Sometime in 2000, Orbecido learned from his son that his wife had obtained a divorce
decree and married an American.

Orbecido filed with the Trial Court a petition for ―Authority to Remarry‖ invoking Article 26
Paragraph 2 of the Family Code, the Court granted the petition.

The Republic, herein petitioner, through the Office of the Solicitor General, sought for
reconsideration but it was denied by the Trial Court.

ISSUE: Whether or not the allegations of the respondent was proven as a fact according to the
rules of evidence.

HELD: Before a foreign divorce decree can be recognized by our own courts, the party pleading
it must prove the divorce as a fact and demonstrate its conformity to the foreign law allowing it.
Such foreign law must also be proved as our courts cannot take judicial notice of foreign laws.
Like any other fact, such laws must be alleged and proved. Furthermore, respondent must also
show that the divorce decree allows his former wife to remarry as specifically required in Article
26. Otherwise, there would be no evidence sufficient to declare that he is capacitated to enter
into another marriage.

However, in the present petition there is no sufficient evidence submitted and on record,
we are unable to declare, based on respondent‘s bare allegations that his wife, who was
naturalized as an American citizen, had obtained a divorce decree and had remarried an
American, that respondent is now capacitated to remarry. Such declaration could only be made
properly upon respondent‘s submission of the aforecited evidence in his favor.

ACCORDINGLY, the petition by the Republic of the Philippines is GRANTED. The


assailed Decision dated May 15, 2002, and Resolution dated July 4, 2002, of the Regional Trial
Court of Molave, Zamboanga del Sur, Branch 23, are hereby SET ASIDE.

(Long version – Dito may explanation yung rulling)

REPUBLIC VS ORBECIDO

G.R. No. 154380


October 5, 2005

FACTS:

In 1981, Cipriano Orbecido III married Lady Myro Villanueva in Lam-an, Ozamis City.

In 1986, Orbecido discovered that his wife had had been naturalized as an American
citizen.

Sometime in 2000, Orbecido learned from his son that his wife had obtained a divorce
decree and married an American.

Orbecido filed with the Trial Court a petition for ―Authority to Remarry‖ invoking Article 26
Paragraph 2 of the Family Code, the Court granted the petition.

The Republic, herein petitioner, through the Office of the Solicitor General, sought for
reconsideration but it was denied by the Trial Court.

ISSUE: Whether or not the allegations of the respondent was proven as a fact according to the
rules of evidence.

HELD: In the present petition there is no sufficient evidence submitted and on record, we
are unable to declare, based on respondent‘s bare allegations that his wife, who was
naturalized as an American citizen, had obtained a divorce decree and had remarried an
American, that respondent is now capacitated to remarry. Such declaration could only be made
properly upon respondent‘s submission of the aforecited evidence in his favor.

ACCORDINGLY, the petition by the Republic of the Philippines is GRANTED. The


assailed Decision dated May 15, 2002, and Resolution dated July 4, 2002, of the Regional Trial
Court of Molave, Zamboanga del Sur, Branch 23, are hereby SET ASIDE.

(Short version – Dito rulling na mismo walang explanation)

FIRST DIVISION

REPUBLIC OF THE PHILIPPINES, G.R. No. 154380


Petitioner,
Present:

Davide, Jr., C.J.,


- versus - (Chairman),
Quisumbing,
Ynares-Santiago,
Carpio, and
Azcuna, JJ.
CIPRIANO ORBECIDO III,
Respondent. Promulgated:

October 5, 2005

x --------------------------------------------------x

DECISION

QUISUMBING, J.:

Given a valid marriage between two Filipino citizens, where one party is later naturalized

as a foreign citizen and obtains a valid divorce decree capacitating him or her to remarry, can

the Filipino spouse likewise remarry under Philippine law?

Before us is a case of first impression that behooves the Court to make a definite ruling

on this apparently novel question, presented as a pure question of law.

In this petition for review, the Solicitor General assails the Decision[1] dated May 15,

2002, of the Regional Trial Court of Molave, Zamboanga del Sur, Branch 23 and

its Resolution[2] dated July 4, 2002 denying the motion for reconsideration. The courta quo had

declared that herein respondent Cipriano Orbecido III is capacitated to remarry. The fallo of the

impugned Decision reads:


WHEREFORE, by virtue of the provision of the second paragraph of Art.
26 of the Family Code and by reason of the divorce decree obtained against him
by his American wife, the petitioner is given the capacity to remarry under the
Philippine Law.
IT IS SO ORDERED.[3]

The factual antecedents, as narrated by the trial court, are as follows.

On May 24, 1981, Cipriano Orbecido III married Lady Myros M. Villanueva at the United

Church of Christ in the Philippines in Lam-an, Ozamis City. Their marriage was blessed with a

son and a daughter, Kristoffer Simbortriz V. Orbecido and Lady Kimberly V. Orbecido.

In 1986, Cipriano‘s wife left for the United States bringing along their son Kristoffer. A

few years later, Cipriano discovered that his wife had been naturalized as an American citizen.

Sometime in 2000, Cipriano learned from his son that his wife had obtained a divorce

decree and then married a certain Innocent Stanley. She, Stanley and her child by him

currently live at 5566 A. Walnut Grove Avenue, San Gabriel, California.

Cipriano thereafter filed with the trial court a petition for authority to remarry invoking

Paragraph 2 of Article 26 of the Family Code. No opposition was filed. Finding merit in the

petition, the court granted the same. The Republic, herein petitioner, through the Office of the

Solicitor General (OSG), sought reconsideration but it was denied.

In this petition, the OSG raises a pure question of law:


WHETHER OR NOT RESPONDENT CAN REMARRY UNDER ARTICLE 26 OF
THE FAMILY CODE[4]

The OSG contends that Paragraph 2 of Article 26 of the Family Code is not applicable to

the instant case because it only applies to a valid mixed marriage; that is, a marriage celebrated

between a Filipino citizen and an alien. The proper remedy, according to the OSG, is to file a
petition for annulment or for legal separation. [5] Furthermore, the OSG argues there is no law

that governs respondent‘s situation. The OSG posits that this is a matter of legislation and not

of judicial determination.[6]

For his part, respondent admits that Article 26 is not directly applicable to his case but

insists that when his naturalized alien wife obtained a divorce decree which capacitated her to

remarry, he is likewise capacitated by operation of law pursuant to Section 12, Article II of the

Constitution.[7]

At the outset, we note that the petition for authority to remarry filed before the trial court

actually constituted a petition for declaratory relief. In this connection, Section 1, Rule 63 of the

Rules of Court provides:


RULE 63
DECLARATORY RELIEF AND SIMILAR REMEDIES

Section 1. Who may file petition—Any person interested under a deed, will,
contract or other written instrument, or whose rights are affected by a statute,
executive order or regulation, ordinance, or other governmental regulation may,
before breach or violation thereof, bring an action in the appropriate Regional
Trial Court to determine any question of construction or validity arising, and for a
declaration of his rights or duties, thereunder.
...

The requisites of a petition for declaratory relief are: (1) there must be a justiciable

controversy; (2) the controversy must be between persons whose interests are adverse; (3) that

the party seeking the relief has a legal interest in the controversy; and (4) that the issue is ripe

for judicial determination.[8]

This case concerns the applicability of Paragraph 2 of Article 26 to a marriage between

two Filipino citizens where one later acquired alien citizenship, obtained a divorce decree, and
remarried while in the U.S.A. The interests of the parties are also adverse, as petitioner

representing the State asserts its duty to protect the institution of marriage while respondent, a

private citizen, insists on a declaration of his capacity to remarry. Respondent, praying for relief,

has legal interest in the controversy. The issue raised is also ripe for judicial determination

inasmuch as when respondent remarries, litigation ensues and puts into question the validity of

his second marriage.

Coming now to the substantive issue, does Paragraph 2 of Article 26 of the Family Code

apply to the case of respondent? Necessarily, we must dwell on how this provision had come

about in the first place, and what was the intent of the legislators in its enactment?

Brief Historical Background

On July 6, 1987, then President Corazon Aquino signed into law Executive Order No.

209, otherwise known as the ―Family Code,‖ which took effect on August 3, 1988. Article 26

thereof states:
All marriages solemnized outside the Philippines in accordance with the
laws in force in the country where they were solemnized, and valid there as such,
shall also be valid in this country, except those prohibited under Articles 35, 37,
and 38.

On July 17, 1987, shortly after the signing of the original Family Code, Executive Order

No. 227 was likewise signed into law, amending Articles 26, 36, and 39 of the Family Code. A

second paragraph was added to Article 26. As so amended, it now provides:


ART. 26. All marriages solemnized outside the Philippines in accordance
with the laws in force in the country where they were solemnized, and valid there
as such, shall also be valid in this country, except those prohibited under Articles
35(1), (4), (5) and (6), 36, 37 and 38.
Where a marriage between a Filipino citizen and a foreigner is validly
celebrated and a divorce is thereafter validly obtained abroad by the alien spouse
capacitating him or her to remarry, the Filipino spouse shall have capacity to
remarry under Philippine law. (Emphasis supplied)

On its face, the foregoing provision does not appear to govern the situation presented by

the case at hand. It seems to apply only to cases where at the time of the celebration of the

marriage, the parties are a Filipino citizen and a foreigner. The instant case is one where at the

time the marriage was solemnized, the parties were two Filipino citizens, but later on, the wife

was naturalized as an American citizen and subsequently obtained a divorce granting her

capacity to remarry, and indeed she remarried an American citizen while residing in the U.S.A.

Noteworthy, in the Report of the Public Hearings [9] on the Family Code, the Catholic

Bishops‘ Conference of the Philippines (CBCP) registered the following objections to Paragraph

2 of Article 26:
1. The rule is discriminatory. It discriminates against those whose
spouses are Filipinos who divorce them abroad. These spouses who are
divorced will not be able to re-marry, while the spouses of foreigners who
validly divorce them abroad can.
2. This is the beginning of the recognition of the validity of divorce
even for Filipino citizens. For those whose foreign spouses validly
divorce them abroad will also be considered to be validly divorced here
and can re-marry. We propose that this be deleted and made into law
only after more widespread consultation. (Emphasis supplied.)

Legislative Intent

Records of the proceedings of the Family Code deliberations showed that the intent of

Paragraph 2 of Article 26, according to Judge Alicia Sempio-Diy, a member of the Civil Code

Revision Committee, is to avoid the absurd situation where the Filipino spouse remains married

to the alien spouse who, after obtaining a divorce, is no longer married to the Filipino spouse.
Interestingly, Paragraph 2 of Article 26 traces its origin to the 1985 case of Van Dorn v.

Romillo, Jr.[10] The Van Dorn case involved a marriage between a Filipino citizen and a

foreigner. The Court held therein that a divorce decree validly obtained by the alien spouse is

valid in the Philippines, and consequently, the Filipino spouse is capacitated to remarry under

Philippine law.

Does the same principle apply to a case where at the time of the celebration of the

marriage, the parties were Filipino citizens, but later on, one of them obtains a foreign

citizenship by naturalization?

The jurisprudential answer lies latent in the 1998 case of Quita v. Court of

Appeals.[11] In Quita, the parties were, as in this case, Filipino citizens when they got married.

The wife became a naturalized American citizen in 1954 and obtained a divorce in the same

year. The Court therein hinted, by way of obiter dictum, that a Filipino divorced by his

naturalized foreign spouse is no longer married under Philippine law and can thus remarry.

Thus, taking into consideration the legislative intent and applying the rule of reason, we

hold that Paragraph 2 of Article 26 should be interpreted to include cases involving parties who,

at the time of the celebration of the marriage were Filipino citizens, but later on, one of them

becomes naturalized as a foreign citizen and obtains a divorce decree. The Filipino spouse

should likewise be allowed to remarry as if the other party were a foreigner at the time of the

solemnization of the marriage. To rule otherwise would be to sanction absurdity and injustice.

Where the interpretation of a statute according to its exact and literal import would lead to

mischievous results or contravene the clear purpose of the legislature, it should be construed
according to its spirit and reason, disregarding as far as necessary the letter of the law. A

statute may therefore be extended to cases not within the literal meaning of its terms, so long as

they come within its spirit or intent. [12]

If we are to give meaning to the legislative intent to avoid the absurd situation where the

Filipino spouse remains married to the alien spouse who, after obtaining a divorce is no longer

married to the Filipino spouse, then the instant case must be deemed as coming within the

contemplation of Paragraph 2 of Article 26.

In view of the foregoing, we state the twin elements for the application of Paragraph 2 of

Article 26 as follows:

1. There is a valid marriage that has been celebrated between a


Filipino citizen and a foreigner; and

2. A valid divorce is obtained abroad by the alien spouse capacitating


him or her to remarry.

The reckoning point is not the citizenship of the parties at the time of the celebration of the

marriage, but their citizenship at the time a valid divorce is obtained abroad by the alien spouse

capacitating the latter to remarry.

In this case, when Cipriano‘s wife was naturalized as an American citizen, there was still

a valid marriage that has been celebrated between her and Cipriano. As fate would have it, the

naturalized alien wife subsequently obtained a valid divorce capacitating her to remarry.

Clearly, the twin requisites for the application of Paragraph 2 of Article 26 are both present in

this case. Thus Cipriano, the ―divorced‖ Filipino spouse, should be allowed to remarry.
We are also unable to sustain the OSG‘s theory that the proper remedy of the Filipino

spouse is to file either a petition for annulment or a petition for legal separation. Annulment

would be a long and tedious process, and in this particular case, not even feasible, considering

that the marriage of the parties appears to have all the badges of validity. On the other hand,

legal separation would not be a sufficient remedy for it would not sever the marriage tie; hence,

the legally separated Filipino spouse would still remain married to the naturalized alien spouse.

However, we note that the records are bereft of competent evidence duly submitted by

respondent concerning the divorce decree and the naturalization of respondent‘s wife. It is

settled rule that one who alleges a fact has the burden of proving it and mere allegation is not

evidence.[13]

Accordingly, for his plea to prosper, respondent herein must prove his allegation that

his wife was naturalized as an American citizen. Likewise, before a foreign divorce decree can

be recognized by our own courts, the party pleading it must prove the divorce as a fact and

demonstrate its conformity to the foreign law allowing it. [14] Such foreign law must also be

proved as our courts cannot take judicial notice of foreign laws. Like any other fact, such laws

must be alleged and proved.[15] Furthermore, respondent must also show that the divorce

decree allows his former wife to remarry as specifically required in Article 26. Otherwise, there

would be no evidence sufficient to declare that he is capacitated to enter into another marriage.

Nevertheless, we are unanimous in our holding that Paragraph 2 of Article 26 of the

Family Code (E.O. No. 209, as amended by E.O. No. 227), should be interpreted to allow a

Filipino citizen, who has been divorced by a spouse who had acquired foreign citizenship and
remarried, also to remarry. However, considering that in the present petition there is no

sufficient evidence submitted and on record, we are unable to declare, based on respondent‘s

bare allegations that his wife, who was naturalized as an American citizen, had obtained a

divorce decree and had remarried an American, that respondent is now capacitated to remarry.

Such declaration could only be made properly upon respondent‘s submission of the aforecited

evidence in his favor.

ACCORDINGLY, the petition by the Republic of the Philippines is GRANTED. The

assailed Decision dated May 15, 2002, and Resolution dated July 4, 2002, of the Regional Trial

Court of Molave, Zamboanga del Sur, Branch 23, are hereby SET ASIDE.

No pronouncement as to costs.

SO ORDERED.

LEONARDO A. QUISUMBING
Associate Justice

WE CONCUR:

HILARIO G. DAVIDE, JR.


Chief Justice
Chairman

CONSUELO YNARES-SANTIAGO ANTONIO T. CARPIO


Associate Justice Associate Justice

ADOLFO S. AZCUNA
Associate Justice

CERTIFICATION

Pursuant to Section 13, Article VIII of the Constitution, it is hereby certified that the
conclusions in the above Decision were reached in consultation before the case was assigned
to the writer of the opinion of the Court‘s Division.

HILARIO G. DAVIDE, JR.


Chief Justice

[1]
Rollo, pp. 20-22.
[2]
Id. at 27-29.
[3]
Id. at 21-22.
[4]
Id. at 105.
[5]
Id. at 106-110.
[6]
Id. at 110.
[7]
Sec. 12. The State recognizes the sanctity of family life and shall protect and strengthen
the family as a basic autonomous social institution. It shall equally protect the life of the
mother and the life of the unborn from conception. The natural and primary right and duty of
parents in the rearing of the youth for civic efficiency and the development of moral
character shall receive the support of the Government.
[8]
Office of the Ombudsman v. Ibay, G.R. No. 137538, 3 September 2001, 364 SCRA 281,
286, citing Galarosa v. Valencia, G.R. No. 109455, 11 November 1993, 227 SCRA
729, 737.
[9]
Held on January 27 and 28, 1988 and February 3, 1988.
[10]
No. L-68470, 8 October 1985, 139 SCRA 139.
[11]
G.R. No. 124862, 22 December 1998, 300 SCRA 406.
[12]
Lopez & Sons, Inc. v. Court of Tax Appeals, No. L-9274, 1 February 1957, 100 Phil. 850,
855.
[13]
Cortes v. Court of Appeals, G.R. No. 121772, 13 January 2003, 395 SCRA 33, 38.
[14]
Garcia v. Recio, G.R. No. 138322, 2 October 2001, 366 SCRA 437, 447.
[15]
Id. at 451.

Essential and Formal Requisites: Art. 2 and 3, Family Code


See Art. 5 and 35(2).
For consequences, See Art. 4 and 35 (2). Also Muslim Code, Art. 15

VI. Consequences of Marriage

1. Personal Relations
2. Property Relations
COLLECTOR VS FISHER

Republic of the Philippines


SUPREME COURT
Manila

EN BANC

G.R. No. L-11622 January 28, 1961

THE COLLECTOR OF INTERNAL REVENUE, petitioner,


vs.
DOUGLAS FISHER AND BETTINA FISHER, and the COURT OF TAX
APPEALS, respondents.

x---------------------------------------------------------x

G.R. No. L-11668 January 28, 1961.

DOUGLAS FISHER AND BETTINA FISHER, petitioner,


vs.
THE COLLECTOR OF INTERNAL REVENUE, and the COURT OF TAX
APPEALS, respondents.

BARRERA, J.:

This case relates to the determination and settlement of the hereditary estate left by the
deceased Walter G. Stevenson, and the laws applicable thereto. Walter G. Stevenson (born in
the Philippines on August 9, 1874 of British parents and married in the City of Manila on
January 23, 1909 to Beatrice Mauricia Stevenson another British subject) died on February 22,
1951 in San Francisco, California, U.S.A. whereto he and his wife moved and established their
permanent residence since May 10, 1945. In his will executed in San Francisco on May 22,
1947, and which was duly probated in the Superior Court of California on April 11, 1951,
Stevenson instituted his wife Beatrice as his sole heiress to the following real and personal
properties acquired by the spouses while residing in the Philippines, described and preliminary
assessed as follows:

Gross Estate
Real Property — 2 parcels of land in
Baguio, covered by T.C.T. Nos. 378 and
379 P43,500.00
Personal Property
(1) 177 shares of stock of Canacao
Estate at P10.00 each 1,770.00
(2) 210,000 shares of stock of Mindanao
Mother Lode Mines, Inc. at P0.38 per
share 79,800.00
(3) Cash credit with Canacao Estate Inc. 4,870.88
(4) Cash, with the Chartered Bank of
India, Australia & China 851.97
Total Gross Assets P130,792.85

On May 22, 1951, ancillary administration proceedings were instituted in the Court of First
Instance of Manila for the settlement of the estate in the Philippines. In due time Stevenson's
will was duly admitted to probate by our court and Ian Murray Statt was appointed ancillary
administrator of the estate, who on July 11, 1951, filed a preliminary estate and inheritance tax
return with the reservation of having the properties declared therein finally appraised at their
values six months after the death of Stevenson. Preliminary return was made by the ancillary
administrator in order to secure the waiver of the Collector of Internal Revenue on the
inheritance tax due on the 210,000 shares of stock in the Mindanao Mother Lode Mines Inc.
which the estate then desired to dispose in the United States. Acting upon said return, the
Collector of Internal Revenue accepted the valuation of the personal properties declared
therein, but increased the appraisal of the two parcels of land located in Baguio City by fixing
their fair market value in the amount of P52.200.00, instead of P43,500.00. After allowing the
deductions claimed by the ancillary administrator for funeral expenses in the amount of
P2,000.00 and for judicial and administration expenses in the sum of P5,500.00, the Collector
assessed the state the amount of P5,147.98 for estate tax and P10,875,26 or inheritance tax, or
a total of P16,023.23. Both of these assessments were paid by the estate on June 6, 1952.

On September 27, 1952, the ancillary administrator filed in amended estate and inheritance tax
return in pursuance f his reservation made at the time of filing of the preliminary return and for
the purpose of availing of the right granted by section 91 of the National Internal Revenue Code.

In this amended return the valuation of the 210,000 shares of stock in the Mindanao Mother
Lode Mines, Inc. was reduced from 0.38 per share, as originally declared, to P0.20 per share, or
from a total valuation of P79,800.00 to P42,000.00. This change in price per share of stock was
based by the ancillary administrator on the market notation of the stock obtaining at the San
Francisco California) Stock Exchange six months from the death of Stevenson, that is, As of
August 22, 1931. In addition, the ancillary administrator made claim for the following deductions:

Funeral expenses ($1,04326) P2,086.52


Judicial Expenses:
(a) Administrator's Fee P1,204.34
(b) Attorney's Fee 6.000.00
(c) Judicial and Administration
expenses as of August 9, 1952 1,400.05
8,604.39
Real Estate Tax for 1951 on
Baguio real properties (O.R. No.
B-1 686836) 652.50
Claims against the estate:
($5,000.00) P10,000.00 P10,000.00
Plus: 4% int. p.a. from Feb. 2 to 22.47 10,022.47
22, 1951
Sub-Total P21,365.88

In the meantime, on December 1, 1952, Beatrice Mauricia Stevenson assigned all her rights
and interests in the estate to the spouses, Douglas and Bettina Fisher, respondents herein.

On September 7, 1953, the ancillary administrator filed a second amended estate and
inheritance tax return (Exh. "M-N"). This return declared the same assets of the estate stated in
the amended return of September 22, 1952, except that it contained new claims for additional
exemption and deduction to wit: (1) deduction in the amount of P4,000.00 from the gross estate
of the decedent as provided for in Section 861 (4) of the U.S. Federal Internal Revenue Code
which the ancillary administrator averred was allowable by way of the reciprocity granted by
Section 122 of the National Internal Revenue Code, as then held by the Board of Tax Appeals in
case No. 71 entitled "Housman vs. Collector," August 14, 1952; and (2) exemption from the
imposition of estate and inheritance taxes on the 210,000 shares of stock in the Mindanao
Mother Lode Mines, Inc. also pursuant to the reciprocity proviso of Section 122 of the National
Internal Revenue Code. In this last return, the estate claimed that it was liable only for the
amount of P525.34 for estate tax and P238.06 for inheritance tax and that, as a consequence, it
had overpaid the government. The refund of the amount of P15,259.83, allegedly overpaid, was
accordingly requested by the estate. The Collector denied the claim. For this reason, action was
commenced in the Court of First Instance of Manila by respondents, as assignees of Beatrice
Mauricia Stevenson, for the recovery of said amount. Pursuant to Republic Act No. 1125, the
case was forwarded to the Court of Tax Appeals which court, after hearing, rendered decision
the dispositive portion of which reads as follows:

In fine, we are of the opinion and so hold that: (a) the one-half (½) share of the surviving
spouse in the conjugal partnership property as diminished by the obligations properly
chargeable to such property should be deducted from the net estate of the deceased
Walter G. Stevenson, pursuant to Section 89-C of the National Internal Revenue Code;
(b) the intangible personal property belonging to the estate of said Stevenson is exempt
from inheritance tax, pursuant to the provision of section 122 of the National Internal
Revenue Code in relation to the California Inheritance Tax Law but decedent's estate is
not entitled to an exemption of P4,000.00 in the computation of the estate tax; (c) for
purposes of estate and inheritance taxation the Baguio real estate of the spouses should
be valued at P52,200.00, and 210,000 shares of stock in the Mindanao Mother Lode
Mines, Inc. should be appraised at P0.38 per share; and (d) the estate shall be entitled
to a deduction of P2,000.00 for funeral expenses and judicial expenses of P8,604.39.

From this decision, both parties appealed.

The Collector of Internal Revenue, hereinafter called petitioner assigned four errors allegedly
committed by the trial court, while the assignees, Douglas and Bettina Fisher hereinafter called
respondents, made six assignments of error. Together, the assigned errors raise the following
main issues for resolution by this Court:

(1) Whether or not, in determining the taxable net estate of the decedent, one-half (½) of the net
estate should be deducted therefrom as the share of tile surviving spouse in accordance with
our law on conjugal partnership and in relation to section 89 (c) of the National Internal revenue
Code;
(2) Whether or not the estate can avail itself of the reciprocity proviso embodied in Section 122
of the National Internal Revenue Code granting exemption from the payment of estate and
inheritance taxes on the 210,000 shares of stock in the Mindanao Mother Lode Mines Inc.;

(3) Whether or not the estate is entitled to the deduction of P4,000.00 allowed by Section 861,
U.S. Internal Revenue Code in relation to section 122 of the National Internal Revenue Code;

(4) Whether or not the real estate properties of the decedent located in Baguio City and the
210,000 shares of stock in the Mindanao Mother Lode Mines, Inc., were correctly appraised by
the lower court;

(5) Whether or not the estate is entitled to the following deductions: P8,604.39 for judicial and
administration expenses; P2,086.52 for funeral expenses; P652.50 for real estate taxes; and
P10,0,22.47 representing the amount of indebtedness allegedly incurred by the decedent during
his lifetime; and

(6) Whether or not the estate is entitled to the payment of interest on the amount it claims to
have overpaid the government and to be refundable to it.

In deciding the first issue, the lower court applied a well-known doctrine in our civil law that in
the absence of any ante-nuptial agreement, the contracting parties are presumed to have
adopted the system of conjugal partnership as to the properties acquired during their marriage.
The application of this doctrine to the instant case is being disputed, however, by petitioner
Collector of Internal Revenue, who contends that pursuant to Article 124 of the New Civil Code,
the property relation of the spouses Stevensons ought not to be determined by the Philippine
law, but by the national law of the decedent husband, in this case, the law of England. It is
alleged by petitioner that English laws do not recognize legal partnership between spouses, and
that what obtains in that jurisdiction is another regime of property relation, wherein all properties
acquired during the marriage pertain and belong Exclusively to the husband. In further support
of his stand, petitioner cites Article 16 of the New Civil Code (Art. 10 of the old) to the effect that
in testate and intestate proceedings, the amount of successional rights, among others, is to be
determined by the national law of the decedent.

In this connection, let it be noted that since the mariage of the Stevensons in the Philippines
took place in 1909, the applicable law is Article 1325 of the old Civil Code and not Article 124 of
the New Civil Code which became effective only in 1950. It is true that both articles adhere to
the so-called nationality theory of determining the property relation of spouses where one of
them is a foreigner and they have made no prior agreement as to the administration disposition,
and ownership of their conjugal properties. In such a case, the national law of the husband
becomes the dominant law in determining the property relation of the spouses. There is,
however, a difference between the two articles in that Article 124 1 of the new Civil Code
expressly provides that it shall be applicable regardless of whether the marriage was celebrated
in the Philippines or abroad while Article 1325 2 of the old Civil Code is limited to marriages
contracted in a foreign land.

It must be noted, however, that what has just been said refers to mixed marriages between a
Filipino citizen and a foreigner. In the instant case, both spouses are foreigners who married in
the Philippines. Manresa,3 in his Commentaries, has this to say on this point:
La regla establecida en el art. 1.315, se refiere a las capitulaciones otorgadas en
Espana y entre espanoles. El 1.325, a las celebradas en el extranjero cuando alguno de
los conyuges es espanol. En cuanto a la regla procedente cuando dos extranjeros se
casan en Espana, o dos espanoles en el extranjero hay que atender en el primer caso a
la legislacion de pais a que aquellos pertenezean, y en el segundo, a las reglas
generales consignadas en los articulos 9 y 10 de nuestro Codigo. (Emphasis supplied.)

If we adopt the view of Manresa, the law determinative of the property relation of the
Stevensons, married in 1909, would be the English law even if the marriage was celebrated in
the Philippines, both of them being foreigners. But, as correctly observed by the Tax Court, the
pertinent English law that allegedly vests in the decedent husband full ownership of the
properties acquired during the marriage has not been proven by petitioner. Except for a mere
allegation in his answer, which is not sufficient, the record is bereft of any evidence as to what
English law says on the matter. In the absence of proof, the Court is justified, therefore, in
indulging in what Wharton calls "processual presumption," in presuming that the law of England
on this matter is the same as our law. 4

Nor do we believe petitioner can make use of Article 16 of the New Civil Code (art. 10, old Civil
Code) to bolster his stand. A reading of Article 10 of the old Civil Code, which incidentally is the
one applicable, shows that it does not encompass or contemplate to govern the question of
property relation between spouses. Said article distinctly speaks of amount of successional
rights and this term, in speaks in our opinion, properly refers to the extent or amount of property
that each heir is legally entitled to inherit from the estate available for distribution. It needs to be
pointed out that the property relation of spouses, as distinguished from their successional rights,
is governed differently by the specific and express provisions of Title VI, Chapter I of our new
Civil Code (Title III, Chapter I of the old Civil Code.) We, therefore, find that the lower court
correctly deducted the half of the conjugal property in determining the hereditary estate left by
the deceased Stevenson.

On the second issue, petitioner disputes the action of the Tax Court in the exempting the
respondents from paying inheritance tax on the 210,000 shares of stock in the Mindanao Mother
Lode Mines, Inc. in virtue of the reciprocity proviso of Section 122 of the National Internal
Revenue Code, in relation to Section 13851 of the California Revenue and Taxation Code, on
the ground that: (1) the said proviso of the California Revenue and Taxation Code has not been
duly proven by the respondents; (2) the reciprocity exemptions granted by section 122 of the
National Internal Revenue Code can only be availed of by residents of foreign countries and not
of residents of a state in the United States; and (3) there is no "total" reciprocity between the
Philippines and the state of California in that while the former exempts payment of both estate
and inheritance taxes on intangible personal properties, the latter only exempts the payment of
inheritance tax..

To prove the pertinent California law, Attorney Allison Gibbs, counsel for herein respondents,
testified that as an active member of the California Bar since 1931, he is familiar with the
revenue and taxation laws of the State of California. When asked by the lower court to state the
pertinent California law as regards exemption of intangible personal properties, the witness cited
article 4, section 13851 (a) and (b) of the California Internal and Revenue Code as published in
Derring's California Code, a publication of the Bancroft-Whitney Company inc. And as part of his
testimony, a full quotation of the cited section was offered in evidence as Exhibits "V-2" by the
respondents.
It is well-settled that foreign laws do not prove themselves in our jurisdiction and our courts are
not authorized to take judicial notice of them. 5 Like any other fact, they must be alleged and
proved.6

Section 41, Rule 123 of our Rules of Court prescribes the manner of proving foreign laws before
our tribunals. However, although we believe it desirable that these laws be proved in
accordance with said rule, we held in the case of Willamette Iron and Steel Works v. Muzzal, 61
Phil. 471, that "a reading of sections 300 and 301 of our Code of Civil Procedure (now section
41, Rule 123) will convince one that these sections do not exclude the presentation of other
competent evidence to prove the existence of a foreign law." In that case, we considered the
testimony of an attorney-at-law of San Francisco, California who quoted verbatim a section of
California Civil Code and who stated that the same was in force at the time the obligations were
contracted, as sufficient evidence to establish the existence of said law. In line with this view, we
find no error, therefore, on the part of the Tax Court in considering the pertinent California law
as proved by respondents' witness.

We now take up the question of reciprocity in exemption from transfer or death taxes, between
the State of California and the Philippines.F

Section 122 of our National Internal Revenue Code, in pertinent part, provides:

... And, provided, further, That no tax shall be collected under this Title in respect of
intangible personal property (a) if the decedent at the time of his death was a resident of
a foreign country which at the time of his death did not impose a transfer of tax or death
tax of any character in respect of intangible personal property of citizens of the
Philippines not residing in that foreign country, or (b) if the laws of the foreign country of
which the decedent was a resident at the time of his death allow a similar exemption
from transfer taxes or death taxes of every character in respect of intangible personal
property owned by citizens of the Philippines not residing in that foreign country."
(Emphasis supplied).

On the other hand, Section 13851 of the California Inheritance Tax Law, insofar as pertinent,
reads:.

"SEC. 13851, Intangibles of nonresident: Conditions. Intangible personal property is


exempt from the tax imposed by this part if the decedent at the time of his death was a
resident of a territory or another State of the United States or of a foreign state or
country which then imposed a legacy, succession, or death tax in respect to intangible
personal property of its own residents, but either:.

(a) Did not impose a legacy, succession, or death tax of any character in respect to
intangible personal property of residents of this State, or

(b) Had in its laws a reciprocal provision under which intangible personal property of a
non-resident was exempt from legacy, succession, or death taxes of every character if
the Territory or other State of the United States or foreign state or country in which the
nonresident resided allowed a similar exemption in respect to intangible personal
property of residents of the Territory or State of the United States or foreign state or
country of residence of the decedent." (Id.)
It is clear from both these quoted provisions that the reciprocity must be total, that is, with
respect to transfer or death taxes of any and every character, in the case of the Philippine law,
and to legacy, succession, or death taxes of any and every character, in the case of the
California law. Therefore, if any of the two states collects or imposes and does not exempt any
transfer, death, legacy, or succession tax of any character, the reciprocity does not work. This is
the underlying principle of the reciprocity clauses in both laws.

In the Philippines, upon the death of any citizen or resident, or non-resident with properties
therein, there are imposed upon his estate and its settlement, both an estate and an inheritance
tax. Under the laws of California, only inheritance tax is imposed. On the other hand, the
Federal Internal Revenue Code imposes an estate tax on non-residents not citizens of the
United States,7 but does not provide for any exemption on the basis of reciprocity. Applying
these laws in the manner the Court of Tax Appeals did in the instant case, we will have a
situation where a Californian, who is non-resident in the Philippines but has intangible personal
properties here, will the subject to the payment of an estate tax, although exempt from the
payment of the inheritance tax. This being the case, will a Filipino, non-resident of California, but
with intangible personal properties there, be entitled to the exemption clause of the California
law, since the Californian has not been exempted from every character of legacy, succession, or
death tax because he is, under our law, under obligation to pay an estate tax? Upon the other
hand, if we exempt the Californian from paying the estate tax, we do not thereby entitle a
Filipino to be exempt from a similar estate tax in California because under the Federal Law,
which is equally enforceable in California he is bound to pay the same, there being no
reciprocity recognized in respect thereto. In both instances, the Filipino citizen is always at a
disadvantage. We do not believe that our legislature has intended such an unfair situation to the
detriment of our own government and people. We, therefore, find and declare that the lower
court erred in exempting the estate in question from payment of the inheritance tax.

We are not unaware of our ruling in the case of Collector of Internal Revenue vs. Lara (G.R.
Nos. L-9456 & L-9481, prom. January 6, 1958, 54 O.G. 2881) exempting the estate of the
deceased Hugo H. Miller from payment of the inheritance tax imposed by the Collector of
Internal Revenue. It will be noted, however, that the issue of reciprocity between the pertinent
provisions of our tax law and that of the State of California was not there squarely raised, and
the ruling therein cannot control the determination of the case at bar. Be that as it may, we now
declare that in view of the express provisions of both the Philippine and California laws that the
exemption would apply only if the law of the other grants an exemption from legacy, succession,
or death taxes of every character, there could not be partial reciprocity. It would have to be total
or none at all.

With respect to the question of deduction or reduction in the amount of P4,000.00 based on the
U.S. Federal Estate Tax Law which is also being claimed by respondents, we uphold and
adhere to our ruling in the Lara case (supra) that the amount of $2,000.00 allowed under the
Federal Estate Tax Law is in the nature of a deduction and not of an exemption regarding which
reciprocity cannot be claimed under the provision of Section 122 of our National Internal
Revenue Code. Nor is reciprocity authorized under the Federal Law. .

On the issue of the correctness of the appraisal of the two parcels of land situated in Baguio
City, it is contended that their assessed values, as appearing in the tax rolls 6 months after the
death of Stevenson, ought to have been considered by petitioner as their fair market value,
pursuant to section 91 of the National Internal Revenue Code. It should be pointed out,
however, that in accordance with said proviso the properties are required to be appraised at
their fair market value and the assessed value thereof shall be considered as the fair market
value only when evidence to the contrary has not been shown. After all review of the record, we
are satisfied that such evidence exists to justify the valuation made by petitioner which was
sustained by the tax court, for as the tax court aptly observed:

"The two parcels of land containing 36,264 square meters were valued by the
administrator of the estate in the Estate and Inheritance tax returns filed by him at
P43,500.00 which is the assessed value of said properties. On the other hand,
defendant appraised the same at P52,200.00. It is of common knowledge, and this Court
can take judicial notice of it, that assessments for real estate taxation purposes are very
much lower than the true and fair market value of the properties at a given time and
place. In fact one year after decedent's death or in 1952 the said properties were sold for
a price of P72,000.00 and there is no showing that special or extraordinary
circumstances caused the sudden increase from the price of P43,500.00, if we were to
accept this value as a fair and reasonable one as of 1951. Even more, the counsel for
plaintiffs himself admitted in open court that he was willing to purchase the said
properties at P2.00 per square meter. In the light of these facts we believe and therefore
hold that the valuation of P52,200.00 of the real estate in Baguio made by defendant is
fair, reasonable and justified in the premises." (Decision, p. 19).

In respect to the valuation of the 210,000 shares of stock in the Mindanao Mother Lode Mines,
Inc., (a domestic corporation), respondents contend that their value should be fixed on the basis
of the market quotation obtaining at the San Francisco (California) Stock Exchange, on the
theory that the certificates of stocks were then held in that place and registered with the said
stock exchange. We cannot agree with respondents' argument. The situs of the shares of stock,
for purposes of taxation, being located here in the Philippines, as respondents themselves
concede and considering that they are sought to be taxed in this jurisdiction, consistent with the
exercise of our government's taxing authority, their fair market value should be taxed on the
basis of the price prevailing in our country.

Upon the other hand, we find merit in respondents' other contention that the said shares of
stock commanded a lesser value at the Manila Stock Exchange six months after the death of
Stevenson. Through Atty. Allison Gibbs, respondents have shown that at that time a share of
said stock was bid for at only P.325 (p. 103, t.s.n.). Significantly, the testimony of Atty. Gibbs in
this respect has never been questioned nor refuted by petitioner either before this court or in the
court below. In the absence of evidence to the contrary, we are, therefore, constrained to
reverse the Tax Court on this point and to hold that the value of a share in the said mining
company on August 22, 1951 in the Philippine market was P.325 as claimed by respondents..

It should be noted that the petitioner and the Tax Court valued each share of stock of P.38 on
the basis of the declaration made by the estate in its preliminary return. Patently, this should not
have been the case, in view of the fact that the ancillary administrator had reserved and availed
of his legal right to have the properties of the estate declared at their fair market value as of six
months from the time the decedent died..

On the fifth issue, we shall consider the various deductions, from the allowance or disallowance
of which by the Tax Court, both petitioner and respondents have appealed..

Petitioner, in this regard, contends that no evidence of record exists to support the allowance of
the sum of P8,604.39 for the following expenses:.
1) Administrator's fee P1,204.34
2) Attorney's fee 6,000.00
3) Judicial and Administrative expenses 2,052.55
Total Deductions P8,604.39

An examination of the record discloses, however, that the foregoing items were considered
deductible by the Tax Court on the basis of their approval by the probate court to which said
expenses, we may presume, had also been presented for consideration. It is to be supposed
that the probate court would not have approved said items were they not supported by evidence
presented by the estate. In allowing the items in question, the Tax Court had before it the
pertinent order of the probate court which was submitted in evidence by respondents. (Exh.
"AA-2", p. 100, record). As the Tax Court said, it found no basis for departing from the findings
of the probate court, as it must have been satisfied that those expenses were actually incurred.
Under the circumstances, we see no ground to reverse this finding of fact which, under Republic
Act of California National Association, which it would appear, that while still living, Walter G.
Stevenson obtained we are not inclined to pass upon the claim of respondents in respect to the
additional amount of P86.52 for funeral expenses which was disapproved by the court a quo for
lack of evidence.

In connection with the deduction of P652.50 representing the amount of realty taxes paid in
1951 on the decedent's two parcels of land in Baguio City, which respondents claim was
disallowed by the Tax Court, we find that this claim has in fact been allowed. What happened
here, which a careful review of the record will reveal, was that the Tax Court, in itemizing the
liabilities of the estate, viz:

1) Administrator's fee P1,204.34


2) Attorney's fee 6,000.00
3) Judicial and Administration expenses as of
August 9, 1952 2,052.55
Total P9,256.89

added the P652.50 for realty taxes as a liability of the estate, to the P1,400.05 for judicial and
administration expenses approved by the court, making a total of P2,052.55, exactly the same
figure which was arrived at by the Tax Court for judicial and administration expenses. Hence,
the difference between the total of P9,256.98 allowed by the Tax Court as deductions, and the
P8,604.39 as found by the probate court, which is P652.50, the same amount allowed for realty
taxes. An evident oversight has involuntarily been made in omitting the P2,000.00 for funeral
expenses in the final computation. This amount has been expressly allowed by the lower court
and there is no reason why it should not be. .

We come now to the other claim of respondents that pursuant to section 89(b) (1) in relation to
section 89(a) (1) (E) and section 89(d), National Internal Revenue Code, the amount of
P10,022.47 should have been allowed the estate as a deduction, because it represented an
indebtedness of the decedent incurred during his lifetime. In support thereof, they offered in
evidence a duly certified claim, presented to the probate court in California by the Bank of
California National Association, which it would appear, that while still living, Walter G.
Stevenson obtained a loan of $5,000.00 secured by pledge on 140,000 of his shares of stock in
the Mindanao Mother Lode Mines, Inc. (Exhs. "Q-Q4", pp. 53-59, record). The Tax Court
disallowed this item on the ground that the local probate court had not approved the same as a
valid claim against the estate and because it constituted an indebtedness in respect to
intangible personal property which the Tax Court held to be exempt from inheritance tax.

For two reasons, we uphold the action of the lower court in disallowing the deduction.

Firstly, we believe that the approval of the Philippine probate court of this particular
indebtedness of the decedent is necessary. This is so although the same, it is averred has been
already admitted and approved by the corresponding probate court in California, situs of the
principal or domiciliary administration. It is true that we have here in the Philippines only an
ancillary administration in this case, but, it has been held, the distinction between domiciliary or
principal administration and ancillary administration serves only to distinguish one administration
from the other, for the two proceedings are separate and independent. 8 The reason for the
ancillary administration is that, a grant of administration does not ex proprio vigore, have any
effect beyond the limits of the country in which it was granted. Hence, we have the requirement
that before a will duly probated outside of the Philippines can have effect here, it must first be
proved and allowed before our courts, in much the same manner as wills originally presented for
allowance therein.9 And the estate shall be administered under letters testamentary, or letters of
administration granted by the court, and disposed of according to the will as probated, after
payment of just debts and expenses of administration. 10 In other words, there is a regular
administration under the control of the court, where claims must be presented and approved,
and expenses of administration allowed before deductions from the estate can be authorized.
Otherwise, we would have the actuations of our own probate court, in the settlement and
distribution of the estate situated here, subject to the proceedings before the foreign court over
which our courts have no control. We do not believe such a procedure is countenanced or
contemplated in the Rules of Court.

Another reason for the disallowance of this indebtedness as a deduction, springs from the
provisions of Section 89, letter (d), number (1), of the National Internal Revenue Code which
reads:

(d) Miscellaneous provisions — (1) No deductions shall be allowed in the case of a non-
resident not a citizen of the Philippines unless the executor, administrator or anyone of
the heirs, as the case may be, includes in the return required to be filed under section
ninety-three the value at the time of his death of that part of the gross estate of the non-
resident not situated in the Philippines."

In the case at bar, no such statement of the gross estate of the non-resident Stevenson not
situated in the Philippines appears in the three returns submitted to the court or to the office of
the petitioner Collector of Internal Revenue. The purpose of this requirement is to enable the
revenue officer to determine how much of the indebtedness may be allowed to be deducted,
pursuant to (b), number (1) of the same section 89 of the Internal Revenue Code which
provides:

(b) Deductions allowed to non-resident estates. — In the case of a non-resident not a


citizen of the Philippines, by deducting from the value of that part of his gross estate
which at the time of his death is situated in the Philippines —
(1) Expenses, losses, indebtedness, and taxes. — That proportion of the deductions
specified in paragraph (1) of subjection (a) of this section 11 which the value of such part
bears the value of his entire gross estate wherever situated;"

In other words, the allowable deduction is only to the extent of the portion of the indebtedness
which is equivalent to the proportion that the estate in the Philippines bears to the total estate
wherever situated. Stated differently, if the properties in the Philippines constitute but 1/5 of the
entire assets wherever situated, then only 1/5 of the indebtedness may be deducted. But since,
as heretofore adverted to, there is no statement of the value of the estate situated outside the
Philippines, no part of the indebtedness can be allowed to be deducted, pursuant to Section 89,
letter (d), number (1) of the Internal Revenue Code.

For the reasons thus stated, we affirm the ruling of the lower court disallowing the deduction of
the alleged indebtedness in the sum of P10,022.47.

In recapitulation, we hold and declare that:

(a) only the one-half (1/2) share of the decedent Stevenson in the conjugal partnership
property constitutes his hereditary estate subject to the estate and inheritance taxes;

(b) the intangible personal property is not exempt from inheritance tax, there existing no
complete total reciprocity as required in section 122 of the National Internal Revenue
Code, nor is the decedent's estate entitled to an exemption of P4,000.00 in the
computation of the estate tax;

(c) for the purpose of the estate and inheritance taxes, the 210,000 shares of stock in
the Mindanao Mother Lode Mines, Inc. are to be appraised at P0.325 per share; and

(d) the P2,000.00 for funeral expenses should be deducted in the determination of the
net asset of the deceased Stevenson.

In all other respects, the decision of the Court of Tax Appeals is affirmed.

Respondent's claim for interest on the amount allegedly overpaid, if any actually results after a
recomputation on the basis of this decision is hereby denied in line with our recent decision
in Collector of Internal Revenue v. St. Paul's Hospital (G.R. No. L-12127, May 29, 1959)
wherein we held that, "in the absence of a statutory provision clearly or expressly directing or
authorizing such payment, and none has been cited by respondents, the National Government
cannot be required to pay interest."

WHEREFORE, as modified in the manner heretofore indicated, the judgment of the lower court
is hereby affirmed in all other respects not inconsistent herewith. No costs. So ordered.

Paras, C.J., Bengzon, Bautista Angelo, Labrador, Concepcion, Reyes, J.B.L., Gutierrez David,
Paredes and Dizon, JJ., concur.

Footnotes
1
ART. 124. If the marriage is between a citizen of the Philippines and a foreigner,
whether celebrated in the Philippines or abroad, the following rules shall prevail: (1) If
the husband is a citizen of the Philippines while the wife is a foreigner, the provisions of
this Code shall govern their property relations; (2) If the husband is a foreigner and the
wife is a citizen of the Philippines, the laws of the husband's country shall be followed,
without prejudice to the provisions of this Code with regard to immovable property."
2
ART. 1325. Should the marriage be contracted in a foreign country, between a
Spaniard and a foreign woman or between a foreigner and a Spanish woman, and the
contracting parties should not make any statement or stipulation with respect to their
property, it shall be understood, when the husband is a Spaniard, that he marries under
the system of the legal conjugal partnership, and when the wife is a Spaniard, that she
marries under the system of law in force in the husband's country, all without prejudice to
the provisions of this code with respect to real property. .
3
IX Manresa, Comentarios al Codigo Civil Espanol, p. 209. .
4
Yam Ka Lim vs. Collector of Customs, 30 Phil. 46; Lim & Lim vs. Collector of Customs,
36 Phil. 472; International Harvester Co. vs. Hamburg-American Line, 42 Phil. 845;
Beam vs. Yatco, 46 O.G. No. 2, p. 530.).
5
Lim vs. Collector of Customs, supra; International Harvester Co. vs. Hamburg-
American Line, supra; Phil. Manufacturing Co. vs. Union Ins. Society of Canton, 42 Phil.
378; Adong vs. Cheong Seng Gee, Phil. 53.
6
Sy Joc Leing vs. Sy Quia, 16 Phil. 138; Ching Huat vs. Co Heong, 77 Phil. 985; Adong
vs. Cheong supra.
7
See Sec. 860, Internal Revenue Code of 1939, 26 USCA 408.
8
In the matter of the testate estate of Basil Gordon Butler, G.R. No. L-3677, Nov. 29,
1951. .
9
Rule 78, Sees. 1, 2 and 3, Rules of Court. See also Hix vs. Fluemer, 54 Phil. 610. .
10
Rule 78, See. 4, lbid.
11
Expense, losses, indebtedness, and taxes which may be deducted to determine the
net estate of a citizen or resident of the Philippines.

HARDEN VS PENA

Republic of the Philippines


SUPREME COURT
Manila

EN BANC
G.R. No. L-1499 November 21, 1950

FRED M. HARDEN, ET AL., petitioners,


vs.
EMILIO PEÑA, ET AL., respondents.

Vicente J. Francisco for petitioners.


The respondent Judge in his own behalf.
Claro M. Recto for respondents.

BENGZON, J.:

When the husband commits frauds against the wife in administration of their conjugal
partnership, what remedies are available to her? That is the question involved in this petition
for certiorari to annul several orders promulgated by the respondent judge and his predecessor
in civil case No. 59634 of the Manila court of first instance.

The new Civil Code contains an appropriate answer; but adjudication of this controversy must
be made according to the principles prevailing before the approval of such recent legislation that
is not retroactive in general.

In December, 1917, and in the City of Manila, Fred M. Harden, an American citizen, domiciled in
the Philippines married Esperanza Perez, a native-born Filipina. Thereafter they engaged in
business and acquired considerable property, personal and real, until the year 1938 when they
stopped living together.

On July 12, 1941, she commenced the aforesaid action in which, alleging unjust abandonment
by her husband, and resulting mismanagement of the business by him and Jose Salumbides his
attorney-in-fact, aggravated by fraudulent transfers and conversions of conjugal property to
prejudice her interest, she asked (1) that she be awarded a monthly alimony; (2) that she be
permitted to participate in the management of the conjugal partnership; (3) that Fred be ordered
to account for and to return to the Philippines the sum of P449,015.44 he had fraudulently
transferred to Hongkong; (4) that Salumbides be required to account for P285,000 belonging to
the partnership; and (5) that certain transfers of mining shares made by Fred be avoided.
Esperanza further prayed for preliminary injunction restraining defendants from making any
transfer of partnership property without the consent of the court.

The injunction was issued upon the filing of a nominal bond. On August 18, 1941, the
defendants moved for its dissolution upon the ground that the complaint stated no cause of
action. Esperanza objected, and the motion remained unacted due to the Pacific War.

Meantime, in October, 1941, Fred answered the complaint, denying his wife's charges of
desertion and deception. He claimed she had no cause of action, inasmuch as their marriage
was still subsisting. And in the matter of properties he invoked his domicile in New Jersey,
U.S.A., according to whose statutes the properties enumerated in the complaint belonged
exclusively to him.

During the Japanese occupation proceedings in the litigation were suspended by virtue of
Instruction No. 28 of the Japanese Military Administration. After the liberation, i.e., on May 15,
1946, Esperanza moved for reconstitution of the records, which has been destroyed during the
battle of Manila; and on October 23, 1946, the respondent judge declared the partial
reconstitution thereof.

A few days later, on November 12, 1946, Esperanza filed a petition for the appointment of a
receiver to take charge of all the belongings of the marital association, because she had
discovered that between March 23, and July 23, 1946, Fred and Salumbides had surreptitiously
remitted to Hongkong more than P220,000 notwithstanding the injunction issued in July, 1941.
The respondent judge, favorably acting on said petition, appointed Abelardo Perez as receiver
with a bond of P10,000.

On March 8, 1947, the defendants prayed for discharge of the receivership. Fred alleged he did
not abandon his wife, having made liberal provisions for her. He affirmed he never intended to
deprive her of any rights, and had given no cause for divorce; and that although he had
evidence of her adultery with John Hart, he continued to provide for her welfare.

On May 24, 1947, the respondent judge denied the motion for dissolution of the injunction of
July 12, 1941. And on September 3, 1947, he denied the motion to discharge the receivership.

This special civil suit is now submitted mainly to question the authority of the trial court to issue
the writ of injunction and to establish the receivership. There are other minor issues, like the
payment of litigation expenses of Esperanza and the allowances for her maintenance; but these
shall be considered later.

Challenging the legality of the injunction, the petitioner Harden quotes our several decisions
holding repeatedly that the interest of the wife in the conjugal partnership prior to its liquidation
is "inchoate, a mere expectancy, which constitutes neither a legal nor an equitable estate, and
does not ripen into title until there appears that there are assets in the community as a result of
the liquidation and settlement".1 He argues that inasmuch as Esperanza no title to the properties
of the partnership, (her rights being dormant or inert) she may not apply for injunction to protect
such mere incipient or dormant interest, until dissolution of the marriage occurs.

He particularly invokes the decisions in Saavedra vs. Ibañez (56 Phil., 33), and Baello vs.
Villanueva (54 Phil., 213) to support his view that "injunction can not be availed of by a wife who
seeks to prevent the defraudation of her rights to the conjugal properties."

The petitioner Harden also cites several decisions in the United States wherein courts refused
to interfere with the husband's reckless and extravagant expenses," (Garrozi vs. Dastas, 204
U.S., 64; 51 Law. ed., 369) holding that he may spend the community property substantially as
he chooses, and if he wastes it in debauchery, the wife has no redress. 2

In reply, the respondents explain that the statements in the decision invoked by petitioner to the
effect that the wife's interest "is inchoate or mere expectancy" were plain obiter dictum, and that,
anyway, such holding was corrected or amplified in Gibbs vs. Government of the Philippine
Islands(59 Phil., 293). They also argue that those decisions only mean that the wife may not
ordinarily prevent or annul the conveyance, by the husband, of specific pieces of the conjugal
property, (because her title to those pieces has not yet vested) — which is not the case here.

As to the Saavedra and Baello decisions, respondents contend that injunction was denied
therein because it was unnecessary; whereas, in this litigation such remedy is necessary, in
view of the kind of property involved and the circumstances disclosed.

Respondents claim that Garrozi vs. Dastas was overruled by Arnett vs. Reade (55 Law. Ed.,
478), and that U.S. vs. Robbins was founded upon a statute in California proclaiming the
husband's power of absolute disposition. And lastly they offer authorities and arguments to
prove that the remedy of injunction is available to the wife to prevent her husband from further
fraudulent disposition of conjugal assets.

The points in contestation have been thoroughly discussed in the pleadings and memoranda of
counsel on both sides.

As the marriage of the Hardens was contracted in Manila where Harden was domiciled and as
the parties made no stipulation regarding their system of ownership, the spouses should be
deemed to have adopted the legal conjugal partnership.3 All property acquired during the
marriage is presumed to be conjugal unless the contrary is proved. 4

The husband is the manager. He may for available consideration alienate and encumber their
property without the consent of the wife. But alienations in fraud of the wife shall not prejudice
her or her heirs.5

Fully aware of the above provisions, specially article 1413, petitioner Harden agrees to
respondents' thesis that when, in his administration of the conjugal affairs the husband makes
fraudulent conveyances prejudicial to the wife, the law affords the latter certain remedies. What
are these? The disagreement begins here. According to petitioners, she may annul the
alienations after the conjugal partnership has been liquidated or else procure an injunction when
she starts divorce proceedings against the husband.

According to respondents, the wife may ask for injunction and receivership, the identical
remedies Esperanza has employed.

The decisions of this Court seem to have recognized that the wife has at last three courses of
action: (a) suit to annul the fraudulent alienation; (b) action for separate maintenance or divorce
with preliminary injunction; and (c) suit to convert the conjugal estate into an ordinary tenancy in
common.

In De la Viña vs. Villareal (41 Phil., 13), we upheld an injunction issued to preserve the wife's
rights to conjugal partnership property and to curtail the husband's power of disposition, saying:

The law making the husband the sole administrator of the property of the conjugal
partnership is founded upon necessity and convenience as well as upon the
presumption that, from the very nature of the relation between husband and wife, the
former will promote and not injure the interest of the latter. So long, as this harmonious
relation, as contemplated by law, continues, the wife cannot and should not interfere with
the husband in his judicious administration of the conjugal property. But when that
relation ceases and, in a proper action, the wife seeks to dissolve the marriage and to
partition the conjugal property, it is but just and proper, in order to protect the interests of
the wife, that the husband's power of administration be curtailed during the pendency of
the action insofar as alienating or encumbering the conjugal property is concerned.
True that the injunction was issued in an action by the wife to dissolve the marriage; but the
decision did not say that the equitable power of injunction may only be exercised when divorce
or liquidation proceedings are started. The divorce proceeding in that litigation was an additional
circumstance lending validity to the injunction. It was not the condition sine qua non. The
divorce proceedings patently showed a cessation of those normal married relations in which the
husband's powers of disposition must be left undisturbed.

Yet divorce is not the only situation where such normal relations have ended. Where, as in this
case, the wife is abandoned and purposely kept in the dark about the affairs of the community
and is victimized by the fraudulent conspiracy between Fred and Salumbides, the harmonious
relations vanish, and the urgency of protecting the wife's interest by injunction becomes
apparent. Even supposing that proceedings for the separation of the spouses are essential to
the issuance of an injunction, the record shows that on November 6, 1941, Esperanza amended
her complaint announcing her determination to sue for divorce and to demand the liquidation of
the conjugal partnership properties. That should have cured any deficiency in the writing of the
injunctive decree.

The petitioner cites the precedent of Saavedra vs. Ibañez, supra, wherein the wife was in
danger of being cheated by probable future conduct of her husband in alienating conjugal
property, unless some means were adopted to safeguard her interests, The court of first
instance declined to grant an injunction, and this Supreme Court approved the denial, saying
that a mere annotation of the suit in the registry of property was enough. However, this
precedent merely shows, in our opinion, either that the Supreme Court did not desire to interfere
with the lower court's discretion in refusing to grant a preliminary injunction or that the Supreme
Court believed a mere registry was ample protection for the wife.

The precarious position of respondent Esperanza Harden is entirely different. The trial judges
were convinced of the necessity of the injunction and authorized it, and reasonable ground
therefor is apparent, considering the petitioner's conduct in making remittances to foreign
countries of substantial amounts of money and securities totalling P449,015.44 in addition to
other deceptions practiced upon her. Surely these expenditures may not be impeded by mere
annotations in the registry.

Defraudations of the wife are alleged to be the following: (a) the fraudulent conspiracy between
the two petitioners to remove funds of the couple from the Philippines to Hongkong; (b) the
deposit of funds and share of stock of said conjugal partnership in the name of third persons to
deceive the respondent Esperanza Harden; (c) and the gratuitous transfers of conjugal
securities to some foreign residents in pursuance of the same tricky combination.

Additional subsequent defraudations are specifically alleged in the consolidated amended


complaint as follows:

Amount
Date 1946 withdrawn Transferred to

March 23 P20,000.00 Hongkong and Shanghai Banking Corp.,


..................... Hongkong

July 23 202,042.90 Do
.........................

September 20 20,196.80 Virginia Recreation Center, Long Beach,


............. California

November 2 50,000.00 Unknown


.............…..

November 203,833.64 Chartered Bank of India, Australia and China,


13...........……. Hongkong

November 21 504,531.32 Do.


..............

Total P1,000,608.66

In those states of the American Union where the community property system
obtains,6 (California, Texas, Washington), the wife during marriage may resort to appropriate
judicial remedies — like injunction — against the husband's fraudulent acts. The courts of equity
will afford adequate relief, and the husband may be restrained from engaging in transactions
involving community property which are clearly inimical to the welfare of the community. 7

In Stewart vs. Stewart (249 Pac. 196), the Supreme Court of California, speaking of the wife's
right to protect her share of the conjugal partnership, said:

. . . She has, by virtue of the share which in her own sphere she has contributed toward
the acquisition and conservation of such properties, right therein which have been
always safeguarded against the fraudulent or inconsiderate acts of her husband with
relation thereto, and for the assertion and safeguarding of which she has been given
access to appropriate judicial remedies both before and after the time when her said
rights and interests would ripen and become vested through the death of the husband or
other severance of the marriage relation, whenever such rights and ultimate interests
were affected by or threatened with such forms of invasion. . . . (Emphasis supplied.)

And in this contrary, it has been held that —

The power to grant preliminary injunctions, both preventive and mandatory, is a logical
and necessary incident of the general powers conferred upon Courts of First Instance in
these Islands, as courts of record of general and unlimited original jurisdiction, both legal
and equitable.

In so far, as the statute limits or prescribes the exercise of this power it must be followed;
but beyond this and in cases not covered by or contemplated by the statute, these courts
must exercise their jurisdiction in the issuance of preliminary injunctions upon sound
principles applicable to the circumstances of each particular case, having in mind the
nature of the remedy and the doctrine and practice established in the courts upon which
our judicial system is modelled. (Manila Electric vs. Del Rosario, 22 Phil., 433; De la
Vina vs. Villareal, 41 Phil., 23-24.)

We conclude on this phase of the controversy that the injunction of July, 1941, should not be
dissolved nor declared void.

Concerning the receivership, the petitioner maintains it is improper because(1) it wrested or


purported to wrest from Fred's hands the possession of conjugal properties, the administration
of which is expressly vested in him by the Code, and (2) he being an American permanently
domiciled in New Jersey, the laws of said state determine the status of their marital belongings.

It is believed the respondents are correct in contending that the receivership does not alter the
right of possession, and that, anyhow, it is a temporary expedient, permitted on equitable
grounds as the most convenient and feasible means of administering or disposing of the
property in litigation for the mutual benefit of the contending parties.

And, notwithstanding petitioner's assertions of permanent domicile in New Jersey to escape the
effects of the Philippine partnership system, the papers seems to indicate he was domiciled in
Manila. At any rate, the lower so found, and this is hardly the proceeding to overturn such
finding of fact.

It may be observed in this connection that the New Civil Code permits the courts to grant relief
to the wife in case of abuse of powers by the husband in the administration of the conjugal
partnership property.

ART. 167. In case of abuse of powers of administration of the conjugal partnership


property by the husband, the courts, on petition of the wife, may provide for a
receivership, of administration by the wife, or separation of property." (New Civil Code.).

ART. 173. The wife may, during the marriage, and within ten years from the transaction
questioned, ask the courts for the annulment of any contract of the husband entered into
without her consent, when such consent is required, or any act or contract of the
husband which tends to defraud her or impair her interest in the conjugal partnership
property. . . . (New Civil Code.)

ART. 178. The separation in fact between husband and wife without judicial approval,
shall not affect the conjugal partnership, except that:

xxx xxx xxx

If the husband has abandoned the wife without just cause for at least one year, she may
petition the court for a receivership, or administration by her of the conjugal partnership
property, or separation of property. (New Civil Code.)

The above express no more than the concepts of equity and justice applied in the various
decisions we have previously adverted to. More, they reflect unmistakable intuitions of
enlightened policy corresponding with the dictates of social convenience, to which court
decisions must necessarily be adjusted whenever possible.

Other decrees of the respondent judge which are challenged by herein petitioner Harden are
these: (a) April 5, 1947, requiring him to pay the transportation expenses to Manila of
Esperanza Harden (b) May 30, 1947, requiring him to her P10,000 for services of her lawyer at
the taking of the deposition which petitioner intends to take in New Jersey; and (c) July 12,
1947, requiring him to spend out of conjugal money a monthly allowance of P2,500 for
Esperanza as alimony pendente lite.

There is no necessity at this time to discuss the validity of such directives, because the
petitioner may raise the same questions in an appeal he may interpose from the decision of the
main case. Anyway, the big amount of property in litigation being unquestioned, any advances
made to the wife will certainly be amply guaranteed by her portion of the marital possessions.
Petition denied, with costs.

Moran, C.J., Paras, Feria, Pablo, Tuason, Montemayor, and Reyes, JJ., concur.

Footnotes
1
Nable Jose, et al., vs. Nable Jose, et al., 41, Phil., 713; Manuel, et al., vs. Losano, et
al., 41 Phil., 855, etc.
2
U.S. vs. Robbins, 70 L. Ed., 285.
3
Article 1315. old Civil Code.
4
Article 1407, supra; Casiano vs. Samaniego (30 Phil., 135); Sison vs. Ambalada (30
Phil., 118).
5
Articles 1412, 1413, supra.
a Uy Coque et al., vs. Sioca, et al., (45 Phil., 430); Gallion vs. Gayares, et al. (53
Phil., 43).
b De la Viña vs. Villareal (41 Phil., 553).
c Aenlle vs. Bertrand Rheims (52 Phil., 553).
6
Osorio vs. Posadas, 56 Phil., 748.
7
41 C.J.S., p. 1976, citing Johnson vs. National Surety Co. (5 P., 2d, 39; 118 Cal. App.,
227); Weir vs. King (Civ. App., 166 S.W., 2d 187); Inre Coffey's Estate, (81 P. 2d, 283;
195 Wash., 379).

Das könnte Ihnen auch gefallen